mcqs pediatrics

155
MCQs PEDIATRICS Dr.Kamel Youssef Hassan, Pediatrician Consultant , Palestine - Gaza E-mail: [email protected] 1 MCQs PEDIATRICS Prepared by Dr.Kamel Hassan MD C.E.S de Pediatrie, C.P.B in Pediatrics Palestine – Gaza E-mail: [email protected]

Upload: ahmed-kh-abu-warda

Post on 03-Jan-2016

1.300 views

Category:

Documents


1 download

TRANSCRIPT

Page 1: Mcqs Pediatrics

MCQs PEDIATRICS

Dr.Kamel Youssef Hassan, Pediatrician Consultant , Palestine - Gaza E-mail: [email protected] 1

MCQsPEDIATRICS

Prepared byDr.Kamel Hassan MD

C.E.S de Pediatrie, C.P.B in PediatricsPalestine – Gaza

E-mail: [email protected]

Page 2: Mcqs Pediatrics

MCQs PEDIATRICS

Dr.Kamel Youssef Hassan, Pediatrician Consultant , Palestine - Gaza E-mail: [email protected] 2

1- DEVELOPMENT1-Theme : STATUREA. Achondroplasia B. Anorexia nervosa C. Constitutional delayD. Cornelia-de-Lange syndromeE. Familial short statureF. Growth hormone deficiencyG. Primordial dwarfismH. Psychosocial poor growthI. Prader-Willi syndromeJ. Russell-silver dwarfismSelect the most appropriate diagnosis from the above options to explain the following presentations:1) A child has short stature, a triangular face and facial and limb length asymmetry. J. Russell-silver dwarfismNote:describes a child with Russell –silver syndrome. A condition of short stature, hemi-hypertrophy and the child often has triangular facies, frontal bossing and clinodactyly of the 5th finger.

2) A child has short stature and is noted to have a large head with a prominent forehead.A. Achondroplasia Note:describes a child with achondroplasia. Short stature, rhizomelia (short limbs especially the proximal part). Affected individuals usually have a large head with hydrocephalus being a complication.

3) A baby is born on the 50th centile for height and weight. By the age of 2 her length has fallen to the 2nd centile and weight remains on the 50th. She has a rounded face with small hands and feet.F. Growth hormone deficiencyNote:describes a growth hormone deficient child. Early recognition and initiation of treatment is required to reach satisfactory adult height.

Page 3: Mcqs Pediatrics

MCQs PEDIATRICS

Dr.Kamel Youssef Hassan, Pediatrician Consultant , Palestine - Gaza E-mail: [email protected] 3

2-Theme : STATUREA. Achondroplasia B. Anorexia nervosa C. Constitutional delayD. Cornelia-de-Lange syndromeE. Familial short statureF. Growth hormone deficiencyG. Primordial dwarfismH. Psychosocial poor growthI. Prader-Willi syndromeJ. Russell-silver dwarfismSelect the most appropriate diagnosis from the above options to explain the following presentations:1) A 9 year old boy statemented for special educational needs is investigated for short stature. He has a weight over the 91st centile. He is noted to have small hands and feet and hypogonadism.I. Prader-Willi syndromeNote:describes a child with Prader-Willi syndrome. A condition characterised by obesity after the first year of life. Hypotonia, hypogonadism and learning difficulties due to a deletion of chromosome 15 (the deletion occurring in the paternal gene, if the deletion of chromosome 15 in the child was inherited from the mother the child would have the phenotype of Angelman's syndrome).2) A 15 year old girl in foster care is referred to outpatients because of delayed puberty. On examination her weight is on the 2nd centile and her height is on the 25th centile. She has poor dentition and her temperature is measured at 35°C.B. Anorexia nervosaNote:describes an adolescent girl with anorexia nervosa. A condition due to fear of becoming obese or disturbed body image. There is usually a tendency to deny hunger, excessive dieting, laxative abuse or excessive physical activity. The diagnosis is classified as weight of 15% below that expected for age and height and is associated with physical characteristics including amenorrhoea, bradycardia, hypothermia and electrolyte disturbances.3) A 7 year old girl presents with short stature with weight being on the 9th centile and height less than 0.4th centile. Examination is normal as well as all investigations including a bone age, which is compatible with her chronological age. Her mother is 5”1 (134cm), her father is 5”5 (143cm).E. Familial short statureNote:describes a child with familial short stature. From the measurements of the parents the child's height is compatible with this diagnosis. It is expected that most children will reach a height within 8cms of the mid-parental height centile.

Page 4: Mcqs Pediatrics

MCQs PEDIATRICS

Dr.Kamel Youssef Hassan, Pediatrician Consultant , Palestine - Gaza E-mail: [email protected] 4

2- CARDIOLOGY1-Theme : EMERGENCY MEDICINEA. Adenosine B. AdrenalineC. AtropineD. DC shock E. Dobutamine F. Diving reflex G. Endotracheal intubationH. Intraosseous lineI. IV MorphineJ. NaloxoneSelect the most appropriate emergency treatment for the following children:1) A 4-year-old child is rescued from a house fire. She is admitted tachypnoeic and tacchycardic. She has soot in her nostrils.G. Endotracheal intubationNote: Item 1 relates to burns. Most deaths following house fires occur secondary to smoke inhalation. Amongst the indicators of inhaled smoke injury is deposits around the mouth and nose. Oedema follows thermal injury and therefore any suspicion of airway compromise should result in endotracheal intubation.

2) A 3 month old baby is admitted with a history of poor feeding. On arrival he has a pulse rate of 220 beats per minute.F. Diving reflexNote: Supraventricular tachycardia is the diagnosis in Item 3. Vagal stimulation is the treatment of choice and the diving reflex is the simple procedure elicited by submerging the baby's face in to ice or placing an ice bag over the face. The diving reflex increases vagal tone, slows AV conduction interrupting the tachycardia.

3) A 13 year old boy is admitted with meningococcaemia. He is in shock. Peripheral cannulation is difficult.H. Intraosseous lineNote: Item 2 relates to a child in shock. In many life-threatening conditions venous cannulation is difficult. It is important to obtain vascular access very quickly and therefore intraosseous infusion is recommended.

Page 5: Mcqs Pediatrics

MCQs PEDIATRICS

Dr.Kamel Youssef Hassan, Pediatrician Consultant , Palestine - Gaza E-mail: [email protected] 5

2-Theme : SYNDROMES ASSOCIATED WITH CONGENITAL HEART DISEASEA. Charcot-Marie- Tooth syndromeB. Down syndromeC. Fragile X syndromeD. Marfan's syndromeE. Noonan syndromeF. Sturge-Weber syndromeG. Tetrology of FallotH. Tourette syndromeI. Turner's syndromeJ. William's syndromeFor each of the cardiac lesions described below, choose the most commonly associated syndrome from the above list of options. Each option may be used once or not at all.1) Supravalvular aortic stenosisJ. William's syndromeNote:William's syndrome � is characterized by short stature, characteristic facies, supravalvular aortic stenosis, mild to moderate learning difficulties and transient neonatal hypercalcemia.

2) Coarctation of AortaI. Turner's syndromeNote:Turner's syndrome � This is characterized by 45, X genotype, ovarian dysgenesis leading to infertility, short stature, webbing of the neck, wide carrying angles and wide spaced nipples. However they have normal intellectual development.

3) Dilatation of aortic root/aortic regurgitationD. Marfan's syndromeNote:Marfan's syndrome � This is an autosomal dominant disorder. The clinical features are tall stature, arachno-dactyly, high arched plate and increase in length of the lower segment of the body compared to the upper segment. The cardiac manifestations include dilated aortic root, aortic incompetence, mitral valve prolapse and mitral incompetence.

Page 6: Mcqs Pediatrics

MCQs PEDIATRICS

Dr.Kamel Youssef Hassan, Pediatrician Consultant , Palestine - Gaza E-mail: [email protected] 6

4) Cardiac cushion defects (leading to ASD, VSD)B. Down syndromeNote:Down syndrome � Features include characteristic facies, hypotonia, severe learning difficulties and small stature. About 40% of patients have cardiac anomalies mainly endocardial cushion defects leading to ASD and VSD.

5) Infundibular pulmonary stenosisG. Tetrology of FallotNote:Tetrology of Fallot This is a cyanotic heart disease and the cardinal features include:

1. Infundibular pulmonary stenosis2. VSD 3. Right ventricular hypertrophy

Over-riding of the aorta

Comments:Charcot-Marie-Tooth syndrome Autosomal dominant peroneal muscular dystrophy.

Fragile X syndrome Moderate learning difficulty, macrocephaly, characteristic facies (long face, large ears, prominent mandible and forehead)Noonan syndrome� Facies, mild learning difficulties, short webbed neck, short stature and congenital heart disease (pulmonary valvular stenosis, ASD, left ventricular hypertrophy)

Sturge-Weber syndrome Haemangiomas in the distribution of trigeminal nerve and in the brain. Sometimes can have intractable epilepsy.

Tourette syndrome Tics, compulsive utterances of obscene words (coprolalia)

Page 7: Mcqs Pediatrics

MCQs PEDIATRICS

Dr.Kamel Youssef Hassan, Pediatrician Consultant , Palestine - Gaza E-mail: [email protected] 7

3-Theme : EMERGENCY TREATMENTSA. Adenosine B. AdrenalineC. AtropineD. DC shock E. DobutamineF. Diving reflex G. Endotracheal intubationH. Intraosseous lineI. IV MorphineJ. NaloxoneSelect the most appropriate emergency treatment for the following children:1) A 5 year old boy is brought to the hospital with 15% scalds to his chest.I. IV MorphineNote:Item 1 relates to a child with serious burns. Children who have been burnt are in severe pain and therefore IV Morphine is the analgesic of choice.

2) A 14-year-old girl with a history of previous overdoses is admitted to A&E apnoeic and unconsciousness. ECG shows ventricular fibrillation. CPR is commenced.D. DC shockNote:Item 2 describes a child in ventricular fibrillation. This is uncommon in childhood although may occur as a result of tricyclic antidepressant overdose and hypothermia. If the arrest is witnessed a precordial thump is carried out otherwise electrical de-fibrillation at 2 joules per kilogram.

3) A term baby is born in poor condition. Apgar scores 3 at 1 minute and 5 at 5 minutes. CPR is commenced. At 10 minutes he remains bradycardic.B. AdrenalineNote:Item 3 describes a baby born in poor condition. A bradycardia in an unstable newborn requires oxygenation, ventilation and cardiac compressions. IV adrenaline is administered as Atropine is ineffective in this age group.

Page 8: Mcqs Pediatrics

MCQs PEDIATRICS

Dr.Kamel Youssef Hassan, Pediatrician Consultant , Palestine - Gaza E-mail: [email protected] 8

4-Theme : GENETICS - CARDIAC ABNORMALITIES IN GENETIC DISORDERSA. Angelman's syndromeB. Beckwith-Wiedemann syndromeC. Congenital Rubella syndromeD. Down syndromeE. Foetal alcohol syndromeF. Glycogen storage diseaseG. Marfan's syndromeH. Noonan's syndromeI. Turner's syndromeJ. Williams syndrome

Match each of the following cardiovascular abnormalities to the single most likely associated genetic disorder.1) Dilation of the aorta with aneurysms.G. Marfan's syndromeNote:In Marfan's syndrome dilatation of the ascending aorta is often seen with or without aneurysms. Less commonly the thoracic abdominal aorta or pulmonary arteries are affected with secondary aortic regurgitation and mitral valve prolapse.

2) Supra-valvular aortic stenosis.J. Williams syndrome Note:In Williams syndrome supra-valvular aortic stenosis is the most common cardiac lesion. Septal defects also occur as well as peripheral branch pulmonary artery stenosis.

3) Pulmonary stenosis.H. Noonan's syndrome Note:In Noonan's syndrome pulmonary valve stenosis due to a dysplastic or thick valve is seen often associated with left ventricular hypertrophy. Branch stenosis of the pulmonary artery also is found in Noonan's syndrome.

Page 9: Mcqs Pediatrics

MCQs PEDIATRICS

Dr.Kamel Youssef Hassan, Pediatrician Consultant , Palestine - Gaza E-mail: [email protected] 9

5-Theme : CONGENITAL CARDIAC DEFECTSA. Angelman's syndromeB. Beckwith-Wiedemann syndromeC. Congenital Rubella syndromeD. Down syndromeE. Foetal alcohol syndromeF. Glycogen storage diseaseG. Marfan's syndromeH. Noonan's syndromeI. Turner's syndromeJ. Williams syndromeMatch each of the following cardiovascular abnormalities to the single most likely associated disorder.1) Endocardial cushion defect.D. Down syndromeNote:In Down syndrome approximately 40% of children have a congenital heart disease. The most common being endocardial cushion defects although VSDs, ASDs and PDA also occur.

2) Coarctation of the aorta.I. Turner's syndromeNote:In Turner's syndrome cardiac defects are common. 30% include bicuspid aortic valves with the second most common heart defect being coarctation of the aorta. Aortic stenosis, mitral valve prolapse and hypertension are also found.

3) Septal defects.E. Foetal alcohol syndromeNote:In foetal alcohol syndrome individuals have poor growth, developmental delay and usually characteristic facial features including microcephaly and a short smooth philtrum. The most common cardiac lesion in these children are septal defects primarily ventricular septal defects.

Page 10: Mcqs Pediatrics

MCQs PEDIATRICS

Dr.Kamel Youssef Hassan, Pediatrician Consultant , Palestine - Gaza E-mail: [email protected] 10

6-Theme : BREATHLESSNESSA. Asthma B. HyperventilationC. Tuberculosis D. Cystic Fibrosis E. Pneumocystis cariniiF. Ventricular septal defect G. Gastroesophageal refluxH. Atrial septal defect I. BronchiolitisJ. Mitral stenosisFor each of these patients with breathlessness, select the most likely diagnosis1) A thirteen-year-old girl who has intermittent episodes of breathlessness, which tend to occur in crowded shops. She feels the need to take deep breaths and then breathes very quickly, complaining of pins & needles around her mouth and in her hands. Her chest is clear and her blood gases show a normal pO2 and low pCO2.B. HyperventilationNote:This description is one of anxiety. The blood gas picture is one of hyperventilation.2) A three-year-old boy who presents with worsening cough and breathlessness of 3 weeks’ duration. His mother was an intravenous drug abuser. He has always been prone to infections. When he was 2 years old he had chicken pox for 4 weeks. On examination, he has an emaciated appearance, his weight is below the 0.4th centile, he has a temperature of 37.6°C and he has generalized crepitations on auscultation of his chest. A blood count shows severe lymphopenia.E. Pneumocystis cariniiNote:This has resulted from congenitally acquired HIV. PCP has an insidious onset and often there are no chest signs in children. Lymphopenia is consistent. Treatment is with septrin or nebulised pentamidine as second line.3) A 4-month-old baby has not gained much weight since birth and only takes small milk feeds, as he appears to become breathless on feeding. He is tachypnoeic, sweaty and has a tachycardia. His liver is enlarged and he has a harsh grade 2-pansystolic murmur at the left lower sternal edgeF. Ventricular septal defectNote:Poor feeding is a symptom of heart failure in babies. A VSD has a pan systolic murmur and if large may have a lower grade as there is less resistance to flow. Hepatomegaly is an early sign of heart failure in infants.

Page 11: Mcqs Pediatrics

MCQs PEDIATRICS

Dr.Kamel Youssef Hassan, Pediatrician Consultant , Palestine - Gaza E-mail: [email protected] 11

7-Theme : CONGENITAL HEART DISEASEA. Ostium secundum atrial septal defectB. Ventricular septal defectC. Transposition of the great arteriesD. Total anomalous pulmonary venous drainageE. Atrioventricular septal defectF. Patent ductus arteriosusG. Pulmonary valve stenosisH. Coarctation of the aortaI. Tetralogy of FallotJ. Hypoplastic left heart syndrome

Which of the above is the most likely diagnosis in the following cases.1) A 13-year-old girl is referred for evaluation of her short stature. She is prepubertal. On auscultation she has an ejection systolic murmur in the second and third left intercostals spaces radiating to the back, but is a symptomatic.G. Pulmonary valve stenosisNote:The murmur describes pulmonary stenosis, which could also be a left peripheral pulmonary stenosis. She is short and has delayed puberty and coupled with the cardiac findings would suggest Noonan’s syndrome.

2) A 7-week-old infant presents with breathlessness on feeding and failure to thrive. On examination his femoral pulses are difficult to feel but present. Chest radiograph shows cardiomegaly and increased vascular markings.H. Coarctation of the aortaNote:Absent or weak femoral pulses suggest coarctation. Remember association with Turner’s syndrome.

3) An infant is seen for his 6-week check and found to have a loud ejection systolic murmur in the third left intercostal space and a single second heart sound on examination. There is no obvious cyanosis but a suggestion of mild desaturation. On the chest X ray there is a concavity on the left heart border and decreased pulmonary vascular markings.I. Tetralogy of FallotNote:Tetralogy of Fallot may present later than in the neonatal period. The ejection systolic murmur is from the infundibular stenosis. The desaturation results from the right to left shunt across the VSD.

Page 12: Mcqs Pediatrics

MCQs PEDIATRICS

Dr.Kamel Youssef Hassan, Pediatrician Consultant , Palestine - Gaza E-mail: [email protected] 12

8-Theme : CONGENITAL HEART DISEASEA. Atrioventricular septal defectB. Coarctation of the aortaC. Hypoplastic left heart syndromeD. Ostium secundum atrial septal defectE. Patent ductus arteriosusF. Pulmonary valve stenosisG. Tetralogy of FallotH. Total anomalous pulmonary venous drainageI. Transposition of the great arteriesJ. Ventricular septal defect

Which is the most likely diagnosis in the following cases?1) An infant is found profoundly cyanosed and lethargic in his cot on day 2. On auscultation there is a soft systolic murmur heard inconsistently at the left sternal edge and a single second sound. The chest X ray shows a narrow upper mediastinum, hypertrophied right ventricle and increased pulmonary vascular markings. The ECG shows a normal neonatal pattern.I. Transposition of the great arteriesNote:Cyanosis on the second day is suggestive of a duct-dependent lesion. The rest of the answer describes TGA.2) A 3-week-old premature infant born at 27 weeks gestation remains ventilated following surfactant deficient respiratory distress syndrome. On auscultation of his chest a systolic murmur is heard at the left sternal edge and pulses are very easy to feel. There is pulmonary plethora on chest X ray.E. Patent ductus arteriosusNote:PDA is a relatively common problem in premature babies. The left to right shunt results in excess blood flow through the lungs and frequently oxygen dependency and difficulty in weaning from the ventilator. A loud systolic murmur radiating to the back with easily palpable pulses are typical.3) A 7-year-old boy is examined for a chest infection. An incidental finding of a short systolic murmur with fixed splitting of the second heart sound is detected. His blood pressure is normal and all pulses are normal.D. Ostium secundum atrial septal defectNote:Ostium primum defects are unlikely to present incidentally but rather with heart failure or pulmonary hypertension. Fixed splitting is typical of ostium secundum defects

Page 13: Mcqs Pediatrics

MCQs PEDIATRICS

Dr.Kamel Youssef Hassan, Pediatrician Consultant , Palestine - Gaza E-mail: [email protected] 13

3- DERMATOLOGY1-Theme : SKIN AND HAIR ABNORMALITIESA. Alopecia areataB. Dermatitis artefactaC. TrichotillomaniaD. Aplasia cutisE. Sutural alopeciaF. Ectodermal dysplasiaG. Acrodermatitis enteropathica H. Menkes Kinky hairI. Trichorrhexis nodosa J. Telogen EffluviumSelect one option from the list above that is most suitable for the following patients1) A genetic syndrome resulting in abnormal dentition and ichthyosisF. Ectodermal dysplasiaNote:Ectodermal dysplasia is an X linked recessive condition with features of alopecia, hypodontia and a defect in sweating. These individuals have characteristic facies.

2) A 15 year old girl with a history of emotional difficulties presents with linear markings to her forearms and an inconsistent history.B. Dermatitis artefactaNote:Dermatitis artefacta, skin lesions are deliberately produced by patients who conceal this fact from their doctors. The condition is usually found in individuals with emotional difficulties arising from a disorder of personality.

3) A congenital diffuse cause of hair loss due to an abnormality of the hair shaft with a child with intractable seizures.H. Menkes Kinky hairNote:Menkes, kinky hair syndrome is a progressive cerebral deterioration associated with seizures and twisted fractured hair. It is X linked recessive and the underlying aetiology is an abnormality of Copper transport resulting in low Copper and low Caeruloplasmin.

Page 14: Mcqs Pediatrics

MCQs PEDIATRICS

Dr.Kamel Youssef Hassan, Pediatrician Consultant , Palestine - Gaza E-mail: [email protected] 14

2-Theme : GENETICS - CLINICAL ABNORMALITIES OF LIMBSA. Bloom's syndromeB. Cockayne's syndromeC. Down's syndromeD. Ehlers Danlos syndromeE. Hunter's syndromeF. Prader-Willi syndromeG. Rubinstein-Taybi syndromeH. Russell Silver syndromeI. Sotos' syndromeJ. Williams syndromeIn a child with a suspected genetic disorder and the following clinical abnormalities choose the single most likely diagnosis from the list of options.1) Simian creasesC. Down's syndromeNote:Simian creases are found in approximately 45% of babies with Down's syndrome. This is a single horizontal palmer crease. 85% of affected individuals have a distal positioning of the palmer axial tri-radius. Also a syndrome 'Sandal gap' is found. This is wide gap between the first and second toes. Plantar creases between the first and second toes are usually deep and there is characteristic dermal ridge pattern.2) Broad thumbs and toes.G. Rubinstein-Taybi syndromeNote:Children with Rubinstein-Taybi syndrome are short in stature, which has a post-natal onset. They tend to have downward slanting palpable fissures and a hypoplastic mandible. Examination of the limbs may reveal broad thumbs and toes, flat feet and deep plantar creases.3) Large hands and feet and a high arched palate.I. Sotos' syndromeNote:Sotos' syndrome is also called cerebral gigantism. Birth length is usually over the 90th centile and there is rapid linear growth there on. Individuals have large hands and feet and macrocephaly. They have downward slanting eyes and a high arched palate and most children with Sotos' syndrome have learning difficulties.Comments:Examination of the hand sand feet are important in the assessment of a dysmorphic infant.

Page 15: Mcqs Pediatrics

MCQs PEDIATRICS

Dr.Kamel Youssef Hassan, Pediatrician Consultant , Palestine - Gaza E-mail: [email protected] 15

3-Theme : CUTANEOUS MANIFESTATIONS OF DISEASEA. Koplik SpotsB. Erythema MultiformeC. Erythema nodosumD. Dermatitis HerpetiformisE. VitiligoF. Gottron's papulesG. Alopecia areataH. Mycosis FungoidesI. Dermatitis artefactaJ. Café au lait spotsFor each of the following diseases choose the most specific mucuous or cutaneous manifestation from the list of options.1) Addison's diseaseE. VitiligoNote:Vitiligo is a disorder resulting from insufficiency of melanocytes deposited in the skin. It is associated with auto-immune conditions.

2) Cutaneous T-cell lymphomaH. Mycosis FungoidesNote:Mycosis Fungoides is acute alias T-cell lymphoma often presenting as superficial patchy dermatitis which is then developed in to tumours.

3) Coeliac diseaseD. Dermatitis HerpetiformisNote:Dermatitis herpetiformis is a pruritic fascicular rash found on elbows, shoulder, buttocks and knees and is associated with Coeliac disease.

Page 16: Mcqs Pediatrics

MCQs PEDIATRICS

Dr.Kamel Youssef Hassan, Pediatrician Consultant , Palestine - Gaza E-mail: [email protected] 16

4-Theme : SKIN AND HAIR ABNORMALITIESA. Alopecia areataB. Dermatitis artefactaC. TrichotillomaniaD. Aplasia cutisE. Sutural alopeciaF. Ectodermal dysplasiaG. Acrodermatitis enteropathicaH. Menkes Kinky hairI. Trichorrhexis nodosaJ. Telogen EffluviumSelect one option from the list above that is most suitable for the following patients1) A congenital cause for localised hair loss.D. Aplasia cutisNote:Aplasia cutis is a localised defect with the scalp. This can be an isolated finding although has been associated with Trisomies, particularly Patau syndrome.

2) An acquired localised area of hair loss related to Tinea capitisA. Alopecia areata Note:Alopecia areata is a local area of hair loss in this case secondary to underlying ringworm.

3) An acquired course of hair loss resulting from an emotionally disturbed teenager.C. TrichotillomaniaNote:Trichotillomania is a term for hair loss as a result of pulling of his/her own hair out, there is often a history of emotional difficulties.

Page 17: Mcqs Pediatrics

MCQs PEDIATRICS

Dr.Kamel Youssef Hassan, Pediatrician Consultant , Palestine - Gaza E-mail: [email protected] 17

5- Theme : CUTANEOUS MANIFESTATIONS OF DISEASEA. Koplik SpotsB. Erythema MultiformeC. Erythema nodosumD. Dermatitis HerpetiformisE. VitiligoF. Gottron's papulesG. Alopecia areataH. Mycosis FungoidesI. Dermatitis artefactaJ. Café au lait spotsFor each of the following diseases choose the most specific mucuous or cutaneous manifestation from the list of options.1) MeaslesA. Koplik Spots Note:Koplik spots are of pathognomonic measles found on the buccal mucosa membrane and resembles coarse granules of salt.

2) DermatomyositisF. Gottron's papulesNote:Gottron's papules are pathognomonic for Dermatomyositis. They are inflammatory papules found over the dorsal interphalangeal joints and they become violaceous in colour and flattened before they atrophy to leave.

3) Stevens-Johnson syndromeB. Erythema MultiformeNote:Hypopigmentation – Erythema multiforme major is also known as Stevens-Johnson syndrome. It is a serious systemic disease which involve the two mucous membranes plus skin. Often follows respiratory symptoms and is associated with a conjunctivitis, uveitis and bullae. It may result in fluid loss, weakness as well as anaemia and neutropenia.

Page 18: Mcqs Pediatrics

MCQs PEDIATRICS

Dr.Kamel Youssef Hassan, Pediatrician Consultant , Palestine - Gaza E-mail: [email protected] 18

4- GENETICS1-Theme : DIAGNOSIS OF CHROMOSOMAL DISORDERSA. Di -George AnomalyB. Fragile XC. Karyotype 45XOD. Karyotype 46XXE. Karyotype 47XYYF. Klinefelter syndromeG. TriploidyH. Trisomy 9I. Trisomy 18J. Trisomy 21For each of the following descriptions of clinical abnormalities choose the single most likely genetic disorder from the list of options1) A 13 year old boy with learning difficulties, undescended testicles and gynaecomastia.F. Klinefelter syndromeNote:Klinefelter syndrome (XXY). Individuals with Klinefelter’s usually have learning difficulties. Phenotypically they are relatively tall and slim individuals with small penis and testicles, gynaecomastia and infertility.2) Gross foetal oedema, a cystic hygroma and ultrasound appearances of female foetus.C. Karyotype 45XONote:Turner’s syndrome (45XO) affects females. Features include short stature and gonadal agenesis. Ultrasound scan of the foetus in the utero may confirm lymphoedema. Other features include a webbed neck, a broad shield like chest, wide spaced nipples and cubitus valgus. Cardiac anomalies are common in Turner’s syndrome the most common being a bicuspid aortic valve and coaptation of the aorta. Renal anomalies such as horse shoe kidneys are often seen3) A cardiac abnormality in a child with immuno deficiency and mild – moderate learning difficultiesA. Di -George AnomalyNote:Di -George syndrome, which is a defect of development of the thymus, parathyroid as well as great vessels. They tend to have characteristic facies with short palpable fissures, micrognathia and a short philtrum. They have reduced cellular immunity and hypoparathyroidism leads to hypocalcaemia and seizures. Cardiac anomalies are also common for example aortic arch defects.

Page 19: Mcqs Pediatrics

MCQs PEDIATRICS

Dr.Kamel Youssef Hassan, Pediatrician Consultant , Palestine - Gaza E-mail: [email protected] 19

2-Theme : DIAGNOSIS OF CHROMOSOMAL DISORDERSA. Di -George AnomalyB. Fragile XC. Karyotype 45XOD. Karyotype 46XXE. Karyotype 47XYYF. Klinefelter syndromeG. TriploidyH. Trisomy 9I. Trisomy 18J. Trisomy 21For each of the following descriptions of clinical abnormalities choose the single most likely genetic disorder from the list of options.1) Abnormal ears and facies, rocker bottom feet and cardiac abnormalitiesI. Trisomy 18Note:Trisomy 18 or Edwards syndrome, which has a very poor prognosis with 50% of affected individuals dying within the first week of life. Growth is poor and cardiac anomalies are severe. Babies with Edwards syndrome tend to have clenched hands with overlapping fingers and their feet are usually deformed with a “rocker bottom appearance” and equinovarus.

2) Eye manifestations such as squints, refractive errors and Brushfield’s spots.J. Trisomy 21Note:The eye manifestations of squints, refractive errors and Brushfield's spors relates to Trisomy 21 or Down syndrome. Affected individuals have learning difficulties. Eye manifestations include strabismus, refractive errors, epicanthic folds, Brushfield’s spots which are flecks on the iris. They tend to have almond shaped eyes with the outer aspect of the palpable fissure being upwardly slanting.

3) Learning difficulties, prominent ears and testicular enlargement.B. Fragile XNote:Fragile X syndrome, which can affect both sexes. Males with Fragile X have macro-orchidism, (testicular enlargement), prominent ears, moderate learning difficulties and often speech delay.

Page 20: Mcqs Pediatrics

MCQs PEDIATRICS

Dr.Kamel Youssef Hassan, Pediatrician Consultant , Palestine - Gaza E-mail: [email protected] 20

3-Theme : GENETICS - CLINICAL FEATURES OF GENETIC DISORDER SYNDROMESA. Angelman's syndromeB. Cri-du-chat syndromeC. Cornelia de-Lange syndromeD. Edwards syndromeE. Kartagener's syndromeF. Marfan's syndromeG. Noonan's syndromeH. Patau's syndromeI. Turner's syndromeJ. Williams syndromeFor each of the following descriptions of clinical abnormalities choose the single most likely genetic disorder from the list of options.1) Synophrys, limb defects and marked retardation of growth.C. Cornelia de-Lange syndromeNote:Cornelia de-Lange – Small stature, moderate to severe learning difficulties. Limb abnormalities for example oligodactyly and phocomelia. Facial dysmorphic features included synophrys, bushy eyebrows and a long philtrum2) Learning difficulties, pectus excavatum and heart defects.G. Noonan's syndromeNote:Noonan’s syndrome is a condition which has a similar phenotype to Turner's syndrome. Individuals have learning difficulties, a webbed neck, pectus excavatum and cardiac anomalies include pulmonary valve stenosis and branched stenosis of the pulmonary artery.3) An elf like face, learning difficulties and hypocalcaemia.J. Williams syndromeNote:Williams syndrome – The disorder including learning difficulties an unusual characteristic, elf like face with prominent lips. Epicanthic folds and peri-orbital puffiness. Hypercalcaemia is an associated finding and cardiovascularanomalies mainly supra-valvular aortic stenosis has been found in children with Williams syndrome.

Page 21: Mcqs Pediatrics

MCQs PEDIATRICS

Dr.Kamel Youssef Hassan, Pediatrician Consultant , Palestine - Gaza E-mail: [email protected] 21

4-Theme : GENETICS - CLINICAL FEATURES OF GENETIC DISORDER SYNDROMESA. Angelman's syndromeB. Cri-du-chat syndromeC. Cornelia de-Lange syndromeD. Edwards syndromeE. Kartagener's syndromeF. Marfan's syndromeG. Noonan's syndromeH. Patau's syndromeI. Turner's syndromeJ. Williams syndromeFor each of the following descriptions of clinical abnormalities choose the single most likely genetic disorder from the list of options.1) Microcephaly, dysmorphic features and an abnormal cry.B. Cri-du-chat syndrome2) Micropthalmia, scalp defects, polydactyly.H. Patau's syndrome3) Arachnodactyly, high arched palate and lens dislocation.F. Marfan's syndromeComments:Cri-du-chat syndrome is due to a deletion of the short arm of chromosome 5. Affected individuals have a low birth weight, microcephaly and learning difficulties. They have a characteristic high pitch cry and facial dysmorphic features including low set ears and epicanthic folds. Congenital heart disease is common. Patau syndrome is Trisomy 13 affected individuals have a poor prognosis with 80% dying within the first month of life. There is an incomplete development of the forebrain and optic nerves and skull defects are common for example cutis aplasia, microphthalmia and colobomata seen as well as cleft palate and polydactyly. Cardiac abnormalities are present in approximately 80% of babies with Patau’s syndrome. Marfan's syndrome – phenotypically children with Marfan’s syndrome have tall stature, slim limbs, arachnodactyly and joint hyper-extensibility. Cardiac anomalies are associated for example dilation of the aorta, aorta regurgitation and mitral valve prolapse. Eye lens dislocation is often seen.

Page 22: Mcqs Pediatrics

MCQs PEDIATRICS

Dr.Kamel Youssef Hassan, Pediatrician Consultant , Palestine - Gaza E-mail: [email protected] 22

5- INFECTIOUS DISEASES

1) DIARRHOEAA. Coeliac diseaseB. Crohns diseaseC. Ulcerative colitisD. Cows milk protein intoleranceE. Toddlers diarrhoeaF. Cystic fibrosisG. Lactose intoleranceH. Irritable bowel syndromeI. Abdominal migraineJ. Haemolytic uraemic syndromeFor the following children presenting with diarrhoea which is the most likely diagnosis:1) An 11 month infant presents with chronic diarrhoea and failure to thrive since breast feeding was discontinued at 9 months. He has been treated by his GP with antibiotics on two occasions for chest infections. Initial investigations reveal low serum potassium, chloride and metabolic alkalosis.F. Cystic fibrosisNote:Failure to thrive, chest infections and diarrhoea (the latter results from failure of pancreatic exocrine function) are indicative of CF. The abnormal biochemistry is caused by salt loss and may be described as pseudo-bartters syndrome

2) A 15 year old boy presents with delayed puberty and short stature. He gives a history of intermittent abdominal pain and diarrhoea. He recently presented to his family doctor with a painful red swelling on his shin which has now resolved.B. Crohns diseaseNote:Crohn’s disease may present in this way, the skin lesions are erythema nodosum which are associated with inflammatory bowel disease.

3) A 8 month old infant has recurrent diarrhoea. Mother dates the start of symptoms from the an episode of gastroenteritis. The diarrhoea is explosive in natureG. Lactose intoleranceNote:Secondary lactose intolerance is a known sequelae of viral gastroenteritis. It is not prevented by regrading milk following the episode of viral infection.

Page 23: Mcqs Pediatrics

MCQs PEDIATRICS

Dr.Kamel Youssef Hassan, Pediatrician Consultant , Palestine - Gaza E-mail: [email protected] 23

2) CONTRA-INDICATIONS TO VACCINESA. BCG B. Diphtheria Tetanus Polio C. Hepatitis B D. Haemophilus Influenzae BE. InfluenzaF. Measles Mumps Rubella G. PolioH. RubellaI. TetanusJ. TuberculinWhich of the vaccines listed above would be contra-indicated in the following scenarios:1) Contra-indication if known to have a hypersensitive reaction to egg.E. InfluenzaNote:relates to hypersensitivity to egg, which contra-indicates the Influenza vaccine as there is residual egg protein present.2) Contra-indication if known to have an allergy to gelatin.F. Measles Mumps RubellaNote:refers to contra-indications to MMR, which include children with allergies to Gelatin, Neomycin or kanamycin as well as children with untreated malignant disease or altered immunity. Those receiving immuno-suppressive drugs or radiotherapy are also contraindicated. Children who have received another live vaccine by injection within 3 weeks should not be given MMR vaccine. MMR should also not be given within 3 months of receiving an Immunoglobulin injection.3) Contra-indication if known to have an allergy to kanamycin.F. Measles Mumps RubellaNote:refers to contra-indications to MMR, which include children with allergies to Gelatin, Neomycin or kanamycin as well as children with untreated malignant disease or altered immunity. Those receiving immuno-suppressive drugs or radiotherapy are also contraindicated. Children who have received another live vaccine by injection within 3 weeks should not be given MMR vaccine. MMR should also not be given within 3 months of receiving an Immunoglobulin injection.

Page 24: Mcqs Pediatrics

MCQs PEDIATRICS

Dr.Kamel Youssef Hassan, Pediatrician Consultant , Palestine - Gaza E-mail: [email protected] 24

3) CHILDHOOD CHEST INFECTIONSA. Chlamydia B. Cytomegalovirus C. Group B Streptococcus D. Haemophilus InfluenzaE. Mycoplasma pneumoniaF. ParainfluenzaG. Respiratory syncytial virusH. Rhinovirus I. Staphylococcus aureusJ. UreaplasmaSelect the most appropriate pathogen from the above list that would account for the following presentations:1) The commonest cause of pneumonia in the 2 month - 6month age group.G. Respiratory syncytial virus2) A 9 year old boy presents with a cough and fever. Chest x-ray demonstrates a reticular nodular pattern with high adenopathy and a small pleural effusion.E. Mycoplasma pneumonia

3) A 5 year old boy presents with painful red swellings of his shins. Examination of his throat reveals an exudative tonsillitis.C. Group B StreptococcusComments: The commonest cause of pneumonia in the 2month – 6 month age group is Respiratory syncytial virus, which results in bronchiolitis and occurs in epidemics. Mycoplasma pneumonia mainly affects older children (between 5 and 15 years) resulting in community acquired pneumonias. Tonsillitis can occur at any age although is frequently seen in the 4-7 age group. The majority of episodes are caused by viruses although beta haemolytic streptococcal infection is also responsible and has systemic reactions such as erythema nodosum, rheumatic fever and glomerular nephritis

Page 25: Mcqs Pediatrics

MCQs PEDIATRICS

Dr.Kamel Youssef Hassan, Pediatrician Consultant , Palestine - Gaza E-mail: [email protected] 25

4) CHILDHOOD DIARRHOEAA. Adenovirus B. Cytomegalovirus C. Epstein-Barr virus D. E. Coli E. Giardia LambliaF. Hepatitis A G. Norwalk virusH. RotavirusI. Salmonella speciesJ. Staphylococcus aureus

Select one option from the list above that is most suitable for the following patients1) Accepted as the most common cause of infectious diarrhoea in children in the developed society. H. RotavirusNote:Rotavirus is the most common virus responsible for diarrhoea worldwide. It is a double stranded RNA virus, especially prevalent in the winter months. It causes fever and watery diarrhoea.

2) Can follow ingestion of dust containing dried faecal material.F. Hepatitis ANote:Hepatitis A is an RNA virus typically transmitted by the faecal-oral route.

3) Attaches the small intestinal border and releases extoxin.D. E. ColiNote:E-coli and enteropathogen is one of the bacterial causes of diarrhoea. Many of its effects are a consequence of an exotoxin. It should be suspected if there is a history of travel or a history of ingestion of poorly prepared food. Bacterial diarrhoea usually results in bloody diarrhoea.

Page 26: Mcqs Pediatrics

MCQs PEDIATRICS

Dr.Kamel Youssef Hassan, Pediatrician Consultant , Palestine - Gaza E-mail: [email protected] 26

5) URINARY TRACT DISEASEA. Haemolytic uraemic syndromeB. Henoch Schonlein purpura C. Nephrotic syndromeD. Polycystic kidneysE. PyelonephritisF. Renal CalculiG. Renal tubular acidosisH. Renal vein thrombosisI. Systemic lupus erythematosusJ. Wilm's tumourSelect the most appropriate diagnosis from the above list of options that would best explain the following cases:1) A 2 year old boy is admitted with a history of bloody diarrhoea, abdominal pain and puffiness around his eyes. He is found to be slightly jaundiced. His urea is 12.6 and Creatinine 163 mmol / literA. Haemolytic uraemic syndromeNote:describes a child with haemolytic uraemic syndrome, which is of unknown aetiology. Various agents have been implicated including E.coli (0157) Salmonella, Shigella and viruses. It usually presents with a prodromal symptoms of an upper respiratory tract infection or gastroenteritis and later develops in to pallor, oliguria and hypertension.Laboratory investigations may reveal microangiopathic haemolytic anaemia,thrombocytopenia and urinalysis may reveal proteinuria.2) A 9 day old baby is admitted with a 2 day history of vomiting and diarrhoea. During the admission haematuria is noted and he has had 2 convulsions. A mass is palpable in the left hypochondrium. H. Renal vein thrombosisNote:describes a child with renal vein thrombosis. In neonates it is associated with dehydration, asphyxia, shock or sepsis. Manifestations include sudden onset of haematuria and identification of an enlarged kidney. If both kidneys are involved it may result in acute renal failure.

3) A 5 year old girl presents with puffiness around the eyes. Urinalysis confirms proteinuria with a trace of blood. C. Nephrotic syndromeNote:relates to a child with Nephrotic syndrome. This occurs when there is proteinuria resulting in hypoalbuminaemia and oedema with an unknown aetiology. Peri-orbital or dependant oedema is usually noted first. There may also be a history of abdominal pain, vomiting and diarrhoea. The child needs to be monitored carefully as hypovolaemia and circulatory collapse. Steroids are the treatment of choice with careful management of fluids.

Page 27: Mcqs Pediatrics

MCQs PEDIATRICS

Dr.Kamel Youssef Hassan, Pediatrician Consultant , Palestine - Gaza E-mail: [email protected] 27

6) NEONATAL JAUNDICE

A. Physiological jaundiceB. Biliary atresiaC. HypothyroidismD. Rhesus incompatibilityE. Congenital spherocytosisF. Congenital cytomegalo virus infectionG. GalactosaemiaH. Glucose-6-phosphate dehydrogenase deficiencyI. Cystic fibrosisJ. Fructose intolerance

For each of the following jaundiced babies, select the most likely cause:

1) A 13 day old infant who was noted to have an umbilical hernia and has very dry skin presents with jaundice. She is a floppy baby. Her bilirubin is checked and is found to be elevated and mainly unconjugated. The community midwife has been unable to gain access to the home for the last week.C. HypothyroidismNote:Umbilical hernia, dry skin, hypotonia and jaundice are features of congenital hypothyroidism.

2) A caucasian infant has required surgery at the regional neonatal unit for meconium ileus and has developed jaundice. The serum conjugated bilirubin is 65 micromol/L. The diagnosis is eventually confirmed from the neonatal screening tests.I. Cystic fibrosis

Note:Meconium ileus and jaundice may be presenting features of CF in the neonatal period. The screening test is immunoreactive trypsin.

3) A 12 day old male baby has a conjugated and unconjugated hyperbilirubinaemia. He is breast fed and has become more disinterested in feeds. He was investigated for sepsis and blood cultures demonstrated E-coli septicaemia, urine cultures were clear. His clotting is deranged.G. Galactosaemia

Note:Galactosaemia typically presents around 2 weeks of age with jaundice. E coli septicaemia is a feature. Treatment is by removing galactose from the diet. Cataracts are a later feature even if treatment instituted early.

Page 28: Mcqs Pediatrics

MCQs PEDIATRICS

Dr.Kamel Youssef Hassan, Pediatrician Consultant , Palestine - Gaza E-mail: [email protected] 28

7) VIRAL INFECTIONSA. MeaslesB. RubellaC. Chicken PoxD. Herpes SimplexE. MumpsF. Glandular feverG. PertussisH. Polio myelitisI. Hepatitis AFor each description below choose the single most likely viral infection from the list of options.1) May result in Giant cell pneumonia.A. MeaslesNote:Measles infection is uncommon due to world-wide immunisation. A child with measles usually has a fever, upper respiratory tract symptoms and a morbilliform rash. Serious complications include a giant cell pneumonia and encephalitis (SSPE).

2) May result in a Keratoconjuctivitis.D. Herpes SimplexNote:Herpes simplex infection may be transmitted verdantly to an infant from their mother's genital tract. It may cause isolated skin lesions, a Keratoconjunctivitis or a paronychia. More seriously it is also responsible for encephalitis.

3) Caused by an RNA virus with no known carrier state.I. Hepatitis ANote:Hepatitis A, which is caused by an RNA virus, is usually transmitted by the oral route. It has an incubation period of between 15-50 days and treatment is usually symptomatic only.

Page 29: Mcqs Pediatrics

MCQs PEDIATRICS

Dr.Kamel Youssef Hassan, Pediatrician Consultant , Palestine - Gaza E-mail: [email protected] 29

8) URINARY TRACT DISEASEA. Haemolytic uraemic syndromeB. Henoch Schonlein purpura C. Nephrotic syndromeD. Polycystic kidneysE. PyelonephritisF. Renal Calculi G. Renal tubular acidosisH. Renal vein thrombosisI. Systemic lupus erythematosusJ. Wilm's tumourSelect the most appropriate diagnosis from the above list of options that would best explain the following cases:1) An 11 year old with a previous history of chronic glomerulonephritis presents with bruising and epistaxis. A full blood count confirms a pancytopenia.I. Systemic lupus erythematosusNote:describes a child with glomerulonephritis and bone marrow failure suggestive of a connective tissue disease such as SLE.

2) A 14 year old boy with a history of recurrent urinary tract infections present with severe abdominal pain radiating to his back. Dysuria and haematuria.F. Renal CalculiNote:relates to Urolithiasis, Renal Calculi. Children present with abdominal pain, voiding abnormalities, dysuria, haematuria may be present. Avoidance of dehydration is important. Treatment may require lithotripsy.

3) A 1 year old girl with a 3 month history of vomiting is investigated for failure to thrive. She is found to be mildly acidotic.G. Renal tubular acidosisNote:Renal tubular acidosis is the answer to Item 3. It is a clinical state of systemic hyperchloraemic acidosis resulting from impaired urinary acidification. Three types exist. Type 1 distal RTA, type 2 proximal RTA and type 4 mineralocorticoid deficiency. Type 3 is thought to be a variant of type 1. Types 1, 2 and 4 each have several causes. Children with isolated forms of proximal or distal commonly present with growth failure; gastrointestinal symptoms are also common. Nephro-calcinosis and hypercalciuria may complicate distal RTA and goals of treatment are to correct the acidosis and to maintain normal Bicarbonate and Potassium levels.

Page 30: Mcqs Pediatrics

MCQs PEDIATRICS

Dr.Kamel Youssef Hassan, Pediatrician Consultant , Palestine - Gaza E-mail: [email protected] 30

9) MUCOCUTANEOUS INFECTIONS AND INFESTATIONSA. Enterobiasis vermicularisB. ChlamydiaC. Candidiasis D. Tinea pedisE. Pityriasis rosea F. ScabiesG. RingwormH. Lichen sclerosisI. Napkin dermatitisJ. Lichen planusFor each of the following children who present with an itch, select the most appropriate diagnosis from the list of options:1) A child complains of an itchy rash over his arms. His brother and sister have similar symptoms.F. Scabies

Note:Scabies is caused by the mite Sarcoptes scabiei hominis. Transmission is through close body contact, the adult mites lay their eggs in burrows in the skin and it results in an eczematous rash with parotitis skin.

2) A baby has an excoriated perineal rash involving the flexures. C. CandidiasisNote:Candidiasis is caused by Candida albicans (yeast). In babies it presents as a perineal rash which usually affects the flexures. Satellite lesions may also be seen.

3) A boy has an itchy rash over the trunk with a solitary large oval lesion on the back.E. Pityriasis rosea

Note:Pityriasis rosea is a benign condition of the skin resulting in oval pink / brown scaly lesions over the trunk, which are usually preceded by a herald patch (a solitary large lesion usually between 1 –10cm). No treatment is required.

Page 31: Mcqs Pediatrics

MCQs PEDIATRICS

Dr.Kamel Youssef Hassan, Pediatrician Consultant , Palestine - Gaza E-mail: [email protected] 31

10) CHILDHOOD RESPIRATORY DISEASESA. AsthmaB. BronchiolitisC. CroupD. Cystic fibrosisE. DiphtheriaF. EpiglottisG. PneumoniaH. InfluenzaI. Retropharyngeal abscessJ. Whooping coughFor each patient below, choose the SINGLE most probable diagnosis from the above list of options. Each option may be used once, more than once or not at all.1) A six month old baby presents with high fever, breathlessness, cough and feeding difficulties. Chest examination reveals dull percussion note over the right base posteriorly with bronchial breath sounds on auscultation.G. PneumoniaNote:Pneumonia may occur at any age. Patients present with fever, tachypnoea, feeding difficulties and cyanosis. Examination reveals bronchial breath sounds and crepitations. Chest X ray may show consolidation. Common organisms are pneumococcus, haemophilus, staphylococcus, mycoplasma, TB and viruses.

2) A 1 year old baby boy is wheezy, coughing, cyanosed and breathless with intercostal recession.B. BronchiolitisNote:Acute bronchiolitis is very common in infancy. In winter epidemics of respiratory syncytial virus infection are the commonest cause. Wheeze, cough, fever and respiratory distress are common. Chest X ray shows hyperinflation.

3) A 4 year old non immunized boy presents with bouts of coughing ending in vomiting. He has an absolute lymphocytosis.J. Whooping coughNote:Whooping cough is caused by Bordetella pertussis infection. Bouts of coughing ending in vomiting, especially at night and after feeding suggest the diagnosis. The characteristic whoop, forced inspiration through a closed glottis may or may

Page 32: Mcqs Pediatrics

MCQs PEDIATRICS

Dr.Kamel Youssef Hassan, Pediatrician Consultant , Palestine - Gaza E-mail: [email protected] 32

not be present. Absolute lymphocytosis is common. Complications include CNS haemorrhages, rectal prolapse and bronchiectasis.

4) A nine month old baby girl is upset and has stridor. Her voice is hoarse and has a barking cough. She has a low grade fever.C. CroupNote:Croup usually occurs in epidemics in autumn or spring. Causative viruses are Parainfluenza (types 1, 2, or 3), respiratory syncytial viruses and measles virus. Onset is over a few days; stridor is harsh and occurs only when child is upset. A barking cough, harsh voice and ability to swallow secretions are typical.5) A 2 year old boy is very unwell. His temperature is 39°C and he is unable to swallow his secretions.F. EpiglottisNote:Acute epiglottis is due to Haemophilus influenza type B infection. It is characterised by sudden onset, high fever, continuous stridor and drooling of secretions. Intravenous antibiotics, anaesthetic support are usually indicated.

Page 33: Mcqs Pediatrics

MCQs PEDIATRICS

Dr.Kamel Youssef Hassan, Pediatrician Consultant , Palestine - Gaza E-mail: [email protected] 33

11) TREATMENT OF INFECTIOUS DISEASEA. AcyclovirB. Acyclovir plus CefotaximeC. Ampicillin plus GentamicinD. BenzylpenicillinE. CeftriaxoneF. Ciprofloxacin G. FlucloxacillinH. NetilmicinI. Symptomatic treatmentJ. TeicoplaninFor each case of infectious disease described below choose the single best treatment option from the list.1) A 4 year old with a 3 day history of vomiting and diarrhoea.I. Symptomatic treatmentNote:A child with a history of gastroenteritis is a very common childhood illness. Conservative treatment only is required.

2) A 2 year old child with an immune deficiency develops Chicken Pox.A. AcyclovirNote:A child who is immuno-deficient and therefore must be provided with Zoster Immune Globulin if exposed to Chicken pox through contact. If the Chicken pox develops, treatment with Acyclovir is required.3) A 7 year old child with widespread impetigo.G. Flu-cloxacillinNote:Impetigo is a common staphylococcal skin manifestation, which is highly contagious. Most frequent sites being nostrils and the peri-oral area. Flucloxacillin is the treatment of choice.

Page 34: Mcqs Pediatrics

MCQs PEDIATRICS

Dr.Kamel Youssef Hassan, Pediatrician Consultant , Palestine - Gaza E-mail: [email protected] 34

12) EXANTHEMA. Staphylococcal scalded skin syndrome B. RubellaC. Measles D. Kawasaki’s disease E. Impetigo F. ScarlatinaG. Infectious mononucleosisH. Henoch – Schonlein purpuraI. Meningococcal infectionJ. Stills disease (systemic onset juvenile chronic arthritis)Match the following descriptions of rash with the illness for which they are the most typical exanthem1) A 5 year old boy has a bright, red, punctate, erythematous rash which blanches on pressure, beginning in the axillae with some perioral pallor and relative facial sparing. The skin feels like “sandpaper”. The rash fades and desquamates on the hands and feet. A thick white exudate develops on the tongue which peels leaving a “strawberry tongue” with prominent papillae.F. Scarlatina(scarlet fever)Note:This description is typical of Scarlatina i.e. beta haemolytic streptococcal infection. The rash may be confused with that of Kawasaki disease. However, desquamation occurs in late in Kawasaki’s disease.

2) A maculopapular rash develops in a child with sore throat and fever who has been treated with ampicillin.G. Infectious mononucleosisNote:This is a known effect of giving ampicillin during EBV infection.

3) A 3 year old child presents with high fever for 7 days, conjunctival injection, fissuring of the lips and strawberry tongue, erythema followed by desquamation of the hands and feet, and a macular rash over the trunk with cervical lymphadenopathy.D. Kawasaki’s diseaseNote:There are major and minor features of Kawasaki disease. High fever and desquamation are typical.

Page 35: Mcqs Pediatrics

MCQs PEDIATRICS

Dr.Kamel Youssef Hassan, Pediatrician Consultant , Palestine - Gaza E-mail: [email protected] 35

13) PRESENTATION OF INFECTIOUS DISEASE IN CHILDHOODA. Chicken PoxB. Herpes simplexC. Infectious mononucleosisD. MeaslesE. MumpsF. MycoplasmaG. PertussisH. RubellaI. TuberculosisFor each presentation of infectious disease choose the single most likely diagnosis from the list of options.1) May be complicated by cerebellar ataxia.A. Chicken PoxNote:Chicken pox is a common childhood illness caused by Varicella. Vesicles usually appear as crops over the trunk. Viral cerebellitis is a complication.2) May present with apnoeas during infancy.G. PertussisNote:Pertussis is caused by Bordetella pertussis. In infancy it can be dangerous often presenting with apnoeas. Signs of an upper respiratory tract infection are also common with a paroxysmal cough (whoop). Babies may also present with vomiting and cyanosis. A blood count will reveal a lymphocytosis. Complications include broncho-pneumonia and bronchiectasis as sequelae is also associated though uncommon.

3) Associated with a maculo-papular rash that typically starts on face and extends to rest of bodyH. RubellaNote:Rubella infection is uncommon now due to widespread MMR vaccination. Typically causes a mild illness which may go undetected in 25-50% of cases. Typical symptoms are transient macular papular rash that starts on the face and extends to body and a low grade fever. Complications include arthritis, encephalitis and thrombocytopenia.

Page 36: Mcqs Pediatrics

MCQs PEDIATRICS

Dr.Kamel Youssef Hassan, Pediatrician Consultant , Palestine - Gaza E-mail: [email protected] 36

14) RASHESA. Staphylococcal scalded skin syndromeB. RubellaC. MeaslesD. Kawasaki’s diseaseE. ImpetigoF. ScarlatinaG. Infectious mononucleosisH. Henoch – Schonlein purpuraI. Meningococcal infectionJ. Stills disease (systemic onset juvenile chronic arthritis)Match the following descriptions of rash with the illness for which they are the most typical exanthem.1) A salmon-coloured, reticulate macular rash develops mainly over the extensor surfaces of the limbs in a 5 year old boy with swinging temperature; hot, swollen, painful knees and left elbow and palpable spleen. The ESR is 95. The blood count, C-reactive protein and chest X-ray are normal.J. Stills disease (systemic onset juvenile chronic arthritis)Note:“Salmon-coloured” is the description used to describe the rash of Still’s disease. The distribution is not that of HSP which covers typically the buttocks and limbs and is a purpuric rash.

2) A 12 year old boy develops petechiae and papules, some of which become purpuric over his buttocks and legs, associated with painful swollen knees. There is microscopic haematuria on testing. The platelet count is normal.H. Henoch – Schonlein purpuraNote:This is a description of HSP and the well recognised complication of HSP nephritis. A proportion of these patients will develop progressive nephritis and end stage renal failure.

3) A 5 day old girl has a high temperature and is irritable. She has areas of desquamation over her finger tips and in the axillae. Her carer notices that her skin blisters easily following minimal contact.A. Staphylococcal scalded skin syndromeNote:SSSS results from infection with staphylococci with the exofoliative toxin A and B. These exotoxins cause disruption to the epidermal layer by interfering with intercellular junctions. Mortality is up to 3% in children. A positive Nikolsky sign (slippage of the superficial layer of the epithelium on gentle pressure) The desquamation occurs concomitantly with the illness unlike Kawaskis disease and Kawasaki disease does not occur in this age group. There may be a history of minimal skin trauma which provides a port of entry for the organism.

Page 37: Mcqs Pediatrics

MCQs PEDIATRICS

Dr.Kamel Youssef Hassan, Pediatrician Consultant , Palestine - Gaza E-mail: [email protected] 37

15) DIARRHOEAA. Coeliac diseaseB. Crohns diseaseC. Ulcerative colitisD. Cows milk protein intoleranceE. Toddlers diarrhoeaF. Cystic fibrosisG. Lactose intoleranceH. Irritable bowel syndromeI. Abdominal migraineJ. Haemolytic uraemic syndromeFor the following children presenting with diarrhoea which is the most likely diagnosis:1) A 6 year old boy with Downs’s syndrome has between 3 and 4 loose stools a day. He is pale but otherwise the examination is unremarkable. When plotting his growth on a chart specific for Downs syndrome children it is clear that both height and weight have fallen across 2 centiles.A. Coeliac diseaseNote:Coeliac disease is more occurs more frequently in Downs’s syndrome. To screen, anti-TTG antibodies taken with serum IgA must be evaluated.

2) A 3 year old has intermittent diarrhoea with stools of varying consistency and sometimes undigested food particles. His growth is satisfactory. His mother had tried to reduce his intake of diary produce but dietary manipulation has had no impact on the symptomsE. Toddlers diarrhoeaNote:Toddler diarrhoea (thought to be a normal variant) occurs mainly in preschool children and is evidenced by undigested food observed in the faeces in a well child. Poor growth would indicate a likely pathological cause.

3) A 4 month baby girl has severe atopic eczema and is regularly reviewed in the paediatric clinic. She had now developed diarrhoea and her weight has been static in recent weeks. A full blood count shows a slight eosinophilia.D. Cows milk protein intoleranceNote:CMPI is associated with eczema and eosinophilia. Cow’s milk should be excluded from the diet and reintroduced between 1-2 years if tolerated. An alternative milk source such as soy milk can be used until this is done.

Page 38: Mcqs Pediatrics

MCQs PEDIATRICS

Dr.Kamel Youssef Hassan, Pediatrician Consultant , Palestine - Gaza E-mail: [email protected] 38

16) CHILDHOOD INFECTIONSA. AdenovirusB. CytomegalovirusC. Epstein-Barr virusD. Escherichia coliE. Giardia lambliaF. Haemophilus influenzae type BG. Listeria monocytogenesH. Mycobacterium tuberculosisI. Mycoplasma pneumoniaeJ. Neisseria meningitidisK. Pseudomonas aeruginosaL. Pneumocystis cariniiM. Respiratory syncytial virusN. RotavirusO. Staphylococcus aureusP. Staphylococcus epIdermidisQ. Streptococcus agalactiaeR. Streptococcus pneumoniaeS. Streptococcus pyogenesT. Toxoplasma gondiiAll of the infectious diseases described below occur in children. For each one, select the most likely underlying causative agent from the list of options1) A 3 year old boy is referred to hospital with a two day history of lethargy, irritability and poor feeding. On examination, he is pyrexial, drowsy and has a purpuric rash on his trunk and extremities. CSF obtained from a lumbar puncture is cloudy and contains 540 white cells/mm3 (90% polymorphs) and 5 red blood cells/mm 3 .J. Neisseria meningitides

2) A 6 year old girl presents with a one week history of febrile illness with sore throat and headache. One day prior to hospital admission, the patient awoke with pain and swelling in the right ankle. On examination, she has a warm swollen right ankle and a systolic heart murmur, consistent with mitral regurgitation.S. Streptococcus pyogenes

3) A new born infant is found to be lethargic and has a distended abdomen immediately after birth. On examination, the infant is jaundiced and has hepatosplenomegaly. A cranial CT scan reveals periventricular calcification.

Page 39: Mcqs Pediatrics

MCQs PEDIATRICS

Dr.Kamel Youssef Hassan, Pediatrician Consultant , Palestine - Gaza E-mail: [email protected] 39

B. Cytomegalovirus

4) A 7 year old male child is referred to hospital by the general practitioner with acute renal failure. The child had bloody diarrhoea and a low grade fever a week ago; both resolved with rehydration.D. Escherichia coli

5) A 4 month old female infant is brought to the hospital with severe respiratory distress. Five days previously, she had a cough and rhinitis. On examination her temperature is 38.9 oC, pulse 180/min and the respiratory rate 80/min. She had subcostal retractions and nasal flaring. On auscultation, there are rhonchi and wheezes all over her chest.M. Respiratory syncytial virusComments:1. The diagnosis of Meningococcal meningitis is clear. 2. Rheumatic fever results from immune-mediated post Group A streptococcal infection, Streptococcus pyogenes being a common pathogen. 3. Congenital CMV infection is associated with petechiae, choroidoretinitis, hepatosplenomegaly, intracerebral calcification which may lead to CNS damage with long term sequaelae. 4. E. coli infection has resulted in Haemolytic Uraemic Syndrome with renal failure. 5. Respiratory Syncytial Virus is the commonest cause of lower respiratory tract infections in children worldwide, and is the leading cause of bronchiolitis and pneumonia in children.

Page 40: Mcqs Pediatrics

MCQs PEDIATRICS

Dr.Kamel Youssef Hassan, Pediatrician Consultant , Palestine - Gaza E-mail: [email protected] 40

17) VIRAL INFECTIONSA. MeaslesB. RubellaC. Chicken PoxD. Herpes SimplexE. MumpsF. Glandular feverG. PertussisH. Polio myelitis I. Hepatitis AFor each description below choose the single most likely viral infection from the list of options1) Caused by a gram negative pleomorphic bacillusG. Pertussis2) Is caused by a paramyxovirus.E. Mumps3) This infection is often followed by a transient immuno-deficiency.F. Glandular feverComments:Whooping cough is not uncommon in infancy. It typically presents with apnoeic episodes or cyanotic episodes during infancy. In the older child upper respiratory tract infections and a paroxysmal cough with a whoop is characteristic. Mumps is caused by a paramyxovirus. Glandular fever virus infects the B lymphocytes which results in an immuno-deficiency which is usually self limiting.

Page 41: Mcqs Pediatrics

MCQs PEDIATRICS

Dr.Kamel Youssef Hassan, Pediatrician Consultant , Palestine - Gaza E-mail: [email protected] 41

18) CHILDHOOD DIARRHOEAA. Adenovirus B. Cytomegalovirus C. Epstein-Barr virus D. E. Coli E. Giardia LambliaF. Hepatitis A G. Norwalk virusH. RotavirusI. Salmonella speciesJ. Staphylococcus aureusSelect one option from the list above that is most suitable for the following patients1) Causes diarrhoea by invading the brush border of the small intestine and causes vacuolation. I. Salmonella speciesNote:Salmonella may contaminate foods improperly foods cooked or stored and invades the brush border of the small intestine. Symptoms include nausea, vomiting and diarrhoea approximately between 1-8 hours after ingestion.

2) Spread is due to personal contact as well as by contaminated water supply.E. Giardia LambliaNote:Giardia Lamblia is a protozoa and may contaminate water or be transmitted by the faecal or oral route or person to person contact. Children are often asymptomatic however diarrhoea and cramps and weight loss may be features of the disease.

3) An organism which is not only associated with gastroenteritis but is also the most common cause of osteomyelitis and arthritis in children.J. Staphylococcus aureusNote:Staphylococcus aureus is a gram positive coccus and the most common cause of osteo-myelitis and arthritis in children. It may contaminate skin resulting in impetigo, cellulitis, folliculitis and furunculosis. It may cause pneumonias and may contaminate food resulting in enterotoxins being released in to the intestinal tract.

Page 42: Mcqs Pediatrics

MCQs PEDIATRICS

Dr.Kamel Youssef Hassan, Pediatrician Consultant , Palestine - Gaza E-mail: [email protected] 42

19) CONTRA-INDICATIONS TO VACCINESA. BCG B. Diphtheria Tetanus Pertussis C. Hepatitis BD. Haemophilus Influenzae BE. Influenza F. Measles Mumps Rubella G. Polio H. RubellaI. TetanusJ. TuberculinWhich of the vaccines listed above would be contra-indicated in the following scenarios:1) Contra-indicated if known to be allergic to Neomycin.F. Measles Mumps RubellaNote:refers to contra-indications to MMR, which include children with allergies to Gelatin, Neomycin or kanamycin as well as children with untreated malignant disease or altered immunity. Those receiving immuno-suppressive drugs or radiotherapy. Children who have received another live vaccine by injection within 3 weeks and children should not be given MMR within 3 months of an Immunoglobulin injection.

2) Contra-indicated in HIV positive patients.A. BCGNote:relates to vaccines in HIV infection. The department of health has advised that HIV positive subjects with or with out symptoms should not receive BCG, yellow fever or typhoid vaccinations.

3) Contra-indicated in subjects with progressive neurological conditions.B. Diphtheria Tetanus PertussisNote:relates to DTP (and the pertussis component) which should not be given to subjects with any progressive neurological disorder particularly epilepsy and immunisation should be delayed until the condition is stable.

Page 43: Mcqs Pediatrics

MCQs PEDIATRICS

Dr.Kamel Youssef Hassan, Pediatrician Consultant , Palestine - Gaza E-mail: [email protected] 43

20) CHILDHOOD RESPIRATORY DISEASESA. AsthmaB. BronchiolitisC. CroupD. Cystic fibrosisE. DiphtheriaF. EpiglottisG. PneumoniaH. InfluenzaI. Retropharyngeal abscessJ. Whooping coughFor each patient below, choose the SINGLE most probable diagnosis from the above list of options. Each option may be used once, more than once or not at all.1) A six month old baby presents with high fever, breathlessness, cough and feeding difficulties. Chest examination reveals dull percussion note over the right base posteriorly with bronchial breath sounds on auscultation.G. PneumoniaNote:Pneumonia may occur at any age. Patients present with fever, tachypnoea, feeding difficulties and cyanosis. Examination reveals bronchial breath sounds and crepitations. Chest X ray may show consolidation. Common organisms are pneumococcus, haemophilus, staphylococcus, mycoplasma, TB and viruses.

2) A 1 year old baby boy is wheezy, coughing, cyanosed and breathless with intercostal recession.B. BronchiolitisNote:Acute bronchiolitis is very common in infancy. In winter epidemics of respiratory syncytial virus infection are the commonest cause. Wheeze, cough, fever and respiratory distress are common. Chest X ray shows hyperinflation.

3) A 4 year old non immunized boy presents with bouts of coughing ending in vomiting. He has an absolute lymphocytosis. J. Whooping coughNote:Whooping cough is caused by Bordetella pertussis infection. bouts of coughing ending in vomiting, especially at night and after feeding suggest the diagnosis. The characteristic whoop, forced inspiration through a closed glottis may or may

Page 44: Mcqs Pediatrics

MCQs PEDIATRICS

Dr.Kamel Youssef Hassan, Pediatrician Consultant , Palestine - Gaza E-mail: [email protected] 44

not be present. Absolute lymphocytosis is common. Complications include CNS haemorrhages, rectal prolapse and bronchiectasis.

4) A nine month old baby girl is upset and has stridor. Her voice is hoarse and has a barking cough. She has a low grade fever.C. CroupNote:Croup usually occurs in epidemics in autumn or spring. Causative viruses are Parainfluenza (types 1, 2, or 3), respiratory syncytial viruses and measles virus. Onset is over a few days; stridor is harsh and occurs only when child is upset. A barking cough, harsh voice and ability to swallow secretions are typical.

5) A 2 year old boy is very unwell. His temperature is 39oC and he is unable to swallow his secretionsF. EpiglottisNote:Acute epiglottis is due to Haemophilus influenza type B infection. It is characterised by sudden onset, high fever, and continuous stridor and drooling of secretions. Intravenous antibiotics, anaesthetic support are usually indicated.

Page 45: Mcqs Pediatrics

MCQs PEDIATRICS

Dr.Kamel Youssef Hassan, Pediatrician Consultant , Palestine - Gaza E-mail: [email protected] 45

21) CONGENITAL AND NEONATAL DEFECTS DUE TO MATERNAL INFECTIONSA. AIDSB. Cytomegalovirus (CMV)C. Coxsackie group B D. Hepatitis BE. Herpes simplexF. ListeriosisG. RubellaH. SyphilisI. ToxoplasmosisJ. VaricellaFor each patient below, choose the SINGLE most probable diagnosis from the above list of options. Each option may be used once, more than once or not at all.1) A newborn baby presents with rudimentary digits, limb hypoplasia and convulsions.J. VaricellaNote:Chicken pox infection within the first 20 weeks of pregnancy may result in the congenital varicella syndrome. This is characterised by cerebral cortical and cerebellar hypoplasia, microcephaly, convulsions, limb hypoplasia and rudimentary digits. Prevention is by administering varicella vaccine even before pregnancy. Varicella immunoglobulin is administered to pregnant women who are exposed to infection. Infection during pregnancy is treated with acylovir.

2) A six week old baby is confirmed to have cataracts, cardiac abnormalities, thrombocytopenia and cerebral calcification.G. RubellaNote:This baby has congenital rubella. It occurs in children of non immunised women. Symptoms are absent in 50% of mothers. The foetus is most vulnerable in the first 16 weeks of pregnancy. Cataracts are associated with infections in weeks 8-9, deafness at 5-7 weeks and cardiac lesions from 5-10 weeks. Diagnosis is based on rising antibody titres in blood taken 10 days apart and the presence of IGM antibodies at 4-5 weeks from incubation period.

3) A pre term neonate has multi-organ disease with granulomas on his skin. His mother had a special liking for soft cheese during her pregnancy.F. Listeriosis

Page 46: Mcqs Pediatrics

MCQs PEDIATRICS

Dr.Kamel Youssef Hassan, Pediatrician Consultant , Palestine - Gaza E-mail: [email protected] 46

Note:Maternal listeriosis is usually a mild infection but transplacental infection and premature labour may occur in about 5% of cases. Avoidance of partially cooked meats, soft cheeses and unpasteurised milk should avoid this infection. Treatment is with ampicillin and gentamicin. Neonatal infection is usually multi-organ and granulomas may be found on the skin and the pharynx.

4) A two week old baby has microcephaly, seizures and chorioretinitis.I. ToxoplasmosisNote:Maternal and foetal toxoplasma infection may be avoided by advising pregnant women to wear gloves when gardening or handling cat litter and to thoroughly cook meat. Affected babies are treated with pyrimethamine, sulphadiazine and folic acid.

5) A new born baby is very unwell with jaundice, hepatosplenomegaly and microcephaly.B. Cytomegalovirus (CMV)Note:Maternal CMV infection is usually mild and asymptomatic. 5 in 1000 live births are affected, 5% will develop cytomegalic inclusion disease. The foetus is most at risk in early pregnancy. There is no effective prevention.

Page 47: Mcqs Pediatrics

MCQs PEDIATRICS

Dr.Kamel Youssef Hassan, Pediatrician Consultant , Palestine - Gaza E-mail: [email protected] 47

22) CHILDHOOD CHEST INFECTIONS

A. Chlamydia B. Cytomegalovirus C. Group B Streptococcus D. Haemophilus InfluenzaE. Mycoplasma pneumoniaF. ParainfluenzaG. Respiratory syncytial virusH. Rhinovirus I. Staphylococcus aureusJ. UreaplasmaSelect the most appropriate pathogen from the above list that would account for the following presentations:1) A 4 year old boy presents with a rapid history of high fever and drooling.D. Haemophilus Influenza2) A 4 year old girl presents with a 24 hour history of rhinitis, a barking cough and hoarseness.F. Parainfluenza3) The commonest cause of pneumonia in the neonate.C. Group B StreptococcusComments:Item 1 describes a child with acute epiglottitis. This is a rare infection mainly caused by Haemophilus influenza. Airway obstruction can develop rapidly due to oedema around the epiglottis. Item 2 describe a child with croup. Acute laryngotracheal bronchitis. It is almost exclusively viral in origin. Mainly Parainfluenza. Streptococcus pneumonia is a common pathogen in the lung and the commonest cause of pneumonia in the neonate.

Page 48: Mcqs Pediatrics

MCQs PEDIATRICS

Dr.Kamel Youssef Hassan, Pediatrician Consultant , Palestine - Gaza E-mail: [email protected] 48

23) MUCOCUTANEOUS INFECTIONS AND INFESTATIONSA. Enterobiasis vermicularisB. ChlamydiaC. Candidiasis D. Tinea pedisE. Pityriasis rosea F. ScabiesG. RingwormH. Lichen sclerosisI. Napkin dermatitisJ. Lichen planusFor each of the following children who present with an itch, select the most appropriate diagnosis from the list of options:1) A child has itching in the vulva region mostly at night.A. Enterobiasis vermicularisNote:Enterobiasis vermicularis is also known as threadworms. It is a common infestation in children and present with nocturnal anal pruritus and a perianal irritation.

2) A child presents with annular regions over the trunk.G. RingwormNote:Ringworm also known as tinea corporis presents with plaques of scaling eczema which are characteristically itchy.3) A diabetic child presents with a vulva rash.C. CandidiasisNote:Candidiasis is caused by Candida albicans (yeast). In babies it presents as a perineal rash which usually affects the flexures. Satellite lesions may also be seen.

Page 49: Mcqs Pediatrics

MCQs PEDIATRICS

Dr.Kamel Youssef Hassan, Pediatrician Consultant , Palestine - Gaza E-mail: [email protected] 49

24) PRESENTATION OF INFECTIOUS DISEASE IN CHILDHOODA. Chicken PoxB. Herpes simplexC. Infectious mononucleosisD. MeaslesE. MumpsF. MycoplasmaG. PertussisH. RubellaI. TuberculosisFor each presentation of infectious disease choose the single most likely diagnosis from the list of options.

1) Commonly causes an acute gingivostomatitis.B. Herpes simplexNote:Herpes simplex - the majority of children have benign manifestations of primary infection with Herpes simplex, for example a gingival stomatitis. The virus is readily spread by direct contact especially to damaged skin e.g. eczema.

2) Causing an acute parotitisE. MumpsNote:Mumps infection is now uncommon due to the vaccination. It is caused by a paramyxovirus and usually causes minimal symptoms. The most common manifestation being an acute parotitis although severe infection with mumps may result in meningoencephalitis plus deafness.

3) Causing an exudative tonsillitis. C. Infectious mononucleosisNote:Glandular fever is also called infectious mononucleosis. It is caused by Epstein-Barr virus and usually presents with an exudative pharyngitis or tonsillitis and cervical lymphadenopathy. It may cause a transient impairment of cellular and humeral immunity, which is usually self limiting.

Page 50: Mcqs Pediatrics

MCQs PEDIATRICS

Dr.Kamel Youssef Hassan, Pediatrician Consultant , Palestine - Gaza E-mail: [email protected] 50

25) CHILDHOOD RESPIRATORY INFECTIONSA. Allergic Bronchopulmonary aspergillosisB. AspergillomaC. Bordetella pertussisD. Chlamydia E. Coxsackie B F. Klebsiella pneumoniaG. Mycoplasma pneumoniaH. Pneumocystis carinii I. Pseudomonas aeruginosaJ. TuberculosisFrom the list above, select the most appropriate diagnosis for the following presentations:1) A 5 year old boy with cystic fibrosis undergoes a routine chest x-ray which reveals an apical round lesion on the left of his chest.B. AspergillomaNote:Aspergilloma is a fungus which may complicate conditions such as asthma and cystic fibrosis. Children present with a cough and wheeze and there may be a positive skin test to aspergillus with an eosinophilia and an elevated IgE. Chest x-ray may reveal an apical round mass.

2) A 5 year old girl with a high temperature presents with painful blisters on thepalms and soles of her feetE. Coxsackie BNote:describes a child with hand foot and mouth, which is typically caused by Coxsackie B. Characteristically individuals develop papules which then progress to vesicles. They are painful and pruritic and typically affect acral areas. It is often associated with a high temperature.

3) A 14 year old boy presents with fever, anorexia and loss of weight of 3 months duration. Scattered crepitations are heard over both lungs. Chest x-ray is abnormal with generalised mottling. J. TuberculosisNote:TB- Mycobacterium Tuberculosis is a primary infection which may occur in the lung, gut or skin. The local infection spreads to the surrounding lymph nodes, which constitutes the primary complex in the lungs. Progression of the primary complex may result in bronchopneumonia or bronchial obstruction secondary to enlargement of the lymph nodes. Pleural effusions may also occur as well as cavitations. Primary TB may spread to the blood stream resulting in TB. Examination may reveal hepatosplenomegaly and fundoscopy may reveal choroid tubercles.

Page 51: Mcqs Pediatrics

MCQs PEDIATRICS

Dr.Kamel Youssef Hassan, Pediatrician Consultant , Palestine - Gaza E-mail: [email protected] 51

26) TREATMENT OF INFECTIOUS DISEASEA. AcyclovirB. Acyclovir plus CefotaximeC. Ampicillin plus GentamicinD. BenzylpenicillinE. CeftriaxoneF. Ciprofloxacin G. FlucloxacillinH. NetilmicinI. Symptomatic treatmentJ. TeicoplaninFor each case of infectious disease described below choose the single best treatment option from the list

1) A 7 year old child develops an exudative tonsillitis and lethargy, monospot is positive.I. Symptomatic treatmentNote:Glandular fever where the treatment is symptomatic / conservative only.

2) A 1 year old child presents with neck stiffness and a purpuric rash.D. BenzylpenicillinNote:A child with meningococcaemia presented to the surgery and Benzylpenicillin needs to be given urgently prior to transfer to hospital.

3) A 4 year old child presents with encephalopathy. The EEG shows high amplitude, abnormal waveforms. The MRI scan is also abnormal. A diagnosis of encephalitis is made.B. Acyclovir plus CefotaximeNote:A child with encephalitis. The majority of cases are secondary to viruses although toxic and metabolic causes need to be considered. Of the viruses Herpes simplex is the most common agent causing encephalitis. When encephalitis is suspected both antibiotics and Acyclovir must be commenced immediately.

Page 52: Mcqs Pediatrics

MCQs PEDIATRICS

Dr.Kamel Youssef Hassan, Pediatrician Consultant , Palestine - Gaza E-mail: [email protected] 52

27) CUTANEOUS MANIFESTATIONS OF INFECTIOUS DISEASEA. Rheumatic feverB. TBC. Lyme's diseaseD. Chicken poxE. HistoplasmosisF. Cat scratch diseaseG. MeaslesH. ParvovirusI. Hepatitis BJ. Herpes simplexFor each cutaneous manifestation described below choose the single most likely associated infectious disease.1) Erythema infectiosum.H. ParvovirusNote:Erythema infectiosum also known as Fifths disease caused by human parvovirus, often results in a low grade temperature, slapped cheek appearance to the face and a reticular lacy rash to the arms.

2) Gianotti-Crosti.I. Hepatitis BNote:Gianotti-Crosti is a syndrome of non-pruritic erythematous papules on the face, buttocks and extremities. Characteristically related to Hepatitis B infection although other viruses for example EBV may be associated.

3) Koplik spots.G. MeaslesNote:Koplik spots are white coarse granules found on the buccal mucosa opposite the back molars and is pathognomonic for measles.

Page 53: Mcqs Pediatrics

MCQs PEDIATRICS

Dr.Kamel Youssef Hassan, Pediatrician Consultant , Palestine - Gaza E-mail: [email protected] 53

28) CONGENITAL AND NEONATAL DEFECTS DUE TO MATERNAL INFECTIONA. AIDSB. Coxsackie group BC. Cytomegalovirus (CMV)D. Hepatitis BE. RubellaF. Herpes simplexG. ListeriosisH. SyphilisI. VaricellaJ. ToxoplasmosisFor each patient below, choose the SINGLE most probable diagnosis from the above list of options. Each option may be used once, more than once or not at all.1) A newborn baby presents with rudimentary digits, limb hypoplasia and convulsions.I. Varicella

2) A six week old baby is confirmed to have cataracts, cardiac abnormalities, thrombocytopenia and cerebral calcification.E. Rubella

3) A pre term neonate has multi-organ disease with granulomas on his skin. His mother had a special liking for soft cheese during her pregnancy.G. Listeriosis

4) A two week old baby has microcephaly, seizures and chorioretinitis.J. Toxoplasmosis

5) A new born baby is very unwell with jaundice, hepatosplenomegaly and microcephaly.C. Cytomegalovirus (CMV)

Comments:Rudimentary digits and limb hypopkasia suggest congenital chicken pox infection. The presence of cataracts, cardiac abnormalities, thrombocytopaenia and cerebral calcification suggest a diagnosis of Congenital rubella. The presence of Choroido-retinitis strongly suggests a diagnosis of Toxoplasmosis. The baby born with granulomas and whose mother has a penchant for soft cheeses suggests Listeria Monocytogenes infection. Chorio-retinitissuggests The presence of Jaundice, HSM and microcephaly suggests CMV infection.

Page 54: Mcqs Pediatrics

MCQs PEDIATRICS

Dr.Kamel Youssef Hassan, Pediatrician Consultant , Palestine - Gaza E-mail: [email protected] 54

29) CHILDHOOD VIRAL INFECTIONSA. AdenovirusB. CoxsackieC. CytomegalovirusD. Epstein barr E. MeaslesF. Molluscum contagiosum G. MumpsH. RotavirusI. RubellaJ. VaricellaFor each patient with the group of symptoms listed below, choose the SINGLE most probable causative agent from the above list of options. Each option may be used once, more than once or not at all.1) A two year old infant boy is admitted to hospital with vomiting, none bloody watery diarrhoea and is dehydrated. It emerges other children from his play group have developed a similar illness.H. RotavirusNote:Rotavirus is the most common cause of severe viral gastroenteritis worldwide. Infection is via the faeco-oral route and often occurs in children aged between six months to six years. This RNA virus replicates in the intestinal mucosal cells damages transport mechanisms leading to salt and water depletion which results in diarrhorea and vomiting. Diagnosis is made from clinical features and culture of virus from stools and also by Polymerase chain reaction techniques. Treatment is mainly re-hydration and correction of any electrolyte imbalance.

2) A two year old boy is mildly unwell. His mother has noticed vesicles in his mouth, palms and soles of his feet.B. CoxsackieNote:Coxsackie A16 virus is the cause of hand, foot and mouth disease characterized by fever, sore throat and ulcerating vesicles in palms, orophaynx and on soles. Incubation period is 5-7 days and these heal without crusting. Treatment is symptomatic.

3) A Three-year baby girl presents with a macular confluent rash which appeared initially behind the ears and is spreading. Over the previous five days she has had a low grade fever, catarrh and conjunctivitis. Her mother is vague about her immunization history.

Page 55: Mcqs Pediatrics

MCQs PEDIATRICS

Dr.Kamel Youssef Hassan, Pediatrician Consultant , Palestine - Gaza E-mail: [email protected] 55

E. MeaslesNote:Measles is caused by and RNA paramyxovirus and occurs worldwide. Outbreaks are common in areas with high numbers of non immunized children. Infection is transmitted via respiratory droplets and incubation period is 10-21 days. The prodromal stage fever conjunctivitis, runny nose and coughing lasts for five days. Koplik’s spots are bright red lesions with a central white dot which appear on the buccal mucosa. These are virtually diagnostic. The typical macular confluent rash appears on the face from day 3-5 and spreads to the rest of the body. Diagnosis is made from clinical features, viral culture from lesions and a grater than 4-fold rise in antibody titres. Otitis media, pneumonia, meningitis and very rarely several years after primary infection subacute sclerosing panencephalitis (SSPE).

4) A Ten year old girl develops an itchy rash on her body which is spreading to her body. She traveled to see her grandmother who was unwell with a painful rash three weeks ago.J. VaricellaNote:Varicella (Chicken pox) is transmitted by respiratory droplets and contact with somebody with shingles. Incubation period is 14-21 days and following a brief period of malaise, an itchy papulovesicular rash appears appears on trunk and spreads to the head and the extremities. The rash evolves from papules to vesicles, pustules and finally crusts. Antiviral therapy is reserved for systemic disease in the immunocompromised.

5) The four month old baby daughter of an HIV positive mother is admitted to hospital with seizures. She has neonatal jaundice and microcephaly.C. CytomegalovirusNote:Cytomegalovirus inclusion disease is the result of infection of the foetus. Many organs may be affected and congenital abnormalities result. Microcephaly, seizures, neonatal jaundice, hepatosplenomegaly, deafness and mental retardation are some of the features that may occur.

Page 56: Mcqs Pediatrics

MCQs PEDIATRICS

Dr.Kamel Youssef Hassan, Pediatrician Consultant , Palestine - Gaza E-mail: [email protected] 56

30) CUTANEOUS MANIFESTATIONS OF INFECTIOUS DISEASEA. Rheumatic feverB. TBC. Lyme's diseaseD. Chicken poxE. HistoplasmosisF. Cat scratch diseaseG. MeaslesH. ParvovirusI. Hepatitis BJ. Herpes simplexFor each cutaneous manifestation described below choose the single most likely associated infectious disease.1) Erythema nodosum in presence of abnormal chest x-ray.B. TBNote:Erythema nodosum may be associated with streptococcal reactions, rheumatic fever and Tuberculosis. In the presence of an abnormal chest X-ray TB is the most likely answer.

2) Erythema marginatumA. Rheumatic feverNote:Erythema marginatum is one of the five major criteria to make a diagnosis of rheumatic fever. It is a pink rash with pale centres and a serpiginous margin, found on the trunk and proximal limbs.

3) Erythema chronicum migrans.C. Lyme's diseaseNote:Erythema chronicum migrans found in Lyme's disease. A febrile illness caused by Borrelia burgdorferi transmitted by bites of animal tic. The rash is characterised by red margins and central clearing.

Page 57: Mcqs Pediatrics

MCQs PEDIATRICS

Dr.Kamel Youssef Hassan, Pediatrician Consultant , Palestine - Gaza E-mail: [email protected] 57

6- PEDIATRICS1) Theme: ABDOMINAL PAINA. PeritonitisB. AppendicitisC. ConstipationD. Urinary tract infectionE. Mesenteric lymphadenitisF. MigraineG. IntussusceptionH. Henoch-Schonlein vasculitisI. PsychosomaticJ. Shigella dysenteryFor the following patients with abdominal pain, choose the most likely diagnosis.

1) A previously well 10 month old baby presenting with drawing up of her legs with crying and some bloody runny stools.G. IntussusceptionNote:This is the right age group for intussception. It is often preceded by an URTI and it is thought that mesenteric lymphadenopathy may precipitate intussception. A red currant jelly-like stool is a late sign. In the older child, the cause of intussception may be HSP or lymphoma.2) A 4-year-old child, with poor eating habits, iron deficiency anaemia and intermittent colicky abdominal pain.C. ConstipationNote:Constipation is common in children and should be managed aggressively to establish regular bowel habit quickly.3) A 2-year-old girl with nephrotic syndrome who has developed a fever and abdominal pain.A. PeritonitisNote:The causes of abdominal pain in nephrotic syndrome are peritonitis (usually pneumococcal), renal vein thrombosis and hypovolaemia.

Page 58: Mcqs Pediatrics

MCQs PEDIATRICS

Dr.Kamel Youssef Hassan, Pediatrician Consultant , Palestine - Gaza E-mail: [email protected] 58

2) Theme: Skin lesionsA. Café au lait spotsB. Capillary haemangiomaC. Port-wine stain D. Shagreen patchE. Adenoma sebaceumF. Cutis MarmorataG. Herald's patchH. Peri-oral pigmentationI. Hypo pigmentationJ. TuberSelect one option from the list above that is most suitable for the following patients

1) An 11 year old boy with learning difficulties has an acne type rash over the noseE. Adenoma sebaceumNote:relates to the condition known as Tuberous sclerosis. It is a condition where by hamartomatous lesions become evident in early childhood. Seizures and learning difficulties are common and skin manifestations include fibrous angiomatous lesions in the naso labial folds known as adenoma sebaceum. White macules are seen over the trunk known as Ash leaf macules and Shagreen patches resemble goose like flesh. Subungual fibromata are also common.

2) A 4 year old boy with a history of focal seizures and a birth markC. Port-wine stainNote:relates to Sturge-Weber syndrome. The association and localization of aberrant vasculature in the facial skin, eyes and meninges. Most commonly in the trigeminal nerve distribution.

3) An infant with Down's syndrome has mottled appearance to the skin.F. Cutis MarmorataNote:relates to Cutis Marmorata, a mottled, marbled type appearance to the skin. Common in Down's syndrome as well as other Trisomies, hypothyroidism and Cornelia-de-Lange syndrome.

Page 59: Mcqs Pediatrics

MCQs PEDIATRICS

Dr.Kamel Youssef Hassan, Pediatrician Consultant , Palestine - Gaza E-mail: [email protected] 59

3) Theme: PoisoningA. Ant psychoticsB. BenzodiazepinesC. Ethylene glycolD. MethanolE. OpiatesF. OrganophosphatesG. SalicylatesH. SympathomimeticI. Tricyclic antidepressantsJ. Volatile solvents

Select the most appropriate agent from the above list that would elicit the following toxic effects:1) Coma, pinpoint pupils, hyperventilationE. Opiates

2) Hyper salivation, broncho-rrhoea, broncho- spasms, perspiration, neuromuscular paralysisF. Organophosphates

3) Tachypnoea, metabolic acidosis, Haematemesis, Reyes Syndrome.G. Salicylates

Comments:A pinpoint pupil suggests � opiates (Heroin – Morphine – codeine). The hyper salivation, tachypnoea and sweating suggest �Organophosphate poisoning. This drug inhibits ant-cholinesterase activity promoting Ach effects. Treatment consists of anticholinergics � (Atropine dose i.v 0.05 mg/kg repeated every 5-10 min as needed adverse effect tachycardia, dry mouth, blurred vision, and urinary retention). Finally, the use of salicylates in children is well-recognized cause of Reyes syndrome, a condition associated with fulminant hepatic failure. Suspected poisoning in children result in thousands of attendances at A&E department each year. Various drugs cause specific signs when taken in overdose. Careful examination of the child should follow A, B, C protocol.

Page 60: Mcqs Pediatrics

MCQs PEDIATRICS

Dr.Kamel Youssef Hassan, Pediatrician Consultant , Palestine - Gaza E-mail: [email protected] 60

4) Theme: ChemotherapySelect the most appropriate chemotherapeutic agent from the above list that corresponds with the following actions and side effects:

1) A drug disrupting microtubule leading to loss of deep tendon reflexes.J. Vincristine

2) A drug inhibiting initiation of DNA synthesis resulting in conjunctivitis and cerebellar toxicity.G. Cytosine arabinoside

3) A drug resulting in dose related lung damage.D. Bleomycin

Comments:All cytotoxic agents have adverse effects. General side-effects include nausea, vomiting and bone marrow suppression, alopecia and stomatitis. More specific side-effects are listed in the questions above. Alkylating agents such as Cyclophosphamide may cause a haemorrhagic cystitis. Asparaginase results in a dose related Pancreatitis.The anthracyclines e.g. Doxorubicin and Daunorubicin are cardiotoxic, which is often very difficult to detect. Monitoring with echocardiograms is advised. Vincristine is an alkaloid agent and results in sensory motor neuropathy with long-term use. It also has an affect on the autonomic system resulting in severe constipation and paralytic ileus. It may also result in sensory changes with parathesis progressing to loss of tendon reflexes. Bleomycin is the main drug resulting in lung damage and occurs in up to 10% of patients. The damage is dose related.

Page 61: Mcqs Pediatrics

MCQs PEDIATRICS

Dr.Kamel Youssef Hassan, Pediatrician Consultant , Palestine - Gaza E-mail: [email protected] 61

5) Theme: Childhood malignanciesA. Burkitt's nasopharyngeal carcinoma B. Ewing sarcomaC. HepatoblastomaD. Hodgkin's lymphomaE. MelanomaF. Nesidio-blastosisG. NeuroblastomaH. Teratoma I. Wilm's tumour J. Xeroderma pigmentosumSelect the most appropriate oncological diagnosis from the above list of options to explain the following scenarios1) A 13-year-old boy presents with abdominal pain. An abdominal mass is found. This boy has a past medical history of biliary atresia.C. Hepatoblastoma

2) A malignant transformation of a skin resulting from an autosomal recessive condition.J. Xeroderma pigmentosum

3) A baby on the neonatal unit is found to be hypoglycaemic.F. Nesidio-blastosis

Comments:Hepatoblastoma is a malignant liver tumour. It may present with anorexia, abdominal mass and jaundice. It is associated with cirrhosis as well as biliary cirrhosis secondary to biliary atresia. Xeroderma pigmentosa is an autosomalrecessive condition of defective DNA repair. Skin damage arise and malignant transformation results at multiple sites. Nesidio-blastosis is also known as beta cell endocrine benign hyperplasia and affects the pancreas. Diagnosis is made by hypoglycaemia with high serum Insulin and treatment is by Pancreatectomy.

Page 62: Mcqs Pediatrics

MCQs PEDIATRICS

Dr.Kamel Youssef Hassan, Pediatrician Consultant , Palestine - Gaza E-mail: [email protected] 62

6) Theme: SYNDROMESA. Down's syndromeB. Turner's syndromeC. Noonan's syndromeD. Klinefelter's syndromeE. Treacher-Collins syndromeF. Marfan's syndromeG. Sotos syndromeH. Klippel-feil syndromeI. Laurence Moon Biedl syndromeJ. MCune Albright syndromeWhich is the most applicable diagnosis for the following children.1) A 3-year-old boy with delayed motor development is reviewed regularly in the child development clinic. He attends for routine annual blood tests to check his thyroid function. A. Down's syndrome

Note:Children with Down’s syndrome are screened yearly for the development of hypothyroidism. They should also be regularly assessed for the development ofmyopia and hearing loss. These children have a predisposition to acute lymphoblastic leukaemia (ALL), Alzheimer’s-like dementia and Atlanto-axial subluxation.

2) A 16-year-old girl is being investigated for delayed menarche and short stature. In her notes it is recorded that she had swollen feet at birth but she was lost to follow-up. B. Turner's syndromeNote:A karyotype should be performed in all girls presenting in this way. Pedal lymphoedema at birth is a known presentation. Other associated features include coarctation of the aorta, infertility, hypothyroidism and horse shoe kidney.3) A girl of 6 years has been found to have hypertension and coarctation of the aorta. She has been referred to the growth clinic because of short stature. B. Turner's syndrome

Note:A karyotype should be performed in all girls presenting in this way. Pedal lymphoedema at birth is a known presentation. Other associated features include coarctation of the aorta, infertility, hypothyroidism and horse shoe kidney.

Page 63: Mcqs Pediatrics

MCQs PEDIATRICS

Dr.Kamel Youssef Hassan, Pediatrician Consultant , Palestine - Gaza E-mail: [email protected] 63

7) Theme: PRECOCIOUS PUBERTYA. CraniopharyngiomaB. Exaggerated adrenarcheC. McCune Albright syndromeD. Hypothalamic hamartomaE. Thelarche variantF. Neurofibromatosis type 1G. Granulosa-theca cell tumourH. HypothyroidismI. Premature thelarche J. Congenital adrenal hyperplasiaWhich is the most likely diagnosis in the following cases:

1) A 5-year-old girl presents with rapid breast development, breast stage 4, over a few months and an abdominal mass. There was no pubic hair or cliteromegaly. G. Granulosa-theca cell tumourNote:The rapid onset of puberty is indicative of a tumour. In this case the alpha-fetoprotein tumour marker will be raised�. Symptoms result from the very high oestradiol levels.

2) An 8 year old boy presents with testicular volumes of 8 mls bilaterally, pubic hair stage 3 and genital stage 3. He has long-standing constipation, deteriorating school performance and increased weight.H. HypothyroidismNote:Long standing hypothyroidism can cause precocious puberty through high levels of TRH driving gonadotrophin secretion and cross reactivity of the FSH receptor with TRH.

3) A 2year old girl presents with isolated bilateral breast development. Examination is otherwise normal as is her height velocity. Subsequent investigation demonstrates undetectable oestradiol levels and a prepubertal response to an LHRH test.I. Premature thelarcheNote:There may be a slight FSH predominance on the basal gonadotrophin levels. Premature thelarche is self-limiting and thought to be the result of the body being exquisitely sensitive to low levels of oestradiol.

Page 64: Mcqs Pediatrics

MCQs PEDIATRICS

Dr.Kamel Youssef Hassan, Pediatrician Consultant , Palestine - Gaza E-mail: [email protected] 64

8) Theme: Drugs During PregnancyA. Amino glycosidesB. AspirinC. Beta BlockersD. LithiumE. PhenytoinF. PethidineG. Sodium ValproateH. SulphonamidesI. Thiazide diuretics J. WarfarinSelect the drug from the above list of options that is most likely to be associated with the following effects if taken during pregnancy:1) This drug when taken in late pregnancy may result in the baby developing jaundiceH. Sulphonamides

2) This drug when taken in pregnancy may result in premature closure of the ductus arteriosis.B. Aspirin

3) Prolonged use of this drug may result in a permanent hearing impairment.A. Aminoglycosides

Comments:Sulphonamides taken in late gestation may compete with Bilirubin for protein binding sites resulting in a neonatal hyperbilirubinaemia.Aspirin has been associated with premature closure of the ductus arteriosis as well as haemorrhagic disorders.Aminoglycosides (such as Gentamicin) infused over a prolonged period during pregnancy will result in ototoxicity in approximately 2-3 % of cases.

Page 65: Mcqs Pediatrics

MCQs PEDIATRICS

Dr.Kamel Youssef Hassan, Pediatrician Consultant , Palestine - Gaza E-mail: [email protected] 65

9) Theme: OBESITYA. Type I diabetesB. PseudohypoparathyroidismC. Persistent hyperinsulinaemic hypoglycaemia of infancyD. Asthma treated with FluticasoneE. Simple obesityF. Prader Willi SyndromeG. Type 2 diabetesH. Hypothalamic tumourI. Cerebral palsyJ. HypothyroidismFor each of the following patients, choose the most likely cause for their obesity.1) A 13-year-old boy presents with polyuria and polydipsia. There have been no previous concerns regarding his health and development. He has mild asthma treated with prophylaxis in modest doses over the winter months only. On examination he has signs of acanthosis nigricans.G. Type 2 diabetes

2) A 3-month-old baby presents with a generalized convulsion. He has fed voraciously since birth, consuming in excess of 250 mls/milk/kg/day. Initial investigations demonstrate a serum calcium concentration of 2.4 mmol/L (normal) and a serum glucose concentration of 1.4 mmol/L.C. Persistent hyperinsulinaemic hypoglycaemia of infancy

3) A 9-year-old boy, previously well, presents with a 6-month history of increasing behavioural disturbances, increasingly severe headache and a convulsion.H. Hypothalamic tumourComments:1. Type 1 DM is unlikely to present with obesity but rather weight loss. Acanthosis nigricans is indicative of insulin resistance and is found typically in the axillae and around the nape of the neck. These findings suggest type 2 DMi.e. insulin resistance. However polydipsia and polyuria suggest insulinopenia and a failing pancreas which may occur in the later stages of type 2 DM. 2. PHHI is a defect of the K+ ATP channels in the beta cell of the pancreas. There is autonomous secretion of insulin that is independent of the serum glucose concentration. These babies will have detectable insulin at the time of hypoglycaemia, which would not occur normally. Treatment is with diazoxide and chlorothiazide, some require pancreatectomy. 3. Behavioural disturbance, headache and seizures suggest intracranial pathology. Coupled with obesitywould suggest a hypothalamic tumour as this area of the brain is involved in appetite regulation.

Page 66: Mcqs Pediatrics

MCQs PEDIATRICS

Dr.Kamel Youssef Hassan, Pediatrician Consultant , Palestine - Gaza E-mail: [email protected] 66

10) Theme: WEAKNESS IN THE LOWER LIMBSA. Guillain-Barre syndromeB. Transverse MyelitisC. Juvenile Multiple sclerosis D. Chronic fatigue syndromeE. Spinal tumourF. DermatomyositisG. Congenital myopathyH. Duchenne muscular dystrophy I. Fascioscapulohumeral dystrophyJ. Spinal abscessFor these children with lower limb weakness select the most likely diagnosis.1) A 15-year-old boy started walking at 18 months of age. He has mild ptosis, absence of facial expression and neck weakness. His mother has similar symptomsI. Fascio-scapulo-humeral dystrophy

Note:This is a description of fascio-scapular-humeral dystrophy, which typically presents in the teenage years. The family history does not suggest an X-linked disorder rather autosomal dominant.2) A 6-year-old child presents with acute onset symmetrical flaccid paralysis initially affecting the legs then involving the trunk, and now some mild weakness in the arms. 2 weeks prior to this he had been unwell with URTI and fever. On examination there is minimal movement with gravity eliminated in all muscle groups in the lower limbs, deep tendon reflexes are absent, plantar responses are up-going. Distal sensory losses are demonstrable. The CSF shows raised protein but no pleocytosis.A. Guillain-Barre syndrome

Note:The description is of Guillain-Barre syndrome, which may be preceded by an URTI. There are lower motor neurone signs but also with absent reflexes due to progressive distal demyelination. Sometimes there is cranial nerve involvement (Miller Fisher syndrome)3) A 9-year-old boy has a gradual onset of proximal weakness in the pelvic girdle and to a lesser extent in the shoulder girdle also. There is a faint violaceous rash over the eyelids and some telangiecasia over the finger knuckles. He has a low-grade fever and the thigh and buttock muscles are notably tender on examination.F. Dermatomyositis

Note:Proximal myopathy, heliotrope rash over eyelids and the description of Gotrens papules over the knuckles indicate dermatomyositis.

Page 67: Mcqs Pediatrics

MCQs PEDIATRICS

Dr.Kamel Youssef Hassan, Pediatrician Consultant , Palestine - Gaza E-mail: [email protected] 67

11) Theme: DELAYED MILESTONESA. 4 weeks B. 2 months C. 7 months D. 10 months E. 12 monthsF. 18 monthsG. 2 yearsH. 3 yearsI. 5 yearsJ. 6 yearsAt what age would you refer a child for investigation if they were unable to do the following:-1) Sit unsupported D. 10 months

2) Walk independently F. 18 months3) Smile B. 2 months

Comments:

It is important to be familiar with the developmental milestones, particularly up to the age of 3 years. Milestones are grouped under 4 headings:

Gross motor Fine motor Hearing and language Social

You should know the age range for each milestone. Developmental milestones are in Illingworth: 'The Normal Child'.

Page 68: Mcqs Pediatrics

MCQs PEDIATRICS

Dr.Kamel Youssef Hassan, Pediatrician Consultant , Palestine - Gaza E-mail: [email protected] 68

12) Theme: Renal diseaseA. Acute nephritisB. Nephrotic syndromeC. Diabetes insipidusD. Urinary tract infectionE. Chronic renal failureF. Diabetes mellitusG. Salt poisoningH. Haemolytic uraemic syndromeI. Barrter syndromeJ. CystinosisWhich diagnosis is most appropriate for the following patients?1) A 2-year-old girl has become puffy in her face and has developed swollen feet. Her urine dipstick shows proteinuria. B. Nephrotic syndrome

Note:Hypoalbuminaemia (serum albumin falls below 25g/L); oedema, proteinuria (exceed 2g/24 hr. or more than 40 mg/m2/hr.)(In urine dipstick +3 – +4 for proteinuria) and hypercholestrolaemia (LDL �, triglycerides�) are features of nephrotic syndrome. So are peritonitis (pneumococcal peritonitis due to low immunoglobulins), renal vein thrombosis and hypotension (due to hypovolaemia & hypercoagulable state with low antthrombin III) .

2) A 15 years old boy has felt tired, lost his appetite and has not grown in the last year. His serum urea is 20mmol/l and serum creatinine is 575µmol/l. His Hb is 9.6g/dl. E. Chronic renal failure

Note:Chronic renal failure may have an insidious onset and may present with slowed poor growth and delayed puberty.

3) A 2 week old boy presents to casualty with weight loss and inconsolable crying. His serum sodium is 150mmol/l and his urine osmolality is 100 mosm/kg (normal > 600).C. Diabetes insipidus

Note:Hypernatraemia with concomitant low urine osmolality suggest DI. Remember it is important to establish whether nephrogenic in origin (treatment indomethacinor chlorthiazide – thiazides – carbamzepine - chloramphenicol) or cranial(treatment DDAVP = exogenous ADH = Desmopressin = ADH analogue).

Page 69: Mcqs Pediatrics

MCQs PEDIATRICS

Dr.Kamel Youssef Hassan, Pediatrician Consultant , Palestine - Gaza E-mail: [email protected] 69

13) Theme : Neonatal JaundiceA. ABO incompatibilityB. Breast milk jaundiceC. CephalhaematomaD. Congenital biliary atresiaE. Congenital cytomegalovirus infectionF. Congenital haemolytic anaemiaG. Congenital spherocytosisH. GalactosaemiaI. Glucose-6-phosphate dehydrogenase deficiencyJ. Haemolytic disease of the newbornK. HypothyroidismL. Preterm birthM. Sepsis

A 6 day old infant is admitted with jaundice (serum bilirubin 300 micromol/l) and feeding poorly for the past 48 hours. The infant was born at a gestational age of 36 weeks and weighed 2680g at birth. The following are some of the clinical findings and results of investigations. In each case, select the most likely cause of the jaundice from the list of options.

1) The infant was born after a difficult forceps delivery and the head appeared to be swollen in the right parieto-occipital area. Jaundice developed after 24 hours. The haemoglobin is 12 g/dl; there is indirect hyperbilirubinaemia.C. Cephalhaematoma

2) The baby has a temperature of 38 oC and tends to bring up its feeds. The circulating white cell count is 18x109 /l. Microscopy of urine from a suprapubic aspirate shows large numbers of neutrophil polymorphs.M. Sepsis

3) The liver and spleen are palpable and there is a family history of lethargy, hepatosplenomegaly and poor feeding. The raised bilirubin is initially unconjugated; in the second sample, the conjugated fraction tended to rise. Urine examination is positive for reducing substances using Clinitest.H. Galactosaemia

4) On initial assessment, petechiae and hepatosplenomegaly are noted, and on investigation, thrombocytopenia and partially conjugated hyperbilirubinaemia are

Page 70: Mcqs Pediatrics

MCQs PEDIATRICS

Dr.Kamel Youssef Hassan, Pediatrician Consultant , Palestine - Gaza E-mail: [email protected] 70

found. Chorioretinitis is noted later and intra-cranial calcification is reported on a CT brain scan.E. Congenital cytomegalovirus infection

5) The baby was mostly breast fed and 20% of the bilirubin is conjugated. Formula milk feeds caused a rapid fall in bilirubin levels.B. Breast milk jaundice

Comments:1. A cephalhaematoma is a subperiosteal swelling which can arise as a result of forceps delivery. Complications result from anaemia due to bleeding into the haematoma, and jaundice due to haemolysis within it 2. Sepsis likely due to urinary tract infection. 3. Galactosaemia is an autosomal recessive condition due to the absence of galactose-1-phosphate uridyl transferase. This results in the intracellular accumulation of galactose-1-phosphate which is toxic. Upon commencement of milk feeds the child develops jaundice, vomiting, diarrhoea and failure to thrive. 4. Congenital Cytomegalovirus infection is typically characterized by the signs described. CNS damage from congenital CMV infection exceeds that due to any other infectious cause. 5. Breast milk jaundice is defined as � neonatal jaundice persisting for more than 28 days, with a total bilirubin level greater than 5.9mg/dl, in an otherwise healthy, thriving, breast-fed infant.

Page 71: Mcqs Pediatrics

MCQs PEDIATRICS

Dr.Kamel Youssef Hassan, Pediatrician Consultant , Palestine - Gaza E-mail: [email protected] 71

14) Theme : Connective tissue diseaseA. Ankylosing spondylitisB. DermatomyositisC. Ehlers Danlos syndrome D. Henoch Schonlein purpuraE. Juvenile chronic arthritisF. Kawasaki diseaseG. Osteogenesis imperfectaH. Rheumatic feverI. SclerodermaJ. Systemic lupus erythematosusSelect one of the above diagnoses that would explain the following presentations:1) A 10 year old boy presents with arthralgia, an urticarial rash on the lower limbs and buttocks and abdominal pain. D. Henoch Schonlein purpura

Note:Henoch Schonlein purpura also known as anaphylactoid purpura result in non-thrombocytopenic = (normal plat. Count) purpura nodules, arthritis, abdominal pain and nephritis.2) A six year old girl presents with fever, arthritis and a photosensitive rash. Dipstick of her urine reveals proteinuria.J. Systemic lupus erythematosus

Note:describes a child with SLE. A multi system disorder which results in fever, malaise, arthritis, rashes typically photosensitive rash and a butterfly rash. Kidneys are also affected resulting in glomerulonephritis. Diagnosis is made by elevated � anti-nuclear antibody, Double stranded DNA with muscle antibodies.

3) A 4 year old boy is admitted to casualty with fracture of her ulna after minor injury. She is noted to have blue sclerae and hearing impairment.G. Osteogenesis imperfecta

Note:Osteogenesis imperfecta, a syndrome of fragile bones due to defect in the α-chain of type 1collagen, which can be divided in to 4 separate types. There is widespread osteopaenia resulting in bone fragility, bow legs. Individuals characteristically have blue sclerae and hearing impairment (conductive deafness). The condition has variable inheritance with autosomal dominant and recessive forms, the latter being more severe. Blue sclera – differential diagnosis

1) Marfan syndrome2) Ehler-Danlos syndrome3) Osteogenesis imperficta

4)Pseudo-xanthoma elasticum

Page 72: Mcqs Pediatrics

MCQs PEDIATRICS

Dr.Kamel Youssef Hassan, Pediatrician Consultant , Palestine - Gaza E-mail: [email protected] 72

15) Theme : NEPHROLOGYA. Acute nephritisB. Nephrotic syndromeC. Diabetes insipidusD. Urinary tract infectionE. Chronic renal failureF. Diabetes mellitusG. Salt poisoningH. Haemolytic uraemic syndromeI. Barrter syndromeJ. CystinosisWhich diagnosis is most appropriate for the following patients?1) A 4 1/2 year old girl was admitted to hospital with a 10 day history of progressive shortness of breath and puffiness of the face. Clinical examination revealed a raised JVP, pulse rate of 160/min, normal heart sounds, hepatomegaly 2cm and bilateral basal lung crepitations. She was seen by her family doctor for a sore throat 2 weeks previously. A. Acute nephritis

Note:The picture is one of fluid overload and with a preceding URTI making post-streptococcal glomerulonephritis the most likely diagnosis. There is often macroscopic haematuria and hypertension.2) A 4 month baby boy is admitted because of inconsolable crying. He had been thriving otherwise and was well previously. He seems settled during assessment, feeds well and is afebrile. Urinalysis reveals white cells 45/HPF, red cells 10/hpf, organisms >100,000/ml. D. Urinary tract infection

Note:UTI should be ruled out in fractious pyrexial infants as the symptoms are non-specific. It is important to document whether there has been previous recent antibiotic usage and where possible a clean catch urine sample should be taken prior to starting antibiotic in this age group. Pyuria alone is not sufficient to make the diagnosis of UTI.3) A 9 year old girl has chronic renal failure of unknown aetiology. She has blond hair and blue eyes and has developed hypothyroidism. She also has cataracts that are worsening over the last 2 years.J. Cystinosis

Note:This is a description of cystinosis which is confirmed by having high white cell cystine levels. Deposits of cystine may occur in the thyroid and lense. Children typically have أشقر blond hair and blue eyes.

Page 73: Mcqs Pediatrics

MCQs PEDIATRICS

Dr.Kamel Youssef Hassan, Pediatrician Consultant , Palestine - Gaza E-mail: [email protected] 73

16) Theme : Personal and Social Development

A. 3 months B. 8 months C. 12 months D. 18 months E. 2 years F. 3 yearsG. 4 yearsH. 5 years I. 6 years

J. 7 years For each description of a child's personal and social development choose, from the list of options, the highest development age the child is most likely to have achieved.1) The baby is reliably reacting to sounds and is able to comply with a formal hearing test.B. 8 month

2) A child who is starting to seek adequate role models and assumes feelings and habit of thought.G. 4 years

3) Is friendly to strangers but is not yet ready to play games such as peek-a-boo.B. 8 months

Comments:This question relates to the personal and social aspects of normal development. 1) Between 8- 10 months babies become anxious when separated from the main carer, usually the mother for example if mother leaves the room. 2) Between 9-10 months the infant is able to wave bye bye and enjoys games such as peek-a-boo. 3) Between 18-24 months most children start to express their toilet needs. 4) Up until the age of 3 play is usually solitary, playing alongside their peers. 5) By 4 years the children usually start to role play for example pretending to be the same sex parent and play involves other children in parallel first and then interactively at around this age.

Page 74: Mcqs Pediatrics

MCQs PEDIATRICS

Dr.Kamel Youssef Hassan, Pediatrician Consultant , Palestine - Gaza E-mail: [email protected] 74

17) Theme : Emergency treatmentA. 100% Oxygen plus nebulised SalbutamolB. 100% oxygen, adrenaline and hydrocortisoneC. Adrenaline IVD. DexamethasoneE. Hyperbaric oxygenF. IntubationG. IV Hydrocortisone plus oral anti-HistaminesH. Paralysis with Sodium ThiopentoneI. Skin desensitisationJ. Supportive treatmentSelect the most appropriate emergency treatment for the following cases:1) A 13 year old boy is brought to A&E with a grossly swollen face and difficulty breathing following a bee sting. لسعة نحلةB. 100% oxygen, adrenaline and hydrocortisone

2) A 2 year old boy presents with a barking cough and stridor.D. Dexamethasone

3) A 4 month old baby with cough fever and mild respiratory distress has widespread crackles on auscultation.J. Supportive treatment

Comments:1) describes an anaphylactic reaction also known as angioneurotic oedema. In this case there is difficulty breathing i.e. life-threatening. Treatment includes 100% oxygen, IM adrenaline and IV Hydrocortisone. Intubation may benecessary. 2) The second case has croup as characterized by the barking cough. Humidified oxygen can help for mild cases but more severe cases require dexamethasone. 3) The principles of management for bronchiolitis includes supportive treatment only. There is no specific treatment other than humidified oxygen; broncho dilator therapy may be of some use

Page 75: Mcqs Pediatrics

MCQs PEDIATRICS

Dr.Kamel Youssef Hassan, Pediatrician Consultant , Palestine - Gaza E-mail: [email protected] 75

18) Theme : Fine Motor DevelopmentA. 3 monthsB. 6 monthsC. 9 monthsD. 12 monthsE. 18 monthsF. 2 yearsG. 3 yearsH. 4 yearsI. 5 yearsJ. 6 yearsFor each description of a child's fine motor development choose, from the list of options, the highest development age the child is most likely to have achieved.1) Can copy a circle and a cross and climbs and descends stairs, one foot per step.H. 4 years

2) A child can scribble and draw a straight line but not yet a circle.F. 2 years

3) Spontaneously scribbles with a crayon شخبطة and is starting to build a طباشيرtower.E. 18 months

Comments: This question relates to the fine motor area of development. 1) At 4 months the infant is able to grasp big objects and move them in to the mid-line. 2) At 6 months the baby can grasp an object such a rattle and starts to transfer it from hand to hand. He or she also starts to discover the rest of his or her body parts. 3) The pincer grip develops between 6 or 9 months and by 10 months the fine pincer grip is achieved with the baby being able to pick up small pellet like objects between thumb and forefinger. 4) At 18 months the baby starts to spontaneously scribble if offered a crayon for example 5) At 24 months the child is able to copy a straight line. 6) By age 3 years the child can copy a circle. 7) By 4 years can copy cross.8) By 4½ years can copy a square.9) By 5 years of age can copy a triangle 10) A diamond at about 6 years of age. 11) At the age of 3 the child starts to draw people usually starting as a “head and stick” extremity person and as development progresses the person becomes more sophisticated.

Page 76: Mcqs Pediatrics

MCQs PEDIATRICS

Dr.Kamel Youssef Hassan, Pediatrician Consultant , Palestine - Gaza E-mail: [email protected] 76

19) Theme : Causes of vomitingA. Cyclical vomitingB. Duodenal atresia C. Over feedingD. Meckel's diverticulumE. Ulcerative colitisF. Meconium ileusG. Gastro-enteritisH. Gastro-oesophageal refluxI. Pyloric stenosisJ. Urinary tract infectionSelect one option from the list above that is most suitable for the following patients1) A 2 day old breast fed baby is vomiting after each feed. Abdominal x-ray demonstrates a double bubble.B. Duodenal atresia

Note:Duodenal atresia is the most common cause of duodenal obstruction in infancy. It is frequently found in babies with Down's syndrome. Babies present with bilious or non-bilious vomiting and a “double bubble” are seen on abdominal x-ray. This is the result of distension of the stomach and duodenum with a constricting pylorus between them.

2) A one year old boy with a history of intermittent bleeding pr presents with features of an interceptionD. Meckel's diverticulum

Note:Meckel's diverticulum is most common under the age of 2. The Meckel's diverticulum is remnant of the vitilo-intestinal duct and can present with features of intussusception or diverticulitis. Commonly presents with painless bleedingand is diagnosed by a Meckel's scan.

3) A 4 month old baby who is thriving has persistent vomiting which is occasionally blood stained.H. Gastro-oesophageal reflux

Note:Gastro-oesophageal reflux. Reflux of gastric contents in to the oesophagus is a common event in infancy. Once the abdominal segment of the oesophagus lengthens in the first few months of life reflux improves.

Page 77: Mcqs Pediatrics

MCQs PEDIATRICS

Dr.Kamel Youssef Hassan, Pediatrician Consultant , Palestine - Gaza E-mail: [email protected] 77

20) Theme : Drugs During PregnancyA. AminoglycosidesB. AspirinC. Beta BlockersD. LithiumE. PhenytoinF. PethidineG. Sodium ValproateH. SulphonamidesI. Thiazide diuretics J. WarfarinSelect the drug from the above list of options that is most likely to be associated with the following effects if taken during pregnancy:1) This drug taken in early pregnancy may result in Ebstein's anomaly in 3% of cases.D. Lithium

2) This drug may result in a neural tube defect in approximately 2% of cases.G. Sodium Valproate

3) This drug, when taken in pregnancy may result in a baby being born with cranio-facial abnormalities, growth impairment and learning difficulties.E. Phenytoin

Comments:1) Lithium, a drug used in the treatment of bipolar affective disorder. If given in early pregnancy is associated with a wide variety of cardiac defects with approximately 8% having severe cardiac disease, Ebstein's anomaly being the most common (3% of all cases).2) Sodium Valproate tends to induce a neural tube defect (spina bifida) in approximately 2% of pregnancies and therefore should be avoided. It is also associated with abnormalities of the orofacial and digits, compromising the foetal Valproate syndrome.3) Phenytoin is well known for causing the foetal hydantoin syndrome, compromising cranio-facial abnormalities and learning difficulties as well growth impairment.

Page 78: Mcqs Pediatrics

MCQs PEDIATRICS

Dr.Kamel Youssef Hassan, Pediatrician Consultant , Palestine - Gaza E-mail: [email protected] 78

21) Theme : Abdominal PainA. Acute gastro-enteritis B. Appendicitis C. Constipation D. Diabetes MellitusE. Henoch Schonlein PurpuraF. Mesenteric AdenitisG. NephroblastomaH. PancreatitisI. Sickle Cell crisisJ. Wilm's TumourSelect one option from the list above that is most suitable for the following patients1) A 12 year old boy with a 3 week history of coryza is brought to A&E complaining of severe abdominal pain. He has swollen ankles and nodules on his buttocks, which do not disappear with pressure.E. Henoch Schonlein PurpuraNote:Henoch Schonlein purpura also known as anaphylactoid purpura has an unknown aetiology. Children present with abdominal pain, possibly melaena due to haemorrhage and oedema of the gut wall. They may have a flitting arthritis affecting large joints. The nephritis presents with microscopic haematuria and proteinuria and the macular, papular, purple rash is rather characteristic with buttocks and extensive surfaces of the legs and arms being affected.2) A 10 year old girl has just returned from Thailand. She is brought to history of severe diarrhoea associated with abdominal pain. Her mother, a known diabetic, has similar symptoms.A. Acute gastro-enteritis

Note:relates to a child with gastroenteritis. In the developed world gastroenteritis is very common and usually mild. 60% of cases are due to viruses for example Rotavirus. In bacterial gastroenteritis; fever is common and children present with colicky abdominal pain, vomiting, diarrhoea and possibly dehydration.3) A 12 year old girl with a 2 month history of weight loss and increased appetite weight loss is brought to Casualty with severe abdominal pain. She denies any history of vomiting or excessive exercise. Her mother says that the child has been wetting her bed over the past 4 months.D. Diabetes MellitusNote:describes a child with diabetes mellitus. The commonest metabolic / endocrine problem in childhood. Diabetes mellitus results from low Insulin levels resulting in abnormal metabolism of carbohydrate, protein and fats. Children present with polyuria, polydipsia and polyphagia. Children sometimes complain of abdominal discomfort and there is a history of weight loss.

Page 79: Mcqs Pediatrics

MCQs PEDIATRICS

Dr.Kamel Youssef Hassan, Pediatrician Consultant , Palestine - Gaza E-mail: [email protected] 79

22) Theme : Causes of HeadachesA. Benign intra-cranial hypertension B. Cerebral tumour C. Cluster headacheD. EncephalitisE. Extradural haematomaF. MeningitisG. MigraineH. Subdural haemorrhageI. Sub arachnoid haemorrhageJ. Tension headacheSelect the most appropriate diagnosis from the above list that would explain the following cases.1) A 16 year old girl develops headache and vomiting. She has a fever and develops a non-blanching rash.F. Meningitis2) A 17 year old male with a 4 day history of cough and fever presents with altered sensorium and vomiting.D. Encephalitis

3) A 10 year old girl with a 6 month history of episodic abdominal pain presents with a severe headache, a complete ophthalmoplegia which lasts several days.G. Migraine

Comments: 1) Bacillus Meningitis is a severe infection that carried serious morbidity and mortality. It may present suddenly or insidiously. Most commonly symptoms start off as being non-specific, for example fever, anorexia, irritability and later develop in to a rash and shock. More specific signs include neck stiffness and seizures. 2) Encephalitis refers to inflammation of the brain and children present with anencephalopathy. Initial symptoms may be non-specific or flu-like followed by changes in mental state, seizures and complications include cerebral oedema, cardiovascular instability, fluid and electrolyte disturbances. Herpes viruses have been implicated as well as other viruses and often there is a devastating consequence. 3) Migraine is a recurrent headache with symptom free interval. Other symptoms include abdominal pain, nausea or vomiting, aura, visual sensory or motor (photophobia may also occur). Other visual disturbances may take the form of blurred vision, fortification spectra and flashing lights. Complicated migraine occurs when there are neurological signs during a headache that may persist after the headache has terminated. For example hemiplegic migraine.

Page 80: Mcqs Pediatrics

MCQs PEDIATRICS

Dr.Kamel Youssef Hassan, Pediatrician Consultant , Palestine - Gaza E-mail: [email protected] 80

23) Theme : Dyspnoea in childrenA. Acute severe asthmaB. Cardiac failureC. CostochondritisD. Diabetic ketoacidosisE. Hysterical over-breathingF. Inhalation of foreign bodyG. Pleura effusionH. Pulmonary embolismI. Tension pneumothoraxJ. Pleural effusionSelect the most appropriate diagnosis from the options above that best explains the following cases:1) A 3 month old baby presents with tachycardia and tachypnoea with widespread crepitationsB. Cardiac failure

2) A 3 year old presents with cough and haemoptysis. Examination reveals reduced breath sounds over one lung and a localized wheeze.F. Inhalation of foreign body3) A boy presents with a dry cough, lethargy and shortness of breath. Examination reveals reduced breath sounds over one side which is dull to percussion.J. Pleural effusion

Comments: Heart failure – A number of congenital heart lesions will result in cardiac failure, commonly in the first year of life. The baby becomes breathless particularly after crying or feeding. The baby may be having difficulty completing feeds and as a result there may be a problem with poor weight gain. Babies also are seen to have sweating as a prominent symptom. On examination the baby may be tachycardic and tachypnoenic. A murmur may be present or a gallop rhythm. Oedema may be seen around the dependant parts of the body. Pulmonary crepitations may also be heard. It may be possible to pick up heart enlargement clinically and chest x-ray may reveal cardiomegaly and plethoric lung fields. In the second case the unilateral nature of the reduced breath sounds and wheeze strongly suggest inspissation of a foreign body. The final case has again unilateral signs but on this occasion the dullness to percussion suggest a pleural effusion.

Page 81: Mcqs Pediatrics

MCQs PEDIATRICS

Dr.Kamel Youssef Hassan, Pediatrician Consultant , Palestine - Gaza E-mail: [email protected] 81

24) Theme : Abdominal diseases of Childhood

A. AerophagiaB. Alpha 1 antitrypsin deficiencyC. GalactosaemiaD. Hirschsprung's diseaseE. IntussusceptionF. Lactose intoleranceG. Mesenteric adenitisH. Reyes syndromeI. ThalassaemiaJ. Wilson's diseaseSelect one of the above options that is the most appropriate diagnosis for the following cases:1) Usually presents with chronic liver disease often in association with neurological features.J. Wilson's diseaseNote:describes a child with Wilson's disease also known as hepatolenticular degeneration. It is an autosomal recessive condition characterized by cirrhosis of the liver, degenerative changes of the brain and Kayser-Fleischer rings on the cornea. It is as a result of low Caeruloplasmin and low Copper excretion.2) A 9 month old baby presents with a sudden onset of abdominal pain and loose stools which have the appearance of red currant jelly.E. Intussusception

Note:Intussusception is an invagination of the bowel in to an adjacent segment. It presents as paroxysmal pain occasionally with vomiting and palpation may reveal a sausage shaped mass � in the right upper abdomen. A delay in diagnosis results in the child passing blood stained mucus rectally some times known as red currant jelly. Intussusception can sometimes be reduced hydro-statically, if unsuccessful surgery is required.

3) A 4 day old infant presents with severe abdominal distension and feculent vomitingD. Hirschsprung's disease

Note:Hirschsprung's disease is absence of ganglia cells in part or all of the wall of the colon. The child presents with bilious or feculent vomiting, constipation and distension.

Page 82: Mcqs Pediatrics

MCQs PEDIATRICS

Dr.Kamel Youssef Hassan, Pediatrician Consultant , Palestine - Gaza E-mail: [email protected] 82

25) Theme : Gross motor developmentA. 1 monthB. 2 monthsC. 4 monthsD. 6 monthsE. 8 monthsF. 10 monthsG. 1 yearH. 18 months I. 2 yearsJ. 3 yearsFor each description of a child's gross motor development choose, from the list of options, the highest development age the child is most likely to have achieved.1) A baby can crawl, weight bare and is starting to cruise.F. 10 months2) A baby rolls from front to back and back to front however is not yet sitting unsupported.D. 6 months

3) A child ascends stairs using alternating feet and is able to stand on one foot for a few seconds.J. 3 yearsComments:This question relates to gross motor skills area of development.

1. At 3 months when placed prone an infant can lift his head with his arms extended. 2. By 4 months head lag is lost and this is evident when raised from supine to the sitting

position. 3. At 5 months the baby starts to roll over and 4. At 6 months the baby is starting to sit up with support for example cushions. 5. Sitting unsupported is usually seen by 6-7 months 6. At 9 months the baby can start to crawl. 7. Cruising is usually achieved by about 10 months. 8. At 12 months the baby can raise يقوم independently and often starts to take a few steps

alone. 9. At 18 months the toddler can run although this is usually in a tiff fashion. He or she can

ascend stairs with assistance 10.At 20 months the child can climb the stairs holding on to a rail.

At 3 years the child can ascend stairs in an adult fashion using alternating feet per step however is unable to descend the stairs in such a fashion until 4 years of age.

Page 83: Mcqs Pediatrics

MCQs PEDIATRICS

Dr.Kamel Youssef Hassan, Pediatrician Consultant , Palestine - Gaza E-mail: [email protected] 83

26) Theme : Treatment of Diarrhoeal illnessA. MetronidazoleB. Gluten free dietC. No actionD. SteroidsE. Lactose free dietF. Pancreatic enzyme supplementsG. Vitamin DH. High fibre dietI. Breast milk allergyJ. Praziquantel

Select one option from the list above that is most suitable for the following patients1) A 2 year old girl is taken to the A&E department by her mother who says that her child has diarrhoea and describes the stool as looking like peas and carrots.C. No action

Note:Describes a child with toddler diarrhoea due to a fast gut transit time. Children between the ages of 6 months and 3 years may have this complaint which manifests as a chronic diarrhoea. Children are otherwise healthy and thriving with all investigations carried out being normal.2) A 7 year old child presents with a 3 month history of abdominal pain. She is found to have anaemia and colonoscopy reveals a cobble stone appearance.D. Steroids

Note:Relates to inflammatory bowel disease, Crohn's disease and ulcerative colitis respectively. In Crohn's disease the colonoscopy will reveal �cobbled stonemucosal appearances with fistulae where as in ulcerative colitis � (crypt abscesses + Collar button ulcers)are present.3) A 12 year old girl whose height and weight are on the 3rd centile complains of diarrhoea with a history that the stools are difficult to flush away. She has an itchy rash over her limbs.B. Gluten free diet

Note:relates to Coeliac disease. A sensitivity and intolerance to the gliadin protein � inrice, wheat and barley. Children present with failure to thrive, diarrhoea, abdominal pain, anorexia and abdominal distension. Diagnosis is suspected by elevated (anti-gliadin / anti-reticulin / anti-endomysial) antibodies and confirmed by jejunal biopsies.

Page 84: Mcqs Pediatrics

MCQs PEDIATRICS

Dr.Kamel Youssef Hassan, Pediatrician Consultant , Palestine - Gaza E-mail: [email protected] 84

27) Theme : TREATMENT OF CHILDHOOD CONDITIONSA. AcyclovirB. CefotaximeC. ErythromycinD. GentamicinE. ImmunoglobulinF. No medicationG. ParacetomolH. PenicillinI. PrednisoloneJ. Quinine

All of the following children present with a febrile illness. Based on the symptoms presented below, select the single treatment option most likely to alter the clinical course of the disease in each case. Each option may be used once, more than once, or not at all

1) A playful and active 5 year old boy presents with a low grade fever and bruising. The rest of the examination is normal. His platelet count is 14 x 109 /LF. No medicationNote:Childhood thrombocytopenia may occur in the setting of viral illness.

2) A 10 year old girl comes back from school camp with a temperature, breathlessness and cough, which has been going on for 10 daysC. Erythromycin

Note:Outbreaks of whooping cough may in school camps. The most vulnerable children are not immunized.

3) A 5 month old baby presents with a fever, lethargy, poor feeding and a generalized convulsion. An examination of the cerebrospinal fluid shows an elevated number of neutrophil polymorphsB. Cefotaxime

Note:Meningitis should be suspected in any ill baby and may present with unusual crying, poor feeding, fever, coma, fits and vomiting. Bulging or tense anterior fontanelle, stiff neck, opisthotonus and shock may be present. For children more than three months old cefotaxime is the treatment of choice.

Page 85: Mcqs Pediatrics

MCQs PEDIATRICS

Dr.Kamel Youssef Hassan, Pediatrician Consultant , Palestine - Gaza E-mail: [email protected] 85

4) A 3 year old Asian girl presents with a high temperature for six days, cervical lymphadenopathy, conjunctivitis and a red tongue. There is a generalized rash, with desquamation of the skin of the fingers and toes.E. Immunoglobulin

Note:Kawasakis disease is characterized by bilateral conjunctival injection, oral mucosal changes with red sore tongue, polymorphous rash, lymphadenopathy and desquamation of palms and soles. Inflammatory markers are elevated and treatment is with intravenous immunoglobulin.

5) A 4 year old girl presents with a temperature of 39oC having just had a generalized convulsion lasting 3 minutes; twenty minutes later, she is completely alert again. Her older brother had similar fits at the age of 3 yearsG. Paracetomol

Note:Febrile convulsions are generalized tonic/clonic symmetrical seizures which occur with rapid rises in temperature in children between the ages 3 months and 5 years who have no history of epilepsy or CNS disease.

Page 86: Mcqs Pediatrics

MCQs PEDIATRICS

Dr.Kamel Youssef Hassan, Pediatrician Consultant , Palestine - Gaza E-mail: [email protected] 86

28) Theme : Fine Motor DevelopmentA. 3 monthsB. 6 monthsC. 9 monthsD. 12 monthsE. 18 monthsF. 2 yearsG. 3 yearsH. 4 yearsI. 5 yearsJ. 6 yearsFor each description of a child's fine motor development choose, from the list of options, the highest development age the child is most likely to have achieved.

1) A child can copy a square and a triangle but not yet a diamond. I. 5 years2) A baby transfers objects to his mouth and has a fine pincer grip using thumb and forefinger.D. 12 months

3) Can only copy a circle and builds a tower of 9 cubes.G. 3 years

Comments:This question relates to the fine motor area of development. 1) At 4 months the infant is able to grasp big objects and move them in to the mid-line. 2) At 6 months the baby can grasp an object such a rattle and starts to transfer it from hand to hand. He or she also starts to discover the rest of his or her body parts. 3) The pincer grip develops between 6 or 9 months and by 10 months the fine pincer grip is achieved with the baby being able to pick up small pellet like objects between thumb and forefinger. 4) At 18 months the baby starts to spontaneously scribble if offered a crayon for example.5) At 24 months the child is able to copy a straight line. 6) By age 3 years the child can copy a circle. 7) By age 4 years the child can copy a cross. 8) By 4½ years the child can copy a square 9) By 5 years of age a triangle, 10) A diamond at about 6 years of age. At the age of 3 the child starts to draw people usually starting as a “head and stick” extremity person and as development progresses the person becomes more sophisticated.

Page 87: Mcqs Pediatrics

MCQs PEDIATRICS

Dr.Kamel Youssef Hassan, Pediatrician Consultant , Palestine - Gaza E-mail: [email protected] 87

29) Theme : PoisoningA. AntipsychoticsB. BenzodiazepinesC. Ethylene glycolD. MethanolE. OpiatesF. OrganophosphatesG. SalicylatesH. SympathomimeticI. Tricyclic antidepressantsJ. Volatile solventsSelect the most appropriate agent from the above list that would elicit the following toxic effects:

1) Blindness, pulmonary oedema, metabolic acidosis, shock.D. Methanol

2) Coma, hypotension, hypoventilation, reduced muscle tone and diminished reflexes.B. Benzodiazepines

3) Dysphagia, Dysphoria, Trismus, oculogyric crisisA. Antipsychotics

Comments:The relevant feature of the first case is the blindness and acidosis which is probably due to Methanol, itself a weak acid but it causes lactic acidosis and optic neuritis. The second case has features of CNS depression and Benzodiazepines are typical. This should respond rapidly to Flumazenil. The third scenario with oculogyric crisis suggests antipsychotics.

Page 88: Mcqs Pediatrics

MCQs PEDIATRICS

Dr.Kamel Youssef Hassan, Pediatrician Consultant , Palestine - Gaza E-mail: [email protected] 88

30) Theme : chest pain in childrenA. Acute severe asthma B. Cardiac failureC. CostochondritisD. Diabetic ketoacidosisE. Hysterical over-breathingF. Inhalation of foreign bodyG. Pleura effusionH. Pulmonary embolismI. Tension pneumothoraxJ. Unilateral pneumoniaSelect the most appropriate diagnosis from the options above that best explains the following cases:1) 14 year old presents to Accident & Emergency with a sudden onset of chest pain and shortness of breath. Examination reveals reduced expansion and hyperresonance over one side with reduced breath sounds.I. Tension Pneumothorax

2) A 17 year old presents with a sudden onset of chest pain. Examination reveals a sinus tachycardia and a pleural rub.H. Pulmonary embolism

3) A 13 year old boy presents with tachypnoea and tachycardia. Examination of the chest reveals poor air entry, however no crackles or wheeze. He has a pulsus paradoxus.A. Acute severe asthma

Comments:Tension pneumothorax may result following trauma. It is a potentially life threatening event with air accumulating under the pleura in the pleural space, which pushes the mediastinum across the chest. Children may be shocked and will be hypoxic with reduced air entry and hyperresonance. Distended neck veins may also be visible.Pulmonary embolism is uncommon in infants and children. It may arise in children as a result of surgery for example scoliosis surgery. It may also occur following prolonged inactivity. In adolescents drug abuse, recent abortion or the oral contraceptive may be predisposing problems. Clinical manifestations may suggest pneumonia. Dyspnoea is common associated with pain. There may be no physical signs although the embolism is sufficiently large there may be a pleural friction rub. The clue in the third case the relatively silent chest associated with pulsus paradoxus. This would suggest severe asthma. Other Causes of pulsus paradoxus include � (pericardial tamponade /effusion/ constriction and myocarditis).

Page 89: Mcqs Pediatrics

MCQs PEDIATRICS

Dr.Kamel Youssef Hassan, Pediatrician Consultant , Palestine - Gaza E-mail: [email protected] 89

31) Theme : Abdominal DiseaseA. Acute gastro-enteritis B. Appendicitis C. Constipation D. Diabetes MellitusE. Henoch Schonlein PurpuraF. Mesenteric AdenitisG. NephroblastomaH. PancreatitisI. Sickle Cell crisisJ. Wilm's TumourSelect one option from the list above that is most suitable for the following patients1) A 9 year old Italian girl has severe abdominal pain. She is found to have sclerae and painful fingers. I. Sickle Cell crisis

Note:Sickle cell disease is a haemoglobinopathy caused by substitution of Valine for Glutamic acid at the 6th position of the Beta chain. It results in a haemolytic anaemia resulting in vaso-occlusive episodes leading to splenic infarction and dactylitis.2) An 11 year old boy with a 12 hour history of anorexia and polyuria complains of severe abdominal pain. On examination he has a temperature of 38°C and is tender posteriorly on pr examination. BM is 4mmol per litre.B. Appendicitis

Note:describes acute appendicitis. The commonest acute surgical emergency of childhood. Usually seen in children over the age of 5. Children present with a temperature, abdominal pain and guarding in the right iliac fossa, reboundtenderness, fever and vomiting. It is as a result of mucus obstructing the lumen causing ischaemia of the mucosa and ultimately perforation.3) An 11 year old boy with cystic fibrosis presents with recurrent attacks of abdominal pain, vomiting and fever.H. Pancreatitis

Note:relates to description of chronic Pancreatitis in a child with cystic fibrosis. It is also a feature of chronic hyperlipidaemia or congenital anomalies of the biliary retract or pancreas. Children present with vomiting and abdominal pain which radiates through tot he back. Clinical suspicion is confirmed by elevated amylase levels and calcification of the pancreas on abdominal x-ray. Treatment is supportive

Page 90: Mcqs Pediatrics

MCQs PEDIATRICS

Dr.Kamel Youssef Hassan, Pediatrician Consultant , Palestine - Gaza E-mail: [email protected] 90

32) Theme : Abdominal diseases of ChildhoodA. AerophagiaB. Alpha 1 antitrypsin deficiencyC. GalactosaemiaD. Hirschsprung's diseaseE. IntussusceptionF. Lactose intoleranceG. Mesenteric adenitisH. Reyes syndromeI. ThalassaemiaJ. Wilson's disease

Select one of the above options that is the most appropriate diagnosis for the following cases:1) A 10 year old child develops severe vomiting and becomes encephalopathic during a viral illness.H. Reyes syndrome

Note:Reyes syndrome is an acute disease of the liver often associated with a viral infection or Salicylate toxicity . Laboratory findings include abnormal liver and muscle enzymes. Treatment is supportive.

2) A 3 year old child with a 2 day history of headache and pharyngitis complains of severe abdominal pain.G. Mesenteric adenitis

Note:Mesenteric adenitis commonly accompanying upper respiratory tract infections and is due to inflammation in the abdominal lymph nodes. There is often a history of pharyngitis and abdominal pain. Examination often reveals cervical lymphadenopathy and central tenderness.

3) A 28 day old jaundice baby with early onset vomiting and diarrhoea.C. Galactosaemia

Note:Galactosaemia results as a result of galactose 1 phosphate uridyl transferase. It accumulates in kidneys, liver and brain and clinical manifestations include jaundice, hepatomegaly, hypoglycaemia, vomiting, seizures and poor weight gain. Diagnosis is made by identifying urinary galactose. Treatment is dietary adjustment.

Page 91: Mcqs Pediatrics

MCQs PEDIATRICS

Dr.Kamel Youssef Hassan, Pediatrician Consultant , Palestine - Gaza E-mail: [email protected] 91

33) Theme : Skin lesionsA. Café au lait spotsB. Capillary haemangiomaC. Port-wine stain D. Shagreen patchE. Adenoma sebaceumF. Cutis MarmorataG. Herald's patchH. Peri-oral pigmentationI. Hypopigmentation J. TuberSelect one option from the list above that is most suitable for the following patients1) A child has an abundance of birth marks associated with axillary freckling.A. Café au lait spots

Note:relates to Neurofibromatosis type 1. Café au lait lesions are brown pigmented marks found in abundance in Neurofibromatosis as well as other genetic conditions for example Tuberous sclerosis.

2) A child with learning difficulties is described as having goose like flesh.D. Shagreen patch

Note:relates to Tuberous sclerosis. It is a condition where by hamartomatous lesions become evident in early childhood. Seizures and learning difficulties are common and skin manifestations include fibrous angiomatous lesions in the naso labial folds known as adenoma sebaceum. White macules are seen over the trunk known as Ash leaf macules and Shagreen patches resemble goose like flesh. Subungual fibromata are also common.

3) A child with a history of Iron deficiency anaemia presents with per rectal bleeding and is found to have this characteristic appearance.H. Peri-oral pigmentation

Note:refers to Peutz-Jeghers syndrome characterized by peri-oral pigmentation. A history of Iron deficiency anaemia, colonoscopy may reveal polyps.

Page 92: Mcqs Pediatrics

MCQs PEDIATRICS

Dr.Kamel Youssef Hassan, Pediatrician Consultant , Palestine - Gaza E-mail: [email protected] 92

34) Theme : Personal and Social DevelopmentA. 3 monthsB. 8 monthsC. 12 monthsD. 18 monthsE. 2 yearsF. 3 yearsG. 4 yearsH. 5 yearsI. 6 yearsJ. 7 yearsFor each description of a child's personal and social development choose, from the list of options, the highest development age the child is most likely to have achieved.1) A baby shows stranger awareness, is able to wave and clap his hands and is starting to spoon feed himself.C. 12 months

2) Can dress and undress including buttons and can imitate the construction of bridges and steps with blocks.G. 4 years

3) A child is starting to indicate the need to sit on the potty, play is usually solitary at this age and the child tries to control objects over other children.E. 2 years

Comments: This question relates to the personal and social aspects of normal development. 1) Between 8- 10 months babies become anxious when separated from the main carer, usually the mother for example if mother leaves the room. 2) Between 9-10 months the infant is able to wave bye bye and enjoys games such as peek-a-boo. Between 18-24 months most children start to express their toilet needs. 3) Up until the age of 3 plays is usually solitary, playing alongside their peers. 5) By 4 years the children usually start to role play for example pretending to be the same sex parent and play involves other children in parallel first and then interactively at around this age.

Page 93: Mcqs Pediatrics

MCQs PEDIATRICS

Dr.Kamel Youssef Hassan, Pediatrician Consultant , Palestine - Gaza E-mail: [email protected] 93

35) Theme : Genital disordersA. Aarskog syndromeB. Fragile X C. Gonadal dysgenesis D. Hermaphroditism E. Polycystic ovariesF. Testicular feminisation syndrome G. XO syndromeH. XXX syndromeI. XXY syndromeJ. XYY syndromeSelect the most appropriate diagnosis from the list above that best describes the following cases1) A 12 year old girl has seizures, depression and a conduct disorder.H. XXX syndrome

Note:describes a girl with XXX syndrome. Individuals are tall and thin. They have behavioural problems and are mildly dysmorphic with epicanthic folds and hypertelorism. Seizures are common2) A 16 year old female presents with obesity, oligomenorrhoea and hirsutism.E. Polycystic ovariesNote:relates to Stein-Leventhal also known as polycystic ovarian syndrome. During adolescence there is change in body habitus and there is excess hair grown and chronic and ovulation.

3) A 14 year old phenotypic female presents with primary amenorrhoea. She has normal female external genitalia but intra-abdominal testicles with an XYkaryotype.F. Testicular feminisation syndrome

Note:The third case has testicular feminisation, a condition associated with testosterone receptor defects that result in insensitivity to testosterone with resultant female genitalia.

Page 94: Mcqs Pediatrics

MCQs PEDIATRICS

Dr.Kamel Youssef Hassan, Pediatrician Consultant , Palestine - Gaza E-mail: [email protected] 94

36) Theme : NEONATAL JAUNDICEA. Physiological jaundiceB. Biliary atresiaC. HypothyroidismD. Rhesus incompatibilityE. Congenital spherocytosisF. Congenital cytomegalo virus infectionG. GalactosaemiaH. Glucose-6-phosphate dehydrogenase deficiencyI. Cystic fibrosisJ. Fructose intoleranceFor each of the following jaundiced babies, select the most likely cause.1) A two-week-old full term baby with a birth weight of 1.9 kg. He has gained a small amount of weight already. On examination he has a white reflex on shining a bright light into both pupils, moderate jaundice, a pansystolic murmur at the left lower sternal edge, hepatosplenomegaly and a rash.F. Congenital cytomegalo virus infectionNote:Low birth weight, white reflex suggesting congenital cataracts, jaundice and hepatosplenomegaly are indicative of a congenitally acquired infection. The rash is likely to be petechiae secondary to bone marrow failure from the infection.2) A 48 hour old baby who presents with poor feeding. She was delivered at home following a concealed pregnancy. She was thought to be pale initially but her skin subsequently became more yellow. Her mother had a late miscarriage one year previously and is blood group A negative. Investigations show a haemoglobin of 10 g/dL and an unconjugated bilirubin of 426 mmol/L. The blood film shows evidence of haemolysis but otherwise normal red cell morphology.D. Rhesus incompatibility

Note:The previous miscarriage has sensitized the maternal immune system to the Rhesus antigen. Haemolysis causes an unconjugated hyperbilirubinaemia. ABO incompatibility unlikely unless the mother is blood group O.3) A 5 day old baby appears jaundiced but otherwise well and feeds normally. The serum bilirubin is checked by the community midwife and found to be moderately elevated at 298 mmol/L. It is mainly unconjugated bilirubin, and the haemoglobin is normal.A. Physiological jaundice

Note:This is a well baby and with only a slightly elevated unconjugated bilirubin. Levels can normally increase to 300 mmol/L with no adverse effects on the baby. Some reports say that levels are safe up to 400 mmol/L in a term well baby.

Page 95: Mcqs Pediatrics

MCQs PEDIATRICS

Dr.Kamel Youssef Hassan, Pediatrician Consultant , Palestine - Gaza E-mail: [email protected] 95

37) Theme : Causes of vomitingA. Gastro-oesophageal refluxB. IntussusceptionC. Meconium ileusD. Meconium plug syndromeE. Meckel's DiverticulumF. Mid gut volvulusG. Necrotising enterocolitisH. Peptic ulcerI. Pyloric stenosisJ. Tracheo-oesophageal fistula with atresiaSelect the most appropriate option from the above list that would account for the following presentations:1) A baby on the postnatal ward is seen to have a cyanotic episode during feeding.J. Tracheo-oesophageal fistula with atresiaNote:Tracheo-oesophageal fistula with oesophageal atresia results in excessive oral secretions. The baby often chokes or coughs with feeds and becomes cyanotic. There is an inability to pass a nasogastric tube and there is often a history of maternal polyhydramnios.2) A baby born at 33 weeks develops vomiting and abdominal distension on day 3. By day 4 the abdomen appears erythematous and the baby becomes apnoeic.G. Necrotising enterocolitis

Note:Necrotising enterocolitis is a life threatening condition resulting in mucosal and transmural necrosis. It is most prevalent in low birth weight babies and the babies present with abdominal distension, gastric retention, bloody stools and signs of a sepsis. Abdominal x-ray may reveal pneumatosis intestinalis.3) A term baby is admitted to Special Care Baby unit with hypoglycaemic episodes. His mother is a poorly controlled diabetic. By day 4 the hypoglycaemia has resolved but the baby has not yet opened his bowels.D. Meconium plug syndrome

Note:Meconium plug syndrome is as a result of a plug of meconium causing obstruction. There is usually an underlying disease such as cystic fibrosis, Hirschsprung's disease, infant of a diabetic mother or maternal drug abuse.

Page 96: Mcqs Pediatrics

MCQs PEDIATRICS

Dr.Kamel Youssef Hassan, Pediatrician Consultant , Palestine - Gaza E-mail: [email protected] 96

38) Theme : Treatment of Diarrhoeal illnessA. MetronidazoleB. Gluten free dietC. No actionD. SteroidsE. Lactose free dietF. Pancreatic enzyme supplementsG. Vitamin DH. High fibre dietI. Breast milk allergyJ. PraziquantelSelect one option from the list above that is most suitable for the following patients:

1) A 7 month old baby has diarrhoea, which mother feels has been brought on by the introduction of weaning. Mum is a known asthmatic.E. Lactose free diet

Note:refers to Cow's milk protein intolerance is mainly a clinical diagnosis with a history that symptoms appeared at the time of cow's milk ingestion. The disorder is usually temporary and can be managed by dietary adjustment.

2) A 13 year old Irish girl complains of diarrhoea. She is Iron deficient and denies any history of GI upset after eating rice or potatoes. She has high anti-reticulin antibodiesB. Gluten free diet

Note:refers to Coeliac disease where a gluten free diet is appropriate. High reticulin or more specifically now anti-TTG antibodies are expected. The diagnosis is confirmed by jejunal biopsy.

3) A 10 year old boy presents with faecal urgency and bloody diarrhoea. He is also under the care of ophthalmologists for a chronic iritis. D. Steroids

Note:relates to inflammatory bowel disease, Crohn's disease and ulcerative colitis respectively. In Crohn's disease � the colonoscopy will reveal cobbled stone mucosal appearances with fistulae where as in ulcerative colitis � crypt abscesses are present.

Page 97: Mcqs Pediatrics

MCQs PEDIATRICS

Dr.Kamel Youssef Hassan, Pediatrician Consultant , Palestine - Gaza E-mail: [email protected] 97

39) Theme : Causes of vomitingA. Cyclical vomitingB. Duodenal atresia C. Over feedingD. Meckel's diverticulumE. Ulcerative colitisF. Meconium ileusG. Gastro-enteritisH. Gastro-oesophageal refluxI. Pyloric stenosisJ. Urinary tract infectionSelect one option from the list above that is most suitable for the following patients1) An 8 year old girl shows signs of moderate dehydration. She has vomited all fluids of 24 hours and the vomit is not bile stained. Her abdomen is soft and she has had 3 similar episode in the past year.A. Cyclical vomiting

Note:Cyclical vomiting. This is thought to be a variant of migraine. The child presents with bouts of vomiting which may persist for days and at times a headache is associated.

2) A 12 week old infant vomits after each feed. He is developmentally normal and is fed by bottle at 260mls per kilo per day.C. Over feedingNote:Over feeding frequently results in regurgitation and vomiting. This may also be accompanied by an excessive weight gain and dietary adjustment is indicated.

3) An 11 year old boy with a 3 month history of abdominal pain and weight loss presents with bloody diarrhoea. E. Ulcerative colitis

Note:The last question relates to ulcerative colitis a form of inflammatory bowel disease. In this case the child presents with recurrent bouts of profuse bloody diarrhoea often with mucus and abdominal pain. Associated features include arthritis, pyoderma gangrenosum, erythema nodosum and anaemia.

Page 98: Mcqs Pediatrics

MCQs PEDIATRICS

Dr.Kamel Youssef Hassan, Pediatrician Consultant , Palestine - Gaza E-mail: [email protected] 98

40) Theme : Drugs to be avoided during breastfeedingA. AmiodaroneB. AspirinC. BenzodiazepineD. ChloramphenicolE. HeparinF. LactuloseG. LithiumH. SulfasalazineI. SulphonamidesJ. WarfarinWhich of the above drugs when prescribed to the mother is associated with the following abnormalities in the breast-fed infant:

1) May result in neonatal hypothyroidism.A. Amiodarone

2) May result in drowsiness and failure to thrive.C. Benzodiazepine

3) May result in neonatal diarrhoea.F. Lactulose

Comments:Administration of certain drugs to breast feeding mums may result in toxicity in the infants if the drug enters the milk in pharmacologically significant quantities. Some drugs also inhibit the babies sucking reflex. Other drugs inhibit lactation.Amiodarone needs to be avoided as it may result in neonatal thyroid abnormalities as there is a theoretical risk from the release of Iodine.Benzodiazepine may render the infant drowsy resulting in poor growth.Laxatives if used over a prolonged period may enter the breast milk resulting in the baby developing diarrhoea.

Page 99: Mcqs Pediatrics

MCQs PEDIATRICS

Dr.Kamel Youssef Hassan, Pediatrician Consultant , Palestine - Gaza E-mail: [email protected] 99

41) Theme : Paediatric endocrinologyA. 3 Beta hydroxysteroid dehydrogenase deficiency B. 17 Hydroxylase deficiency C. 21 Hydroxylase deficiency D. Adrenal hyperplasiaE. CraniopharyngiomaF. HypoadrenalismG. PhaeochromocytomaH. Panhypopituitarism I. Polyglandular endocrinopathyJ. Primary aldosteronismSelect the most appropriate diagnosis from the above options to explain the following presentations:1) An 8 year old boy presents with a history of enuresis. Examination reveals a bi-temporal hemianopia and papilloedema.E. Craniopharyngioma

Note:refers to a child with a Craniopharyngioma which is the commonest supratentorial tumour found in children. Children present with accelerated head growth or headaches and examination findings include papilloedema and a bi-temporal hemianopia This tumour may result in diabetes insipidus, hypothyroidism and growth hormone deficiency.

2) A term baby is born with ambiguous genitalia. Examination reveals a phallus with pigmented labial scrotal folds and the baby is found to be hyponatraemic.C. 21 Hydroxylase deficiency

Note:describes a baby with congenital adrenal hyperplasia. 95% of cases are due to 21 Hydroxylase deficiency and can be diagnosed by increased levels of 17OHP. The salt wasting results in hyponatraemia (Na �) and hyperkalaemia (k �).3) A 10 year old girl complains of headaches and vomiting. On examination she is noted to be pale and her blood pressure measures 135/90. Numerous café au lait lesions are found over her trunk. G. Phaeochromocytoma

Note:describes a child with Neurofibromatosis complicated by � phaeochromocytoma,which is a tumour arising from the adrenal medulla or chromaffin cells. Children present with headaches, palpitations, pallor, vomiting and night sweats. They are found to be hypertensive.

Page 100: Mcqs Pediatrics

MCQs PEDIATRICS

Dr.Kamel Youssef Hassan, Pediatrician Consultant , Palestine - Gaza E-mail: [email protected] 100

42) Theme : Drugs to be avoided during breastfeeding

A. AmiodaroneB. AspirinC. BenzodiazepineD. Chloramphenicol E. HeparinF. LactuloseG. LithiumH. SulfasalazineI. SulphonamidesJ. Warfarin

Which of the above drugs, when prescribed to the mother, is associated with the following abnormalities in the breast-fed infant:

1) May result in hypotonia and cyanosis.G. Lithium

2) May result in prolonged jaundice.I. Sulphonamides

3) May result in platelet dysfunction.B. Aspirin

Comments:Administration of certain drugs to breast-feeding mums may result in toxicity in the infants if the drug enters the milk in pharmacologically significant quantities. Some drugs also inhibit the babies sucking reflex. Other drugs inhibit lactation.Lithium is also to be avoided as it may result in reduced tone and cyanotic episodes.Aspirin may result in a possible risk of Reyes syndrome; regular use may impair platelet function, as well as producing a hypoprothrombinaemia, especially if the infant is low in Vitamin K stores.

Page 101: Mcqs Pediatrics

MCQs PEDIATRICS

Dr.Kamel Youssef Hassan, Pediatrician Consultant , Palestine - Gaza E-mail: [email protected] 101

43) Theme : Genital disordersA. Aarskog syndromeB. Fragile X C. Gonadal dysgenesis D. HermaphroditismE. Polycystic ovariesF. Testicular feminisation syndrome G. XO syndromeH. XXX syndromeI. XXY syndromeJ. XYY syndrome

Select the most appropriate diagnosis from the list above that best describes the following cases

1) A young child is noted to have Cryptorchidism and a shawl scrotum.A. Aarskog syndrome

Note:describes a boy with Aarskog syndrome. Features include mild – moderate learning difficulties, short stature, facial dysmorphic features include hypertelorism anteverted nostrils and a webbed neck. Genital abnormalities include shawl شال scrotum and cryptorchidism ► (hidden or obscure testis).

2) A 12 year old boy is noted to have a long thin face, micropenis, hypospadias and has learning difficultieJ. XYY syndrome sNote:relates to a child with XYY syndrome. Individuals are usually tall and have learning difficulties and abhorrent مشمئز behaviour.3) A child is noted to have macrocephaly and macroorchidism.B. Fragile X

Note:describes a boy with Fragile X syndrome. Features include moderate learning difficulties, speech delay and autistic features. They have large heads and prominent ears as well as testicular enlargement.

Page 102: Mcqs Pediatrics

MCQs PEDIATRICS

Dr.Kamel Youssef Hassan, Pediatrician Consultant , Palestine - Gaza E-mail: [email protected] 102

44) Theme : ABDOMINAL PAINA. PeritonitisB. AppendicitisC. ConstipationD. Urinary tract infectionE. Mesenteric lymphadenitisF. MigraineG. IntussusceptionH. Henoch-Schonlein vasculitisI. PsychosomaticJ. Shigella dysentryFor the following patients with abdominal pain, choose the most likely diagnosis.

1) A 7 year old girl who has felt anorexic since the previous evening and has vomited 3 times since. She has abdominal pain, which was made worse by the bumpy car journey to hospital. B. Appendicitis

Note:A typical description of appendicitis. In the younger child/ baby there is often a later presentation with perforation of the appendix being relatively more common, as these children are not able to articulate their pain and localization of pain is more difficult to discern with examination.

2) A 4 year old boy who has a sore throat, mild fever and peri-umbilical abdominal pain. E. Mesenteric lymphadenitis

Note:Preceding sore throat and periumbilical pain are typical of mesenteric lymphadenitis in a relatively well child.

3) A 9 year old boy has intermittent severe generalized colicky abdominal pain which is getting worse. He has started to vomit over the last 24 hours. He has a history of recurrent abdominal pain. Examination reveals a mass in the left iliac fossa.C. Constipation

Note:This is a description of chronic severe constipation, which in extreme cases can cause obstruction. There is faecal loading implying the chronicity. When it is this severe it is important to rule out other pathology and consider the possibility that is a manifestation of child abuse

Page 103: Mcqs Pediatrics

MCQs PEDIATRICS

Dr.Kamel Youssef Hassan, Pediatrician Consultant , Palestine - Gaza E-mail: [email protected] 103

45) Theme : PRECOCIOUS PUBERTYA. CraniopharyngiomaB. Exaggerated adrenarcheC. McCune-Albright syndromeD. Hypothalamic hamartomaE. Thelarche variantF. Neurofibromatosis type 1G. Granulosa-theca cell tumourH. HypothyroidismI. Premature thelarche J. Congenital adrenal hyperplasiaWhich is the most likely diagnosis in the following cases:1) A 4 year old girl present s with vaginal bleeding and café –au-lait pigmentation which stops at the midline. Subsequent investigations demonstrate gonadotrophin-independent puberty C. McCune-Albright syndrome

Note:McCune Albright syndrome is caused by an activating mutation of the GS alpha subunit. This results in autonomous secretion of certain hormones e.g. thyroxine, cortisol, oestradiol. There may be associated skin pigmentation and fibrous dysplasia.2) A 3 year old girl presents with pubic hair development, cliteromegaly and acnes. There was an acceleration in her height velocity and no symptoms of oestrogen excess. Neurological examination was normal.J. Congenital adrenal hyperplasiaNote:Virilising Congenital adrenal hyperplasia (CAH) typically presents with symptoms of androgen excess. There is often behavioural disturbance with mood swings. Diagnosis is confirmed by a raised 17 hydroxyprogesterone. Cliteromegaly and a sustained increase in height velocity does not occur in adrenarche.3) A 2year old girl presents with isolated bilateral breast development. Examination is otherwise normal as is her height velocity. Subsequent investigation demonstrates undetectable oestradiol levels and a prepubertal response to an LHRH test.I. Premature thelarcheNote:There may be a slight FSH predominance on the basal gonadotrophin levels. Premature thelarche is self limiting and thought to be the result of the body being exquisitely sensitive to low levels of oestradiol.

Page 104: Mcqs Pediatrics

MCQs PEDIATRICS

Dr.Kamel Youssef Hassan, Pediatrician Consultant , Palestine - Gaza E-mail: [email protected] 104

46) Theme : Causes of vomitingA. Gastro-oesophageal refluxB. IntussusceptionC. Meconium ileusD. Meconium plug syndromeE. Meckel's DiverticulumF. Mid gut volvulusG. Necrotising enterocolitisH. Peptic ulcerI. Pyloric stenosisJ. Tracheo-oesophageal fistula with atresiaSelect the most appropriate option from the above list that would account for the following presentations:1) 5 week old breast fed baby is admitted with a 2 week history of vomiting and dehydration. On examination an olive sized mass is palpable.I. Pyloric stenosisNote:Pyloric stenosis is most prevalent amongst first born male and age of onset is usually between 3-6 weeks of age. Babies present with projectile vomiting and examination may reveal visible peristaltic waves and an olive sized mass may be palpable in the upper epigastrium.

2) A 3 month old baby boy is admitted with episodic screaming. On examination he is mottled and has a delayed capillary refill time. Palpation of the abdomen reveals a sausage shaped mass.B. Intussusception

Note:Intussusception – a segment of bowel becomes telescoped in to an adjacent of bowel often in areas where the bowel is aperistaltic, for example Peyer's patches, Meckel's diverticulum, tumour or an area of oedema secondary to Henoch Schonlein purpura. The child presents with pain, vomiting and signs of shock. A mass may be palpable in the right upper quadrant.

3) A baby with cystic fibrosis develops vomiting and abdominal distension. A ground glass appearance is seen on x-ray.C. Meconium ileus

Note:Meconium ileus syndrome is the commonest form of small bowel obstruction and the baby presents with distension, vomiting and constipation. An x-ray may reveal a ground glass appearance.

Page 105: Mcqs Pediatrics

MCQs PEDIATRICS

Dr.Kamel Youssef Hassan, Pediatrician Consultant , Palestine - Gaza E-mail: [email protected] 105

47) Theme : Gross motor developmentA. 1 monthB. 2 monthsC. 4 months D. 6 monthsE. 8 monthsF. 10 monthsG. 1 yearH. 18 months I. 2 yearsJ. 3 yearsFor each description of a child's gross motor development choose, from the list of options, the highest development age the child is most likely to have achieved.1) A baby has good head control with no head lag when pulled from supine to sitting.C. 4 months

2) A baby can rise independently and stands momentarily alone. Is starting to take 1-2 steps.G. 1 year

3) A child who is walking and runs stiffly can climb stairs if hand is held.I. 2 yearsComments:This question tests your knowledge of the development stages of gross motor skills.

1) At 3 months when placed prone an infant can lift his head with his arms extended. 2) By 4 months head lag is lost and this is evident when raised from supine to the sitting

position. 3) At 5 months the baby starts to roll over. 4) At 6 months the baby is starting to sit up with support for example cushions. 5) Sitting unsupported is usually seen by 6-7 months and 6) At 9 months the baby can start to crawl. 7) Cruising is usually achieved by about 10 months.8) At 12 months the baby can rise independently and often starts to take a few steps

alone. 9) At 18 months the toddler can run although this is usually in a tiff fashion. He or she can

ascend stairs with assistance.10)At 20 months the child can climb the stairs holding on to a rail.

11)At 3 the child can ascend stairs in an adult fashion using alternating feet per step, however is unable to descend the stairs in such a fashion until 4 years of age.

Page 106: Mcqs Pediatrics

MCQs PEDIATRICS

Dr.Kamel Youssef Hassan, Pediatrician Consultant , Palestine - Gaza E-mail: [email protected] 106

48) Theme : SHORT STATUREA. Constitutional delay of growthB. Growth hormone deficiencyC. Psychosocial deprivationD. Familial short statureE. Skeletal dysplasiaF. HypothyroidismG. CraniopharyngiomaH. Turner’s syndromeI. Cushing’s syndromeWhich is the most likely diagnosis in the following cases?1) A previously abused 4 year old child presents with growth failure over the last 12 months. She was placed in foster care for the last 18 months and is said to much happier. Her height velocity has slowed down and she has gained weight. She suffers with constipation and her teacher has noticed her attention span has got worse recently.F. HypothyroidismNote:Height velocity will improve once abuse is removed. Constipation and poor attention span are common symptoms of hypothyroidism in children.2) A 7 year old boy was referred to clinic for investigation of short stature. He is well and apart from occasional headaches has no other complaints. His height is below the 0.4 th centile and within the target centile range calculated from his parents heights. Previous height measurements from his GP show no evidence of a reduction in height velocity.D. Familial short stature

Note:Short stature with no evidence of growth failure and a height within the target centile range indicate familial short stature which requires no further investigation.3) A 3 year old girl presents with stature well below the 0.4 th centile. She had a past history of pedal oedema at birth and uncomplicated repaired coarctation of the aorta. She has been treated for constipation in the past but this has resolved, other wise she is very well. An insulin tolerance test demonstrates a peak growth hormone level of 25 mU/l.H. Turner’s syndrome

Note:The insulin tolerance test should only be performed if there is growth data that suggests growth failure and other causes of short stature have been ruled out. GH deficiency is not suggested here. She is very short and also well i.e. other pathology unlikely. A previous history of coarctation and pedal oedema suggests Turner’s syndrome.

Page 107: Mcqs Pediatrics

MCQs PEDIATRICS

Dr.Kamel Youssef Hassan, Pediatrician Consultant , Palestine - Gaza E-mail: [email protected] 107

49) Theme : Childhood malignancies

A. Burkitt's nasopharyngeal carcinomaB. Ewing sarcomaC. HepatoblastomaD. Hodgkin's lymphomaE. MelanomaF. NesidioblastosisG. NeuroblastomaH. TeratomaI. Wilm's tumourJ. Xeroderma pigmentosumSelect the most appropriate oncological diagnosis from the above list of options to explain the following scenarios.

1) A baby is born with a solitary mass over the right buttock.H. Teratoma

2) A young child presents with fevers and a mass in the femur is noted and a diagnosis of malignancy is made. This malignancy is 30 times more common in white children compared to black children.B. Ewing sarcoma

3) A child of Nigerian parents presents with a unilateral tender cervical node. A diagnosis of malignancy is made with this malignancy being 7 times more common in Black Children compared with White children.A. Burkitt's nasopharyngeal carcinoma

Comments:Item 1 relates to a baby born with a sacrococcygeal teratoma. A solid tumour found in new-borns with girls being affected more than boys. It is a mixed germ cell tumour and may be associated with gastrointestinal or genital urinary symptoms. Ewing sarcoma is a tumour of the bone, usually found in long bones particularly the femur. It may present with pain, fever and tenderness and sometimes pathological fractures. There is a poor prognosis. It is 30 times more common in white children than black children. This genetic predisposition is contrary to the Burkitt's nasopharyngeal carcinoma which is more common in black children than white (by 7 fold). This tumour has been associated with the Epstein-Barr virus and children present with sore throats, unilateral tenderness, cervical lymphnodes, trismus and weight loss.

Page 108: Mcqs Pediatrics

MCQs PEDIATRICS

Dr.Kamel Youssef Hassan, Pediatrician Consultant , Palestine - Gaza E-mail: [email protected] 108

50) Theme : Cranial NervesA. AbducensB. Auditory C. FacialD. Glossopharyngeal E. OculomotorF. OlfactoryG. OpticH. Trochlear I. Trigeminal J. VagusWhich of the above cranial nerves is affected in the following cases?1) A 3 year old girl recovering from a pneumococcal meningitis complains of a painful right eye. The pupil is enlarged and the eyes deviated inferiorly and laterally. There is a right sided ptosis.E. Oculomotor

2) A 12 year old boy complains of a numb feeling to the left side of his face. He is unable to whistle or show is teeth on that side.C. Facial

3) A 15 year old girl on treatment for Nephrotic syndrome presents with a vesicular rash involving the right conjunctiva and right forehead, which is very tender to touch.I. Trigeminal

Comments:The oculomotor nerve innovates the superior inferior and medial rectus as well as the inferior oblique and the levator palpebrae muscles. Complete paralysis of the oculomotor nerve causes ptosis, dilatation of the pupil and displacement of the eye outwards and downwards. An upper motor neurone facial nerve palsy is diagnosed by decreased voluntary movement of the lower face with flattening of the nasal folds, where as a lower motor neurone lesion tends to involve the upper and lower facial muscles.The Trigeminal nerve – sensory distribution to the face is divided in to ophthalmic, maxillary and the mandibular routes and motor function may be tested by examining the masseter and temporalis muscles.

Page 109: Mcqs Pediatrics

MCQs PEDIATRICS

Dr.Kamel Youssef Hassan, Pediatrician Consultant , Palestine - Gaza E-mail: [email protected] 109

51) Theme : Causes of HeadachesA. Benign intra-cranial hypertension B. Cerebral tumour C. Cluster headacheD. EncephalitisE. Pseudotumour CerebriF. MeningitisG. MigraineH. Subdural haemorrhageI. Sub arachnoid haemorrhageJ. Tension headache

Select the most appropriate diagnosis from the above list that would explain the following cases.1) A 6 year old girl with a history of Still's disease presents with a 2 month history of headache and diplopia She is admitted to hospital. Investigations carried out include a normal CT scan, normal CSF profile and normal opening pressures.E. Pseudotumour Cerebri

2) A 4 month old infant presents with poor feeding and irritability. On examination he has a torticollis and his head circumference has increased form the 50th – 98th centile since his last measurement 6 weeks ago.I. Sub arachnoid haemorrhage

3) A 16 year old obese girl with a long history of headaches is found to have papilloedema. The rest of the neurological examination is normal as is the CT scan.A. Benign intra-cranial hypertension

Comments:Intra-cranial tumours- brain tumours are second to leukaemia as the most prevalent malignancy in childhood. Children present with either signs of increased intra-cranial pressure or with focal neurological signs. Infratentorial tumours are more common in the paediatric age group than supratentorial tumours and the MRI scan is the best radiological tool for delineating brain tumours. Pseudotumour cerebri is a clinical syndrome mimicking brain tumours and characterised by normal CSF pressure and cell count and a normal structure to the brain. Causes of which are multiple for example metabolic disorders, infections, drugs, haematological disorders. Treatment focuses towards the underlying aetiology and pseudo tumour cerebri is mainly self limiting. Benign intracranial hypertension is associated with pregnancy as well as drugs such as the oral contraceptive, tetracyclines and rarely Growth Hormone therapy. It is more common in association with obesity. LP usually reveals high opening pressures and treatment includes removal of any precipitant with possible serial LP and removal of CSF.

Page 110: Mcqs Pediatrics

MCQs PEDIATRICS

Dr.Kamel Youssef Hassan, Pediatrician Consultant , Palestine - Gaza E-mail: [email protected] 110

52) Theme : Causes of abdominal painA. Acute appendicitisB. Inflammatory bowel diseaseC. Irritable bowel syndromeD. Meckel's diverticulumE. Muscle strainF. Ovarian cystsG. Pelvic inflammatory diseaseH. Psoas haematomaI. PyelonephritisJ. Ureteric calculusSelect the most appropriate diagnosis from the above list for the following presentations1) A 13 year old boy with umbilical pain for the last 4 days presents with anorexia, nausea and has not passed a bowel motion for 24 hours.A. Acute appendicitis

Note:The pain of acute appendicitis starts off as a peri-umbilical pain which then locates to the right iliac fossa and is some times referred to the back. It is sharp and steady; it results in nausea, emesis and local tenderness.

2) A 10 year old boy with lower abdominal pain for the last 10 days presents with a history of passing 6-8 loose stools. Temperature is 38.8°C. He is tender in the right lower quadrant and has an anal fistula.B. Inflammatory bowel diseaseNote:Inflammatory bowel disease results in recurrent lower abdominal pain, often radiating to the back. The pain is dull and crampy in nature and is associated with fever, weight loss and tenesmus.

3) A 14 year old haemophiliac patient presents with lower abdominal pain and a limp. Attempts to straighten his leg results in excruciating painH. Psoas haematoma

Note:Bleed into the Psoas muscle which may be due to coagulopathy or secondary to sepsis (DIC) results in back pain radiating to the hip with painful flexion at the hip joint.

Page 111: Mcqs Pediatrics

MCQs PEDIATRICS

Dr.Kamel Youssef Hassan, Pediatrician Consultant , Palestine - Gaza E-mail: [email protected] 111

53) Theme : NORMAL DEVELOPMENTA. 4 weeks B. 2 months C. 7 months D. 10 months E. 12 monthsF. 18 monthsG. 2 yearsH. 3 yearsI. 4 yearsJ. 5 yearsAt what age would you expect the average child to achieve the following milestones:-1) Show casting, able to pick up small objects with a neat pincer grasp and have developed concept of object permanence. E. 12 months

2) Able to skip, draw a triangle from copy, name the heavier of 2 weights, count up to ten and ask questions about meaning of words. J. 5 years

3) Able to identify coloursH. 3 years

Comments:

It is important to be familiar with the developmental milestones, particularly up to the age of 3 years.

Milestones are grouped under 4 headings:

Gross motor Fine motor Hearing and language social

You should know the age range for each milestone. Developmental milestones are in Illingworth: 'The Normal Child'.

Page 112: Mcqs Pediatrics

MCQs PEDIATRICS

Dr.Kamel Youssef Hassan, Pediatrician Consultant , Palestine - Gaza E-mail: [email protected] 112

54) Theme : ChemotherapyA. 6-Mercaptopurine B. Actomyosin DC. AsparaginaseD. Bleomycin E. CisplatinF. CyclophosphamideG. Cytosine arabinoside H. DoxorubicinI. MethotrexateJ. VincristineSelect the most appropriate chemotherapeutic agent from the above list that corresponds with the following actions and side effects:

1) An alkylating agent resulting in haemorrhagic cystitis.F. Cyclophosphamide

2) A drug inhibiting protein synthesis may result in a dose related Pancreatitis.C. Asparaginas

3) An anthracycline with a serious risk of cardio-toxicity. H. Doxorubicin

Comments:All cytotoxic agents have adverse effects. General side-effects include nausea, vomiting and bone marrow suppression, alopecia and stomatitis. More specific side-effects are listed in the questions above. Alkylating agents such as Cyclophosphamide may cause a haemorrhagic cystitis. Asparaginase results in a dose related Pancreatitis.The Anthracyclines e.g. (Doxorubicin and Daunorubicin) are cardiotoxic, which is often very difficult to detect. Monitoring with echocardiograms is advised.Vincristine is an alkaloid agent and results in sensory motor neuropathy with long-term use. It also has an affect on the autonomic system resulting in severe constipation and paralytic ileus. It may also result in sensory changes with parathesis progressing to loss of tendon reflexes.Bleomycin is the main drug resulting in lung damage and occurs in up to 10% of patients. The damage is dose related.

Page 113: Mcqs Pediatrics

MCQs PEDIATRICS

Dr.Kamel Youssef Hassan, Pediatrician Consultant , Palestine - Gaza E-mail: [email protected] 113

55) Theme : SYNDROMESA. Down's syndromeB. Turner's syndromeC. Noonan's syndromeD. Klinefelter's syndromeE. Treacher-Collins syndromeF. Marfan's syndromeG. Sotos syndromeH. Klippel-feil syndromeI. Laurence Moon Biedl syndromeJ. MCune Albright syndromeWhich is the most applicable diagnosis for the following children?1) A boy with learning difficulties and short stature is found to have pulmonary stenosis when investigated for a murmur. He has some dysmorphic features including wide carrying angle of the arms. C. Noonan's syndrome

Note:Children with Noonan’s syndrome have short stature and may have delayed puberty. There is also associated short sightedness. Pulmonary stenosis is the most frequent congenital heart lesion.

2) Boy is seen in cardiology clinic for investigation of a murmur heard on routine check. His father died at an early age from aortic dissection. He is noted to be above the 98th centile in height for his age. F. Marfan's syndromeNote:Marfans syndrome is also associated with a high arched palate, arachnodactly and a predisposition to pneumothoraces.3) A child with learning difficulties was noticed to be long at birth. He has a large head, downslanting eyes, bossed forehead and prominent jaw. He is clumsy and previously been investigated for hypotonia.G. Sotos syndrome

Note:This is a typical description of Sotos syndrome which is one of the overgrowth syndromes.

Page 114: Mcqs Pediatrics

MCQs PEDIATRICS

Dr.Kamel Youssef Hassan, Pediatrician Consultant , Palestine - Gaza E-mail: [email protected] 114

56) Theme : Connective tissue diseaseA. Ankylosing spondylitisB. DermatomyositisC. Ehlers Danlos syndrome D. Henoch Schonlein purpuraE. Juvenile chronic arthritisF. Kawasaki diseaseG. Osteogenesis imperfectaH. Rheumatic feverI. SclerodermaJ. Systemic lupus erythematosusSelect one of the above diagnoses that would explain the following presentations:1) A 12 year old girl presents with oesophageal dysmotility, interstitial pneumonitis and has cardiac conduction abnormalities (arrhythmias).I. SclerodermaNote:describes a child with scleroderma, a chronic fibrotic disturbance which tends to be relapsing and remitting. It affects the GI tract, heart lungs and kidneys.

2) A 5 year old presents with conjunctivitis fever and desquamation of the periphery. F. Kawasaki diseasNote:Kawasaki's disease � is also known as mucocutaneous lymph node syndrome. It is a clinical diagnosis made by a history of fever for 5 days, bilateral conjunctivitis, mucosal involvement and peripheral changes including erythema and desquamation. Cardiac involvement is present in approximately 30% of cases and this is a serious complication.

3) A 10 year old girl presents with weight loss, peri-orbital oedema, weakness and is noted to have a photosensitive rash.B. Dermatomyositis

Note:relates to the diagnosis of Dermatomyositis, a multi system disease where individuals have a violaceous heliotrope rash around the eyes, Guttron’s patches on the knuckles and inflammation of muscles and joints resulting in weakness in muscles and arthritis. The condition is often associated with underlying malignancy.

Page 115: Mcqs Pediatrics

MCQs PEDIATRICS

Dr.Kamel Youssef Hassan, Pediatrician Consultant , Palestine - Gaza E-mail: [email protected] 115

57) Theme : Causes of abdominal painA. Acute appendicitisB. Inflammatory bowel diseaseC. Irritable bowel syndromeD. Meckel's diverticulumE. Muscle strainF. Ovarian cystsG. Pelvic inflammatory diseaseH. Psoas haematomaI. PyelonephritisJ. Ureteric calculusSelect the most appropriate diagnosis from the above list for the following presentations1) A 7 year old boy presents with a 2 weeks history of flank pain and rigors, intermittent fevers.I. PyelonephritisNote:Pyelonephritis - an infection of the renal tract causes acute sudden pain in the lower abdomen and back. This is associated with urinary frequency and dysuria, costochondral tenderness and fever.

2) A 13 year old girl presents with recurrent abdominal pain and haematuria. She is apyrexialJ. Ureteric calculusNote:Urolithiasis or urinary tract calculi present with haematuria (macro or microscopic) abdominal pain, dysuria and voiding abnormalities.

3) An 8 year old girl with a history of recurrent central abdominal pain presents with pallor and an Iron deficiency anaemia.D. Meckel's diverticulumNote:Meckel's diverticulum results in peri-umbilical sharp pain associated with haematocheziaHaematochezia (bright red blood per rectum)

Page 116: Mcqs Pediatrics

MCQs PEDIATRICS

Dr.Kamel Youssef Hassan, Pediatrician Consultant , Palestine - Gaza E-mail: [email protected] 116

58) Theme : Causes of headache and feverA. Bacterial meningitisB. Congenital rubellaC. Congenital toxoplasmosisD. Herpes simplex encephalitisE. Leukaemic infiltrationF. Meningococcal meningitisG. Pneumococcal meningitisH. Status epilepticusI. Tuberculous meningitisJ. Varicella cerebellitisSelect the most likely diagnosis from the above list that would account for the following presentations1) A child is admitted with alternating levels of consciousness and a high temperature. EEG shows periodic, lateralised epileptiform discharges.D. Herpes simplex encephalitis

2) A baby is born at term weighing 2.5kilos. He is noted to have microcephaly. An ultrasound scan of his head reveals calcification.C. Congenital toxoplasmosis3) A baby of 6 months has a full septic screen. His CSF shows 10 white cells predominantly lymphocytes, high protein and low glucose.I. Tuberculous meningitisComments:Examination of the cerebral spinal fluid is essential in confirming the diagnosis of conditions such as meningitis, encephalitis and sub arachnoid haemorrhage. The main contraindication for performing a lumbar puncture is any condition that may lead to “Coning” or Herniation of intra-cranial contents. Therefore fundoscopy to exclude the presence of papilloedema is essential. Routine CSF study includes red cell count, white cell count, glucose protein and bacterial cultures. In meningitis typical CSF findings include �A neutrophilic pleocytosis with an elevated protein and reduced glucose. Gram stains reveal organisms in approximately 80% of cases of bacterial meningitis.In encephalitis CSF is typically � normal. In Tuberculous meningitis � the cell count is elevated and the predominance of lymphocytes. Glucose is reduced (1/3 the serum) however the protein concentration is elevated (0.5-6 ��) (normal protein g/l 0.2-0.4). With Leukaemic infiltration of the central nervous system atypical � Leukaemic cells may be present in the CSF. Further cytopathology studies may be indicated. Congenital toxoplasmosis �affected infants may have fever, rash, petechiae, hepatosplenomegaly. They may have dysmorphic features such as microcephaly or hydrocephalus, microphthalmia and chorioretinitis. Cerebral calcification is common. Prognosis is guarded as affected patients may have persistent neurological disability. (Brain Calcification in CMV infection �periventricular , in toxoplasmosis � widespread No calcification in rubella)

Page 117: Mcqs Pediatrics

MCQs PEDIATRICS

Dr.Kamel Youssef Hassan, Pediatrician Consultant , Palestine - Gaza E-mail: [email protected] 117

59) Theme : Cranial NervesA. AbducensB. Auditory C. FacialD. Glossopharyngeal E. OculomotorF. OlfactoryG. OpticH. Trochlear I. Trigeminal J. VagusWhich of the above cranial nerves is affected in the following cases?

1) A 16 year old female who is pregnant complains of headaches and has blurring of the disc margin on fundoscopy.G. Optic

2) A baby is born with an ipsilateral diffuse birth mark over the right side of his face. He develops seizures in the second week of life.I. Trigeminal

3) A 15 year old boy recovering from a head injury is unable to abduct his right eye beyond the mid-line.A. Abducens

Comments: The first case has probable benign intracranial hypertension causing papilloedema. The optic nerve is examined by fundoscopy as well as visual acuity and visual fields. The Trigeminal nerve – sensory distribution to the face is divided in to ophthalmic, maxillary and the mandibular routes and motor function may be tested by examining the masseter and temporalis muscles. The abducens nerve innovates bilateral rectus muscle of the eye therefore paralysis causes medial; deviation of the eye and inability to abduct the eye beyond the mid-line.

Page 118: Mcqs Pediatrics

MCQs PEDIATRICS

Dr.Kamel Youssef Hassan, Pediatrician Consultant , Palestine - Gaza E-mail: [email protected] 118

60) Theme: Childhood syndromes.A. Alport syndromeB. Down’s syndromeC. Duchenne muscular dystrophyD. Edward’s syndromeE. Gaucher’s syndromeF. Klinefelter’s syndromeG. Pierre robin syndromeH. Sturge webber syndromeI. Treacher collins syndromeJ. Turner’s syndromeFor each patient below choose from the list above the single most likely diagnosis from the list above. Each option may be chosen more than once or not at all.1) A neonate is seen with short chin cleft palate and feeding and respiratory difficulties.G. Pierre robin syndrome

Note:Pierre Robin syndrome. Results in neonatal difficulty in feeding, micrognathia (short chin due to hypoplasia of mandible) ± cleft palate and eye abnormality. These babies should be nursed prone to prevent obstructive apnoea.2) A 9 month old baby with low short receding chin low set ears, deafness and notching of lower eye lids.I. Treacher collins syndrome

Note:Treacher Collins syndrome. Due developmental abnormalities of first and second pharyngeal pouch. Low set ears (80%), deafness 40%, under developed mandible.3) A 16 year old girl presents with primary amenorrhoea, short stature, and radio-femoral delay.J. Turner’s syndrome

Note:Turner’s syndrome. 45XO Chromosomal abnormality. Associated with short stature (<130 cm), primary amenorrhoea, rudimentary gonads, wide carrying angle, webbing of the neck cardiac abnormalities.4) A 6 week old baby with low set ears, receding chin umbilical hernia and rocker bottom feet.D. Edward’s syndrome

Note:Edward’s syndrome (Trisomy 18). Rigid baby, flexion of limbs, mental retardation. Mean survival less than 10 months.5) A 5 year old boy presents with waddling gait and positive Gower’s sign.C. Duchenne muscular dystrophy

Note:Duchenne’s muscular dystrophy X-linked recessive .

Page 119: Mcqs Pediatrics

MCQs PEDIATRICS

Dr.Kamel Youssef Hassan, Pediatrician Consultant , Palestine - Gaza E-mail: [email protected] 119

61) Theme : Childhood BreathlessnessA. Asthma B. Bronchiolitis C. BronchiectasisD. Broncho pulmonary dysplasia E. Cardiac failureF. LaryngomalaciaG. Tracheal haemangiomaH. Tracheal oesophageal fistulaI. Vitamin D deficiencyJ. Vocal cord palsySelect the most appropriate choice from the above list that is the most appropriate diagnosis in the following cases1) A premature baby presents with a cough and cyanotic episodes during feeds.H. Tracheal oesophageal fistula

2) A girl with hypogammaglobulinaemia develops haemoptysis and a purulent sputum. C. Bronchiectasis

3) A baby is born by a ventouse delivery following failed forceps. She is born in poor condition and stridor is noted shortly after birth.J. Vocal cord palsyComments: Item 1describes a baby with tracheo-oesophageal fistula, which typically presents with a cough and choking cyanotic episodes during feeds. Bronchiectasis typically produces clubbing and haemoptysis as well as persistent crepitations in the chest. This process may follow a severe pneumonia for example pertussis, measles, TB. It can also complicate immune-deficiency states and Cystic fibrosis. Kartagener's syndrome is a triad of 1) bronchiectasis 2) sinusitis 3) dextrocardia. The features result from abnormal ciliary function. Vocal cord paralysis is the answer to Item 6. Stridor from birth may occur as a result of birth trauma or an abnormality of the central nervous system, such as hydrocephalus.

Page 120: Mcqs Pediatrics

MCQs PEDIATRICS

Dr.Kamel Youssef Hassan, Pediatrician Consultant , Palestine - Gaza E-mail: [email protected] 120

62) Theme : Paediatric endocrinologyA. 3 Beta hydroxysteroid dehydrogenase deficiency B. 17 Hydroxylase deficiency C. 21 Hydroxylase deficiency D. Adrenal hyperplasiaE. CraniopharyngiomaF. HypoadrenalismG. PhaeochromocytomaH. Panhypopituitarism I. Polyglandular endocrinopathyJ. Primary aldosteronismSelect the most appropriate diagnosis from the above options to explain the following presentations:1) A child is admitted to ITU with meningococcaemia. On arrival hyponatraemia and hyperkalaemia is revealed. Urinary electrolytes reveal high levels of Sodium and Chloride and low levels of Potassium.F. HypoadrenalismNote:describes a child with meningococcaemia complicated by hypoadrenalism of Waterhouse-Friderichsen syndrome. This is due to haemorrhage into the adrenal glands. If the condition is unrecognised adrenal crisis may result and therefore treatment must be immediate and vigorous (Hydrocortisone, IV fluids and Glucose).2) A child attending diabetic clinic complains of feeling tired. She is found to have thin hair, patches of vitiligo and the tongue is coated in white. Blood tests reveal low Calcium and high Phosphate.I. Polyglandular endocrinopathyNote:describes a child with Polyglandular endocrinopathy. In this case a child with diabetes, vitiligo, Addison's disease, hypoparathyroidism and chronic mucocutaneous Candidiasis (type 1 Polyglandular endocrinopathy.) Type 2 Polyglandular endocrinopathy is diagnosed if there is history of hyperthyroid disease or diabetes plus Addison's disease.3) A baby is born at term. Birth weight is on the 50th centile. Within the first few weeks hypoglycaemia is diagnosed. The growth of the baby is monitored closely and By 9 months of age parameters have fallen to the second centile.H. PanhypopituitarismNote:The last question describes a baby with growth hormone deficiency. From the above option Panhypopituitarism is the answer. Babies with growth hormone deficiency present with episodes of hypoglycaemia and may present as an emergency with apnoeic episodes or cyanotic episodes. They are usually normal in birth size; however by 1 year of age growth has tailed off. In Panhypopituitarism the deficiency may be of growth hormone only or of multiple hormones and therefore laboratory investigations should include thyroid function tests, cortisol and ACTH, gonadotrophin's and prolactin as well as growth hormone.

Page 121: Mcqs Pediatrics

MCQs PEDIATRICS

Dr.Kamel Youssef Hassan, Pediatrician Consultant , Palestine - Gaza E-mail: [email protected] 121

63) Theme : WEAKNESS IN THE LOWER LIMBS

A. Guillain-Barre syndrome B. Transverse MyelitisC. Multiple sclerosisD. Chronic fatigue syndromeE. Spinal tumourF. DermatomyositisG. Congenital myopathyH. Duchenne muscular dystrophy I. Fascioscapulohumeral dystrophyJ. Spinal abscess

For these children with lower limb weakness select the most likely diagnosis

1) A 4 year old boy is brought to clinic by his mother who feels he has weak legs and is finding climbing the stairs difficult. He walked independently at 20 months and has received speech therapy for a mild speech delay. On examination his lower limb reflexes are normal, his muscles are well developed distally although weak proximally.H. Duchenne muscular dystrophy

Note:DMD occurs in boys(X-linked recessive) and presents with delayed walking i.e.> 18months. Speech delay, cardiomyopathy, scoliosis and calf pseudohypertrophy are also features.

2) A 12 year old girl presents with a 5 day history of progressive weakness in her legs and is now unable to walk. She has been passing urine frequently and with difficulty. On examination there is power with gravity eliminated in her legs, very brisk ankle and knee jerks and upgoing plantar responses. Her bladder is palpable up to the umbilicus. Sensation is reduced to touch below T5, vibration and position sense are preserved 3 weeks previously she had a presumed viral illness. MRI brain is normal but shows swelling and increased signal over the spinal cord T5-7 on T1- weighted images. Lumbar puncture shows 33 lymphocytes and monocytes per microlitre and protein 570 mg/L in the CSF.B. Transverse Myelitis

Note:The features are suggestive of a lower motor neurone lesion which could also be

Page 122: Mcqs Pediatrics

MCQs PEDIATRICS

Dr.Kamel Youssef Hassan, Pediatrician Consultant , Palestine - Gaza E-mail: [email protected] 122

caused by a spinal tumour / abscess however the MRI would demonstrate these latter pathologies. The CSF lymphocytosis and increased protein may be seen in transverse myelitis. Transverse myelitis has unknown aetiology but has been postulated to have a viral or mycoplasma origin.

3) A 14 year old girl has been off school for 4 months with headaches, tiredness and weakness of her legs. In the initial weeks of her illness she had a low-grade fever, sore throat and cervical lymphadenopathy. A blood count showed atypical lymphocytes and liver function tests show a raised AST. These were re-checked 1 month later when they had normalised and thyroid function, urine culture urea & electrolytes were also normal. It is difficult to get her full co-operation for examination but there are no objective signs of weakness and reflexes are all present, symmetrical with downgoing plantar responses.D. Chronic fatigue syndrome

Note:The previous viral illness and subsequent lethargy with no consistent abnormal investigations suggest chronic fatigue syndrome.

Page 123: Mcqs Pediatrics

MCQs PEDIATRICS

Dr.Kamel Youssef Hassan, Pediatrician Consultant , Palestine - Gaza E-mail: [email protected] 123

64) Theme : FRACTURES IN CHILDRENA. Accidental fractureB. Child physical abuseC. OsteoporosisD. RicketsE. Metabolic bone disease of prematurityF. HypoparathyroidismG. Hypophosphatemic ricketsH. PseudohypoparathyroidismI. Osteogenesis imperfectaJ. OsteopetrosisFor each patient select the most likely diagnosis.1) A 15 month girl is in foster care following child protection procedures when she presented with a spiral fracture at the age of 11 months. Her mother is single, poorly supported and had been suffering from postnatal depression at the time. There had been no explanation offered for the original injury. She now presents with a greenstick fracture of the femur following a trivial fall in the care of the experienced foster-mother who has been known to social services for many years. She is just walking independently. Skull X-ray shows several Wormian bones.I. Osteogenesis imperfectaNote:Wormian bones are indicative of osteogenesis impefecta. This condition is an important differential of NAI.2) An 11 month old girl is brought to casualty refusing to move her left arm. An x-ray demonstrates a spiral fracture of the humerus. The mother says that 30 minutes previously she lifted her daughter out of the bath and lost grip of her right hand and her body momentarily twisted round her left arm.A. Accidental fracture

Note:There is no delay in presentation and the mechanism of injury fit the description of the episode.3) A 10 year old girl presents with back pain. A spinal X ray shows collapsed lumbar vertebrae at L3 and L4 and generalized osteopenia. She has a history of 3 previous limb fractures after relatively trivial falls. She is otherwise healthy and well-grown for her age.I. Osteogenesis imperfectaNote:Collapsed vertebra and osteopenia are uncommon in children. Coupled with low trauma fractures and no dysmorphism, type 1 osteogenesis imperfecta is likely.

Page 124: Mcqs Pediatrics

MCQs PEDIATRICS

Dr.Kamel Youssef Hassan, Pediatrician Consultant , Palestine - Gaza E-mail: [email protected] 124

65) Theme : FRACTURES IN CHILDRENA. Accidental fractureB. Child physical abuseC. OsteoporosisD. RicketsE. Metabolic bone disease of prematurityF. HypoparathyroidismG. Hypophosphatemic ricketsH. PseudohypoparathyroidismI. Osteogenesis imperfectaJ. OsteopetrosisFor each patient select the most likely diagnosis.1) A toddler of 3 years presents to the accident and emergency department with swelling of his leg and is found to have a spiral fracture of the tibia. His mother reports that he had tripped and fallen the previous day but she had not noticed any sign of injury at the time. She is a single parent with little family support. The child is not on the child protection register.B. Child physical abuse

Note:It is imperative to determine whether the mechanism of injury is compatible with the history. There is delay in presentation and also spiral fracture indicates “twisting injury” rather than falling over.2) An infant is admitted with symptoms and signs of respiratory infection and is found to have several posterior rib fractures on chest radiograph. He was born prematurely at 37 weeks gestation and was observed overnight on the special care baby unit for tachypnoea which settled by the following day. On assessment it is also apparent that his head circumference has increased at an excessive rate and has crossed 3 centiles since birth.B. Child physical abuse

Note:Posterior rib fractures are highly suggestive of non accidental injury (NAI). The excessive head growth is likely to be secondary to a previous shaking injury which has resulted in an intracerebral bleed and hydrocephalus.3) A 4 month baby girl is presented with swelling of her right arm and is found to have a spiral fracture of the humerus. She had been in the care of her mother’s boyfriend who reported that he had nearly dropped her that day when reaching for her bottle and had inadvertently pulled on her arm to save her.A. Accidental fracture

Note:This history could be compatible with an accidental injury.

Page 125: Mcqs Pediatrics

MCQs PEDIATRICS

Dr.Kamel Youssef Hassan, Pediatrician Consultant , Palestine - Gaza E-mail: [email protected] 125

66) Theme : DELAYED MILESTONESA. 4 weeks B. 2 months C. 7 months D. 10 months E. 12 monthsF. 18 monthsG. 2 yearsH. 3 yearsI. 5 yearsJ. 6 yearsAt what age would you refer a child for investigation if they were unable to do the following:-1) Speak in intelligible short sentences G. 2 years

2) Be reliably dry at night (i.e. primary nocturnal enuresis) whilst continent of urine and faeces by day and otherwise well and asymptomatic. J. 6 years3) Be able to sit unsupportedD. 10 months

Comments:

It is important to be familiar with the developmental milestones, particularly up to the age of 3 years.

Milestones are grouped under 4 headings:

Gross motor Fine motor Hearing and language Social

You should know the age range for each milestone. Developmental milestones are in Illingworth: 'The Normal Child'.

Page 126: Mcqs Pediatrics

MCQs PEDIATRICS

Dr.Kamel Youssef Hassan, Pediatrician Consultant , Palestine - Gaza E-mail: [email protected] 126

67-Theme : GASTRO-INTESTINAL DISORDERS IN NEONATES

A. Ano-rectal atresiaB. Hirschsprung’s diseaseC. Infantile hypertrophic pyloric stenosisD. Intestinal atresiaE. IntussusceptionF. Meconium ileusG. Necrotizing enterocolitisH. Tracheo-oesophageal fistulaI. Volvulus neonatorum

Select the most appropriate diagnosis from the list above that would explain the presentation of the following neonates:

1) A newborn baby girl presents with gross abdominal distension and bilious vomiting. She has also got cystic fibrosis and her abdominal x-ray shows distended coils of bowel, but no fluid levels.F. Meconium ileus

2) A premature infant (31-week gestation) presents with distended and tense abdomen. She is passing blood and mucus per rectum, and she is also manifesting signs of sepsis.G. Necrotizing enterocolitis

3) A newborn baby boy presents with mild abdominal distension and failure to pass meconium after 24 hours. X-ray reveals dilated loops of bowel with fluid levels.B. Hirschsprung’s disease

Comments:One in 15 000 newborns will have a distal small bowel obstruction secondary to abnormal bulky and viscid meconium. Ninety percent of these infants will have cystic fibrosis and the abnormal meconium is the result of deficient intestinal secretions. This condition presents during the first days of life with gross abdominal distension and bilious vomiting. X-ray of the abdomen shows distended coils of bowel and typical mottled ground glass appearance. Fluid levels are scarce as the meconium is viscid. Necrotizing enterocolitis is more common in premature infants. Mesenteric ischemia causes bacterial invasion of

Page 127: Mcqs Pediatrics

MCQs PEDIATRICS

Dr.Kamel Youssef Hassan, Pediatrician Consultant , Palestine - Gaza E-mail: [email protected] 127

the mucosa leading to sepsis. Terminal ileum, caecum and the distal colon are commonly affected. The abdomen is distended and tense, and the infant passes blood and mucus per rectum. X-rays of the abdomen shows distended loops of intestine and gas bubbles may be seen in the bowel wall. Hirschsprung’s disease is an absence of ganglion cells in the neural plexus of the intestinal wall. It is more common in boys than girls. The delayed passage of meconium together with distension of abdomen is the usual clinical presentation. A plain abdominal x-ray will demonstrate dilated loops of bowel with fluid levels and a barium enema can be helpful when it demonstrates a cone with dilated ganglionic proximal colon and the distal aganglionic bowel failing to distend.

Page 128: Mcqs Pediatrics

MCQs PEDIATRICS

Dr.Kamel Youssef Hassan, Pediatrician Consultant , Palestine - Gaza E-mail: [email protected] 128

7- PEDIATRICS BEST REPONSE.

1) The following laboratory results were returned in a 6 week old boy admitted with 6 days of severe projectile vomiting:

pH 7.51 (7.35-7.45)PO2 12 KPa (95 mmHg)PCO2 4.7 KPa (35 mmHg)Blood Urea 11 mmol/lSodium 131 mmol/lPotassium 3 mmol/lChloride 83 mmol/l

Which of the following is true concerning this patient?

1-He has respiratory alkalosis2-He is likely to have a bulging anterior fontanelle3-He should be resuscitated immediately with normal saline4-He should be commenced immediately on half strength soy protein, low lactose formula5-X-ray of abdomen is likely to show dilated loops of small bowelComments:He has a metabolic, rather than respiratory alkalosis as CO2 is not reduced. He is likely to have a sunken fontanelle as he is very dehydrated.Pyloric stenosis would highly likely as because of the age and hypochloraemic, hypokalaemic metabolic alkalosis. He should be resuscitated with normal saline first. Dilated bowel would not be expected, as pyloric rather than small bowel obstruction is present.

Page 129: Mcqs Pediatrics

MCQs PEDIATRICS

Dr.Kamel Youssef Hassan, Pediatrician Consultant , Palestine - Gaza E-mail: [email protected] 129

2) A 7 year old girl presents with a 3 day history of rash and ankle swelling. She had a cold 4 weeks previously, but has otherwise been healthy. 39+1/40, 2.96kg. No neonatal problems. No drugs or medications. Fully immunized. On examination she has palpable non-blanching purple spots 1-4 mm in diameter especially over the shins and buttocks. Her left ankle is swollen, warm and tender, with restricted movement. What is the most likely diagnosis?

1-Viral infection2-Trauma/ child abuse3-Streptococcal infection4-Vasculitis eg HSP5-ThrombocytopaeniaComments: The history is of preceding URTI followed by vasculitis on the shins and buttocks, and ankle swelling. This is classical of Henoch-Schonlein purpura. The pathological lesion is a vasculitis, hence the lesions are often palpable. In contrast thrombocytopaenic purpura are not raised. The classical features are rash, joint swelling, haematuria, and GI symptoms (vomiting, abdominal pain, PR bleeding, occasional intussusception).

3) A 2 month old baby is admitted with seizures and failure to thrive. Follwoing a seizure he is found to have a blood glucose concentration of 1.2 mmol/l. Other biochemistry reveals a lactic acidosis and hyperlipidaemia. Examination reveals hepatomegaly with palpable kidneys. What is the most likely diagnosis?1-Gaucher's disease2-McArdle's disease3-Niemann-Pick disease4-Pompe Disease5-Von Gierke diseaseComments: This baby has hypoglycaemia accounting for fits, associated with lactic acidosis and hepatomegaly which would point to a diagnosis of von Gierke's disease. This is a glycogen storage disease due to deficiency of glucose-6-phosphatase. Glycogen is stored in both liver and kidney causing enlargement. Both Gaucher and Niemann-Pick disease are lipid storage disorders. McArdle's disease is a glycogen storage disease affecting muscle rather than liver as is Pompe's disease. Neither would be expected to produce profound hypoglycaemia.

Page 130: Mcqs Pediatrics

MCQs PEDIATRICS

Dr.Kamel Youssef Hassan, Pediatrician Consultant , Palestine - Gaza E-mail: [email protected] 130

4) A 15 month old child with flexural eczema presents with a 3 day history of cough and wheeze. There is a family history of hay fever. On examination his saturations are 92% in air, with a respiratory rate of 55/min and moderate recession. The heart rate is 150/min.

What is the most likely diagnosis?

1-Gastroesophageal reflux2-Allergic rhinitis3-Sinusitis4-Asthma5-Croup

Comments:This child has virus-induced wheeze on an atopic background. He may well be an early asthmatic, in which case recurrent episodes of cough and wheeze can be anticipated.

5) An overweight twelve year old boy attends A+E complaining of hip and knee pain after a minor fall from his bike. On examination his knee appears normal but there is restricted range of movement at the hip. The likely diagnosis is:

1-Chondromalaichae patella2-Osteosarcoma3-Perthe’s disease4-Septic arthritis5-Slipped upper femoral epiphysis

Comments:Slipped upper femoral epyphesis is most common in obese, adolecent boys with a positive family history. 25% are bilateral. Presentation may be with a coxalgic externally rotated gait, decreased internal rotation, thigh atrophy and hip, thigh and knee pain.

Page 131: Mcqs Pediatrics

MCQs PEDIATRICS

Dr.Kamel Youssef Hassan, Pediatrician Consultant , Palestine - Gaza E-mail: [email protected] 131

6) A 15 year old boy presents with a painful lower right arm after falling from his motorbike. X-ray reveals a fracture of the distal radius. What is the name for this fracture?1-Bennett's fracture2-Colle's fracture3-Galeazzi' s fracture4-Monteggia' s fracture5-Pott's fracture

Comments: Bennett's fracture is an intra-articular fracture of the base of the first metacarpal. Galeazzi's fracture involves the radial shaft with dislocation of the distal radioulnar joint.Pott's fracture is a general term applied to fractures around the ankle.

7) A 12 day old girl presents with poor feeding, vomiting and lethargy. Born at 37+6/40, 2.98kg, no neonatal problems. Uncomplicated pregnancy and delivery. No drugs or medications. No immunizations. No FH/SH of note.

On examination she is thin and wasted. She has occasional twitching of the eyelids and mouth. She responds slightly to voice. Weight <3%, OFC 25%, temperature is 36.5°C (tympanic), RR 40/min, HR 150/min. Chest clear, no murmur. No organomegaly. No rash or dysmorphic features.

What is the most likely diagnosis?

1-Congenital adrenal hyperplasia2-Congenital heart disease3-Inborn errors of metabolism4-SepsisComments:This child is dehydrated, vomiting and lethargic. Examination suggests possible subtle fits. Although infection and a cardiac abnormality are possible, a metabolic condition should be strongly suspected. Evaluation should include electrolytes, NH4, blood gases, serum and urine aminoacids, and urine organic acids.

Page 132: Mcqs Pediatrics

MCQs PEDIATRICS

Dr.Kamel Youssef Hassan, Pediatrician Consultant , Palestine - Gaza E-mail: [email protected] 132

8) 7 year old girl presents with high fever and severe left-sided throat pain. Shehas had difficulty in swallowing over the last 2 days, and has been finding it increasingly uncomfortable to open her mouth. Her voice is muffled and she dribbles saliva. She was born at 41/40 gestation weighing 4.0kg and there were no neonatal problems.

On examination she looks ill. Her temperature is 40.2°C (tympanic), RR 15/min and HR 100/min. ENT examination shows left tonsillar enlargement and exudate, with a uvula deviated to the right.

What is the most likely diagnosis?

1-Retropharyngeal abscess2-Foreign body aspiration3-Anaphylaxis4-Croup5-Peritonsillar abscessComments:The history suggests a peritonsillar abscess (quinsy) on the left side, a complication of Group A Strep. sore throat. A fever greater than 39.4°C is associated with severe disease, and treatment is by surgical drainage.

9) Feeding difficulties in the neonatal period may be due to all of the following except:

1-Prematurity.2-Hiatus hernia.3-Cerebral birth trauma.4-Physiological jaundice5-Congenital heart diseaseComments:All of the options are associated with feeding difficulty except for physiological jaundice which is benign, shortlived and not generally associated with symptoms.

Page 133: Mcqs Pediatrics

MCQs PEDIATRICS

Dr.Kamel Youssef Hassan, Pediatrician Consultant , Palestine - Gaza E-mail: [email protected] 133

10) Features of Down’s syndrome include all of the following except:1-A third fontanelle.2-Duodenal atresia.3-Ataxic gait.4-Increased incidence of hypothyroidism.5-Susceptibility to acute leukaemiaComments: Cerebellar dysfunction is not a feature of Down’s syndrome, however an Alzheimer’s like syndrome of memory loss and cognitive decline develops when patients reach the mid thirties.

11) Which of the following statements regarding messenger RNA (mRNA) is correct?

1-mRNA never contains introns.2-mRNA is translated into proteins in the nucleus.3-mRNA contains the bases cytosine and thymine.4-Reverse transcriptase uses mRNA as a template to produce complementary DNA.5-mRNA is used in the Southern blotting technique.Comments: The structure of mRNA is similar to DNA except that uracil replaces thymine as one of the bases. Both coding (exons) and non-coding regions of DNA are initially transcribed into mRNA. Splicing is required for mature mRNA to be produced only consisting of introns. Translation occurs in the cytoplasm. Southern blotting is a technique that uses denatured fragments of DNA in a gel to bind to DNA probes in order to detect the presence of particular genes or sequences of DNA. The enzyme reverse transcriptase can be used by viruses to insert viral mRNA into the host genome.

Page 134: Mcqs Pediatrics

MCQs PEDIATRICS

Dr.Kamel Youssef Hassan, Pediatrician Consultant , Palestine - Gaza E-mail: [email protected] 134

12) Which of the following is a recognized treatment for complications of cystic fibrosis?1-DNAase to assist in reinflating collapsed lung segments.2-Rectal pull-through and anastamosis for rectal prolapse.3-Pancreatic transplant for diabetes mellitus.4-Nebulised Tobramycin for pseudomonas colonization of the lower respiratory tract.5-Hypotonic saline drinks for hypernatraemic dehydration.Comments:Human recombinant DNAase given as a single daily aerosol seems to improve pulmonary function, decrease the frequency of chest exacerbations, and promotes a sense of well-being in patients with mild to moderate disease with purulent secretions. This may be because, in the inflamed airway, the nuclei from dead cells accounts for much of the viscidity of secretions. Rectal prolapse is usually idiopathic, occurring between 1 and 5 years. Intestinal parasites, malnutrition, acute diarrhoea, ulcerative colitis, pertussis, Ehler's Danlos Syndrome, meningocele, cystic fibrosis, and chronic constipation can also predispose to it. Following defecation the prolapse usually resolves spontaneously, or through manual reinsertion by the patient or parent. Nebulised Tobramycin or Gentamicin may be given when airway pathogens are resistant to oral antibiotics, or where infection is difficult to control at home. Hypernatramic dehydration should be treated in the usual way.

13) Characteristic features of physiological jaundice include all of the following except:

1-Onset after the first 24 hours of life.2-Disappearance by the tenth day of life.3-A serum bilirubin level below 250 mmol/L.4-Normal feeding behaviour5-Associated anaemia.Comments:Physiological jaundice occurs in 90% of infants and manifests after the first 2-3 days of life it is related to increased red cell breakdown and relative ‘immaturity’ of hepatic function. Anaemia if present would imply pathological cause of jaundice eg haemolytic anaemia.

Page 135: Mcqs Pediatrics

MCQs PEDIATRICS

Dr.Kamel Youssef Hassan, Pediatrician Consultant , Palestine - Gaza E-mail: [email protected] 135

14) A 3 year old by presents with a 2d history of coryza and cough, particularly at night. On examination he has Harrison's sulci and is on the 3rd centile for weight and height. Chest auscultation reveals scattered wheeze.

What is the most likely diagnosis?

1-Gastroesophageal reflux2-Allergic rhinitis3-Sinusitis4-Asthma5-CroupComments:The examination findings suggest chronic undertreated asthma. If he does not respond rapidly to asthma therapy a sweat test may be indicated to exclude CF. It is likely that he will respond to regular inhaled low-dose steroids.

15) A 4 year old girl is referred with behavioural problems. Her speech is generally well-formed. However, she finds it difficult to sustain conversations because she talks obsessively about her own interests. She avoids eye contact. She plays on her own for hours with her dolls, dressing and undressing them repeatedly. Attempts to play with others results in tantrums Born at .نوبات غضب39+6/40, 3.18kg no neonatal problems. No drugs, allergies. Fully immunized. No FH/SH of note.

On examination she is on the 50% for height and weight. There are no specific abnormalities to find.

What is the most likely diagnosis?1-Speech delay, isolated2-Pervasive developmental disorder3-Global developmental delay4-Impaired hearing5-AutismComments:This child has problems with reciprocal social interaction, with repetitive obsessive play. Her speech is normal in form. These symptoms are characteristic of Asperger syndrome. This is part of the spectrum of Pervasive Developmental Disorders, and is regarded by some as 'high-functioning autism'. Such children may be regarded as eccentric by their peers.

Page 136: Mcqs Pediatrics

MCQs PEDIATRICS

Dr.Kamel Youssef Hassan, Pediatrician Consultant , Palestine - Gaza E-mail: [email protected] 136

16) A 9 year old boy presents with a history of headache and persistent green nasal discharge. At night he has a cough and snores loudly. The headache is exacerbated by leaning forwards.

On examination he is apyrexial, but has a persistent nasal obstruction and nasal speech. He is tender over the maxillae and forehead.

What is the most likely diagnosis?

1-Gastroesophageal reflux2-Allergic rhinitis3-Sinusitis4-Asthma5-CroupComments:The picture is one of upper airways obstruction associated with nasal discharge, most likely due to sinusitis. In this case the maxillary and frontal sinuses are most likely to be involved.

17) 15 year old boy is knocked out for 2 minutes while playing rugby. He recovers rapidly, but cannot recall the incident. Apart from mild asthma requiring beta-agonists before sport he is otherwise well.

On examination his temperature is 36.5°C (tympanic), RR 12/min and HR 65/min. His GCS is 15/15 and no focal neurological abnormalities are present.

What is the most likely diagnosis?

1-Child abuse2-Head injury3-Hypoglycaemia4-Diabetic ketoacidosis5-Substance abuse

Comments:He has had a traumatic concussion, which refers to a diffuse brain injury. It is not necessary to be knocked out to have concussion. The Colorado Medical Society guidelines grade such injuries into 3 groups:

Page 137: Mcqs Pediatrics

MCQs PEDIATRICS

Dr.Kamel Youssef Hassan, Pediatrician Consultant , Palestine - Gaza E-mail: [email protected] 137

1. Confusion alone 2. Confusion and amnesia 3. Confusion, amnesia and loss of consciousness

To avoid the Second Impact Syndrome return to sport should be allowed after 20 mins (group 1), 1 week (group 2) or 1 month (group 3) respectively.

18) A 17 year old student presented with recurrent attacks of dizziness. Which one of the following additional features is most suggestive that she has an anxiety disorder?

1-elevated diastolic blood pressure2-nocturia3-paraesthesiae in the hands4-rotational vertigo5-tinnitusComments: Parasthesiae is often associated with anxiety of any description. Rotational vertigo and tinnitus suggest an organic disorder, whilst anxiety disorder would normally be associated with raised systolic blood pressure

19) A young boy is presents with aches and pains. Which of the following features would argue against a diagnosis of osteomalacia?

1-Bow legs2-raised alkaline phosphatase3-Low serum phosphate level4-Normal serum calcium5-advanced bone age

Comments: Clinical features include: bowing of tibia and fibula, craniotabes (soft skull), thickening of forearm at the wrist, and of the costo chondral junction (rachitic rosary). Dietary Vitamin D deficiency stimulates secondary hyperparathyroidism, which causes increased renal excretion of phosphate, low serum phosphate, and reduced urinary calcium excretion. Calcium concentrations may be low or low normal. Raised alkaline phosphatase results from osteoblasts forming unmineralised matrix. Short stature and delayed bone age are associated, rather than advanced bone age.

Page 138: Mcqs Pediatrics

MCQs PEDIATRICS

Dr.Kamel Youssef Hassan, Pediatrician Consultant , Palestine - Gaza E-mail: [email protected] 138

20) A 16 year old girl presents with a 2 day history of deteriorating breathlessness and dyspnoea. Blood gas analysis shows a pH of 7.25, a PCO2

of 7.0kPa, a PO2 of 8.5kPa, and a base excess of -4. Which of the following interpretations is correct?

1-Results are consistent with bronchopulmonary dysplasia.2-Blood gases suggest type 1 respiratory failure.3-Immediate intubation is required.4-Results are consistent with late severe asthma.5-Bicarbonate may be necessary to correct the acidosis.

Comments:In interpreting blood gas results, the following sequence may be useful:

Inspect the pH: Is it low, normal or high? Inspect the CO2: Is it low, normal or high? Inspect the PO2: Is it low, normal or high?

If the pH is low then an acidosis is present, and inspecting the CO2 will enable you to determine whether this is due to respiratory or metabolic causes. Inspecting the PO2 will tell you whether the patient is hypoxic or not. In this case, the pH is reduced, and the CO2 is high, with a base deficit of only -4, insufficient to explain the acidosis from metabolic causes. This is, therefore, a respiratory acidosis, and the PO2 is also low suggesting type 2 respiratory failure. Possible causes would include severe pneumonia, end stage asthma or neurogenic causes such as guillain-Barre. In asthma, the initial stages show a low CO2, with this climbing only to accompany failing respiration. The results would therefore be consistent with late severe asthma. In bronchopulmonary dysplasia, there is usually long-term CO2 retention with compensatory increase in bicarbonate leading to a positive base excess and normal pH. Bicarbonate is usually only considered if the base deficit exceeds about -8.

Page 139: Mcqs Pediatrics

MCQs PEDIATRICS

Dr.Kamel Youssef Hassan, Pediatrician Consultant , Palestine - Gaza E-mail: [email protected] 139

21) An 18 month old boy because of maternal concerns about delayed speech. He was born at 39+4/40 weighing 2.6kg and there were no neonatal problems. He sat at 12/12, and now pulls to stand. He is able to make a tower of 2 inch-high bricks. He is saying 2-3 single words which his mother can understand and seems to hear well. He drinks from a bottle. He is fully immunized, on no medications, and there is no FH/SH of note.

On examination he looks well. He is on the 25th centile for height and weight and OFC. There are no specific findings of note.

What is the most likely diagnosis?

1-Speech delay, isolated2-Pervasive developmental disorder3-Global developmental delay4-Impaired hearing5-AutismComments:The history suggests moderate global developmental delay. He is functioning at about the 12 month level in gross motor, fine motor and vision, hearing and speech, and social development. The history gives no indication of an underlying cause for the delay. A detailed developmental assessment should be performed plus investigations for an underlying cause. At minimum these should include TFTs and karyotype to rule out Fragile X Syndrome. Other investigations may be indicated if a specific disorder is suspected.

Page 140: Mcqs Pediatrics

MCQs PEDIATRICS

Dr.Kamel Youssef Hassan, Pediatrician Consultant , Palestine - Gaza E-mail: [email protected] 140

22) A 15 month old girl presents with stridor and respiratory distress. She has had a crusty nasal discharge for 2 days with low grade fever. She went to bed as usual at 8pm, but awoke 2 hours later. Previous history was unremarkable.

On examination she has a fever of 37.8°C (tympanic), RR of 55/min (crying), and HR of 150/min. She has marked stridor with moderate recession and a barking cough. O2 saturations are 95% in air.

What is the most likely diagnosis?1-Retropharyngeal abscess2-Foreign body aspiration3-Anaphylaxis4-Croup5-Peritonsillar abscessComments:The history suggests a viral croup (laryngotracheobronchitis). Oral dexamethasone or inhaled budesonide are effective in reducing symptoms, but are not required in mild cases. Reassurance of the mother often calms down both child and mother.

23) A 14 year old girl presents with a history of cough and breathlessness on exercise. She has seasonal rhinitis, and admits to have started smoking. Clinical examination is unremarkable.

What is the most likely diagnosis?1-Gastroesophageal reflux2-Allergic rhinitis3-Sinusitis4-Asthma5-CroupComments:A typical history of asthma in later childhood, with exercise-induced symptoms and a general deterioration on commencement of smoking. Unfortunately this is all too common these days.

Page 141: Mcqs Pediatrics

MCQs PEDIATRICS

Dr.Kamel Youssef Hassan, Pediatrician Consultant , Palestine - Gaza E-mail: [email protected] 141

24) A 10 year old boy attends A+E with a two day history of a limp. He has recently been unwell. On examination he is apyrexial and movement at his hip is uncomfortable.1-Congenetal dislocation.2-Perthes disease3-Septic arthritis4-Slipped upper femoral epiphesis5-transient synovitis

Comments:Transient synovitis is the most common cause of hip pain in childhood but other more serious causes should be excluded before it is diagnosed. The cause is unknown but it can be related to viral infection, allergic reaction or trauma.

25) A 9 week old infant is brought in unresponsive. Mother says she has well apart from episodes of screaming every evening, which last several hours at a time. She was born at 38+2/40 weighing 3.1kg and there were no neonatal problems. Parents are unmarried teenagers.

On examination she has 3 small possible bruises on her face. She has a temperature of 34.5°C, RR 30/min (irregular pattern), HR 130/min. She is floppy with a full fontanelle. Fundoscopy shows flame-shaped haemorrhages. BM stix is 3.9 mmol/l.

What is the most likely diagnosis?

1-Child abuse2-Hepatic failure3-Hypoglycaemia4-Diabetic ketoacidosis5-Substance abuse

Comments:The history suggests a previously healthy child with colic of inexperienced parents. The facial bruising suggests grip marks. Given the full fontanelle, comatose state of the child, and flame-shaped haemorrhages, Shaken baby syndrome is most likely. It is important to exclude infection with DIC or a coagulopathy.

Page 142: Mcqs Pediatrics

MCQs PEDIATRICS

Dr.Kamel Youssef Hassan, Pediatrician Consultant , Palestine - Gaza E-mail: [email protected] 142

26) A 2 year old child presents with delayed language development. He was born at term weighing 3.21kg and there were no neonatal problems. Gross motor, vision and fine motor development are normal. Mother complains that he 'doesn't listen' especially when he is in another room. His speech is restricted to single words, and only mother understands them.

On examination he is apyrexial, RR 16/min, HR 90/min. No dysmorphic features. Chest is clear, there is no murmur nor organomegaly. ENT examination reveals normal tonsils, clear nose and bilaterally dull drums.

What is the most likely diagnosis?1-Speech delay, isolated2-Pervasive developmental disorder3-Global developmental delay4-Impaired hearing5-AutismComments:The child has hearing impairment and isolated language delay. The examination suggests bilateral 'glue ear' (Otitis media with effusion). This usually resolves with time, though it may require tympanostomy tube insertion. These children often need Speech Therapy input.

27) A 3 year old girl presents with a 5 day history of neck pain and fever, with increasing difficulty in swallowing. Over the previous night she woke several times with difficulty in breathing. She was born at 36+6/40 weighing 2.8kg and there were no neonatal problems.

On examination her head held to the right. She has audible stridor when agitated, and she has very tender glands in the neck. Her temperature is 38.6°C, RR 20/min and HR 100/min. She has moderate neck stiffness.

What is the most likely diagnosis?

1-Retropharyngeal abscess2-Foreign body aspiration3-Anaphylaxis4-Croup5-Peritonsillar abscess

Page 143: Mcqs Pediatrics

MCQs PEDIATRICS

Dr.Kamel Youssef Hassan, Pediatrician Consultant , Palestine - Gaza E-mail: [email protected] 143

Comments:The history suggests a tonsillitis progressing to retropharyngeal abscess with increasing upper airways obstruction. ENT examination in children with stridor should take place where facilities are available for immediate intubation. In this case it revealed a massively enlarged right tonsil with profuse exudate. This required surgical drainage and removal with antibiotic cover.

28) A 2.5 year old boy is referred with behaviour problems and poor speech. He speaks very seldom, and is unable to sustain a conversation. He always prefers to play on his own, lining up his Thomas the Tank engine set repetitively, ignoring his older and younger siblings. He throws tantrums when his routine is disturned. He was born at 37+3/40 weighing 3.51kg and there were no neonatal problems. Pregnancy had been uncomplicated.

On examination he is apyrexial, and there are no specific abnormalities to find.

What is the most likely diagnosis?1-Speech delay, isolated2-Pervasive developmental disorder3-Global developmental delay4-Impaired hearing5-AutismComments:The history is of poor speech development, ritualistic behaviour and lack of reciprocal social interaction. The most likely diagnosis is Autism, which starts before 30 months of age and is characterized by a qualitative impairment in verbal and nonverbal communication, in imaginative activity, and in reciprocal social interactions. It is part of the wider spectrum of pervasive developmental disorders. This includes: autism, Aspergers, childhood disintegrative disorder, and Rett's syndrome.

Page 144: Mcqs Pediatrics

MCQs PEDIATRICS

Dr.Kamel Youssef Hassan, Pediatrician Consultant , Palestine - Gaza E-mail: [email protected] 144

29) A 12 year old boy complains of persistent clear nasal discharge during the spring and summer. He constantly rubs his nose and his eyes. Sometimes he has an associated cough. The symptoms usually get better in the autumn. He was diagnosed with asthma at 2 years, but this has improved with age. He rarely requires beta agonist inhaler.

On examination he has a clear nasal discharge, nasal speech, and pink non-purulent conjunctivae.

What is the most likely diagnosis?1-Gastroesophageal reflux2-Allergic rhinitis3-Sinusitis4-Asthma5-Croup Comments:This atopic boy has classical symptoms of seasonal rhinitis. Although skin prick testing may be helpful in resistant cases, it is usual to treat sufferers symptomatically with oral antihistamines or topical steroids to eyes and nose.

30) 5 month old boy has had a mild coryza for 2 days. Mother hears him making odd noises on the baby monitor. When she investigates she finds him floppy, pale and not breathing. She stimulates him, attempts mouth-to-mouth resuscitation and calls an ambulance. They give bag ventilation and he starts breathing again. O2 is given on the way to hospital. Born at 32/40 gestation weighing 1.7kg he required 3d ventilation for surfactant-deficient lung disease. Since discharge he has been thriving.

On arrival at hospital he is self-ventilating in facemask oxygen, with saturations of 94%. Temperature is 37.8°C, with RR 35/min, mild recession, and HR of 140/min. Scattered coarse crepitations are audible in both lung fields.

What is the most likely diagnosis?1-Acute life-threatening event2-Cardiac dysrhythmias3-Seizures4-Sudden infant death syndromeComments:

Page 145: Mcqs Pediatrics

MCQs PEDIATRICS

Dr.Kamel Youssef Hassan, Pediatrician Consultant , Palestine - Gaza E-mail: [email protected] 145

This child has RTI followed by acute life-threatening event (ALTE). There are a large number of potential causes. In this case bronchiolitis is most likely, given the examination findings.

31) A 3 year old child presents with delayed speech. Although he seems to understand full sentences and commands he says only occasional single words. He was born at 41/40 weighing 3.4kg and there were no neonatal problems. He is fully immunised, on no medications, and comes from a middle class articulate family.

On examination he is on the 25th centile for height, weight and OFC. There are no specific abnormalities to find. ENT examination is unremarkable.

What is the most likely diagnosis?

1-Speech delay, isolated2-Pervasive developmental disorder3-Global developmental delay4-Impaired hearing5-AutismComments:The history is of isolated speech delay in a child who apparently hears normally. Hearing should be formally checked, as it is the commonest reason for speech delay.In this case the problem involves mainly expression rather than reception of speech. Such speech problems may interfere significantly with schooling.

32) The parents of an eight year child divorce. The child lives with his mother with his father visiting at weekends. On his father's visits, the child is sullen and irritable but when his father leaves the boy cries, clings to his father and askes his father to come back to live with his mother. What is the most appropriate response that the father can make during these episodes?1-Big boys don't cry2-I left your mother but I am not leaving you.3-I will see you next weekend4-You take care of the house now I've left

Page 146: Mcqs Pediatrics

MCQs PEDIATRICS

Dr.Kamel Youssef Hassan, Pediatrician Consultant , Palestine - Gaza E-mail: [email protected] 146

5-Sorry son but I just couldn't live with your motherComments: The child's reactions typically represent a feeling of abandonment and it is important that his father reaffirms that he is not leaving his son.

33) An infant is diagnosed with pyloric stenosis. Which of the following is true of this diagnosis?

1-Typically presents between 6 and 12 months of age.2-Is more common in females.3-There is a strong familial predisposition.4-Is due to hypertrophy of the longitudinal muscle layer of the pylorus.5-Presents with bile-stained projectile vomiting.Comments: Infantile hypertrophic pyloric stenosis presents most commonly at approximately 6 weeks of age. It is more common in boys with a male: female ratio of 4:1. There is a strong familial predisposition with 5% of affected infants having a mother who also developed the problem. Pathologically, the pyloric 'tumour' arises from hypertrophy the circular muscle layer. Patients present with projectile vomiting that is not bile stained. Following resusciation and correction of any electrolyte abnormality, surgical treatment is by pyloromyotomy.

34) A study has been designed to investigate whether a certain drug plus physiotherapy treatment is better than drug treatment alone in the management of rheumatoid arthritis. After randomizing the patients a small proportion of the drug plus physiotherapy group decide to drop out of the study or omit some treatment sessions specified in the research protocol. What is the correct way of analysing the subsequent data?

1-Assume the patients have withdrawn their consent2-Exclude these patients from all analysis3-Extend the trial recruitment to make up the numbers4-Include these patient outcomes in the drug plus physiotherapy group5-Interview the patients and report their group separately

Page 147: Mcqs Pediatrics

MCQs PEDIATRICS

Dr.Kamel Youssef Hassan, Pediatrician Consultant , Palestine - Gaza E-mail: [email protected] 147

Comments: This is the principle of 'intention to treat'. It is possible that the physiotherapy intervention was harmful to the patients and this is why they left. Intention to treat helps to reduce bias by sticking to the original allocation of treatment and analyzing the patient in that treatment group even (and concentrate for this bit) even if they don''t get it!

35) A 17 year old female presents with acute breathlessness. She has had asthma for approximately 3 years and recently commenced new therapy. Which agent may be responsible for this exacerbation?1-Salmeterol2-Theophylline3-Beclomethasone4-Ipratropium bromide5-MonteleukastComments: Salmeterol has been reported to produce an acute exacerbation of asthma, possibly through an acute hypersensitivity reaction.

36) A 17 year old male is brought to clinic as his parents are concerned about changes in his behaviour. Which of the following suggest a diagnosis of Schizophrenia?

1-incongruity of affect تضارب المشاعر–مشاعر متضاربة 2-auditory hallucinations with clouding of consciousness3-memory impairment4-feelings of panic in buses and shops5-grandiose ideations

Comments: Incongruity of affect is emotion inappropriate to circumstances. There may be intellectual defects from prolonged institutionization or اإلقامة بمؤسسات ذوى الھاھاتtreatment rather than the illness itself. Auditory hallucinations with clouding of conciousness suggest delirium and memory impairment possible organic brain disease. The panic in crowds suggests an anxiety disorder and grandiose ideations suggest hypomania

Page 148: Mcqs Pediatrics

MCQs PEDIATRICS

Dr.Kamel Youssef Hassan, Pediatrician Consultant , Palestine - Gaza E-mail: [email protected] 148

37) An 18 year old female is reluctant to eat food that is prepared for her. Which one of the following would be most consistent with a diagnosis of anorexia nervosa?1-she believes the food is poisoned2-she has a full-time job3-she has bouts of heavy drinking4-she regards herself as ill5-she secretly abuses anabolic steroids

Comments: Anorexia nervosa is associated with the abnormal perception of body image. They generally feel well despite the protestations of others who feel that they look awful. They exercise avidly and until the very late stages of the disease hold down full time jobs. There is no delusion with regard to the food being poisoned. Heavy drinking associated with food refusal would suggest alcoholism and alcoholic gastritis. The secretive abuse of laxatives would fit with the diagnosis rather than anabolic agents.

38) Which of the following organelles contains enzymes responsible for the digestion of constituents of cells and tissues?1-endoplasmic reticulum2-Gogli apparatus3-lysosomes4-microtubules5-mitochondriaComments:The lysosomes contain the enzymes and molecules such as oxidases, free radical etc responsible for the breakdown of intracellular components. Micortubules are involved in mitotic processes and intracellular transportation. The mitochondria produce energy for cellular functions.

Page 149: Mcqs Pediatrics

MCQs PEDIATRICS

Dr.Kamel Youssef Hassan, Pediatrician Consultant , Palestine - Gaza E-mail: [email protected] 149

39) Which one of the following is true regarding protein metabolism?1-There are 5 essential amino acids.2-The absence of any of the essential amino acids results in a negative nitrogen balance3-Protein constitutes about 30-45% of total daily calorie intake in infants and children4-Essential amino acids can be synthesized in the liver and kidneys5-Human milk as the only food provides an insufficient source of protein for a 1 year old healthy infant

Comments: There are 9 essential amino acids; all are required to maintain nitrogen balance. Carbohydrates constitute 30-45% of total caloric intake in children. Essential amino acids cannot be synthesized and must be obtained from the diet. Human milk contains the proteins casein, lactoglobulin and lactalbumin, which provide a sufficient source of protein.

40) Apoptosis is the process of programmed cell death and occurs in cells that have damaged DNA. A mediator of this process is a tumour suppressor gene that inhibits mitosis and promotes apoptosis. This gene is:-

1-bcl-22-caspases3-fas (CD95)4-p535-ras

Comments: bcl-2 is an inhibitor of apoptosis. fas is a cell receptor and caspases are present in all cells both promote apoptosis but are not tumour suppressor genes. ras is an oncogene.

Page 150: Mcqs Pediatrics

MCQs PEDIATRICS

Dr.Kamel Youssef Hassan, Pediatrician Consultant , Palestine - Gaza E-mail: [email protected] 150

41)Which one of the following conditions is transmitted by means of an X-linked recessive inheritance :1-Cystic fibrosis2-Galactosaemia3-Huntington’s chorea4-Duchenne muscular dystrophy5-Facio-humero-scapular dystrophyComments:Cystic fibrosis and Galactossaemia are autosomal recessively inherited conditions. Haemophilia and Duchenne muscular dystrophy are X- linked recessive and Facio-humero-scapular dystrophy and Huntington’s chorea are inherited in an autosomal dominant manner.

42)A 10 year old child is found to have neurofibromatosis. Which of the following statements correctly applies:1-She will most probably be mentally retarded2-The finding of 2 café au lait spots in her 6 year old sibling suggests he may be affected3-Her condition was inherited in an autosomal recessive fashion4-Freckling of the back would be expected5-There may be no family history of the conditionComments:Mental retardation may be a clinical feature of the condition but is not a usual clinical feature. Six or more café au lait spots before the age of puberty would assist in making a diagnosis, along with other clinical findings such as Lisch nodules, and optic nerve gliomas. The condition is autosomal dominant. Freckling of the inguinal and axillary regions occur. 30-50% of new cases are due to spontaneous mutations.

Page 151: Mcqs Pediatrics

MCQs PEDIATRICS

Dr.Kamel Youssef Hassan, Pediatrician Consultant , Palestine - Gaza E-mail: [email protected] 151

43)Which of the following statements applies to fragile X syndrome?

1-Mental retardation is found in all males2-Mental retardation is not found in females3-Cytogenetic finding of fragile-X is induced in a vitamin B12 deficiency culture medium4-Affected children are taller than average5-The condition can only be diagnosed after birth.Comments:Affected males are usually but not always mentally retarded .One third of females with the mutation will be mentally retarded. The diagnosis of fragile X syndrome was originally based on the expression of a folate- sensitive fragile X site (X q27.3) induced in cell culture under conditions of folate deprivation. Affected children are taller and may have high arched palate, long ears, a long face and macro orchidism. Diagnosis can be made by detection of the mutant FMR 1 gene by chorionic villus sampling , confirmatory amniocentesis may be required in some cases.

Page 152: Mcqs Pediatrics

MCQs PEDIATRICS

Dr.Kamel Youssef Hassan, Pediatrician Consultant , Palestine - Gaza E-mail: [email protected] 152

1- A 6-month-old previously healthy infant presents with a 2-day history of cough and difficulty breathing. On physical examination, she appears lethargic, pale, and poorly perfused. She exhibits marked tachypnea, and auscultation of the chest reveals decreased breath sounds bilaterally and poor aeration. She has supraclavicular and intercostal retractions.Of the following, the MOST appropriate therapy for this infant isA. continuous positive airway pressure via face maskB. endotracheal administration of surfactantC. endotracheal intubation and positive pressure ventilationD. negative pressure ventilationE. nitric oxide

2- A previously healthy 12-year-old girl presents to the emergency department with sudden onset of wheezing after running through the woods. Findings on physical examination are normal except for wheezing only in the mid-right lung field.Of the following, the MOST likely diagnosis isA. bacterial pneumoniaB. exercise-induced asthmaC. inhaled foreign bodyD. poorly controlled asthmaE. vascular ring

3- During the health supervision visit for a 5-year-old child, his father states that the boy is "always coughing." His mother dismisses this, stating that her son "keeps a cold." Further questioning reveals that the child coughs when he has an upper respiratory tract infection, during exercise, upon exposure to the neighbor's cat, and almost every night. Results of the physical examination are normal.Of the following, the MOST likely diagnosis for this child isA. asthmaB. chronic sinusitisC. cystic fibrosisD. no abnormalityE. postnasal drip

Page 153: Mcqs Pediatrics

MCQs PEDIATRICS

Dr.Kamel Youssef Hassan, Pediatrician Consultant , Palestine - Gaza E-mail: [email protected] 153

4- A 7-year-old girl coughs frequently during a health supervision visit. On further questioning, her mother reports that over the past several years the child has coughed at night, during exercise, and whenever she has an upper respiratory tract infection. A thorough history and physical examination is otherwise unrevealing.In addition to chest radiography and a sweat test, the MOST appropriate initial diagnostic study to determine the etiology of this child's cough isA. allergy testingB. barium swallowC. bronchoscopyD. pulmonary function testingE. skin testing for tuberculosis

5- A 9-year-old girl presents to the emergency depart-ment with an exacerbation of her asthma. She felt warm to touch at home, but is afebrile. Physical examination reveals mild respiratory distress and diffuse expiratory wheezing. Chest radiography shows a consolidation between the fissures on the right side with volume loss.Of the following, the MOST likely interpretation of this radiographic finding would be a(n)A. atelectasis of the right middle lobeB. hyperinflated chest without infiltrateC. pleural effusionD. pneumothoraxE. right middle lobe pneumonia

6- The father of a 12-year-old girl is concerned about her recent lack of interest in physical activity. She says that it is "just too hard, and I get out of breath." Findings from a complete history, physical examination, and chest radiography are normal. An exercise test is terminated when fatigue and breathlessness are induced, but the results of pulmonary function testing remained normal throughout the test.Of the following, The MOST likely explanation for these symptoms isA. asthmaB. cardiac diseaseC. depressionD. muscle weaknessE. sarcoidosis

Page 154: Mcqs Pediatrics

MCQs PEDIATRICS

Dr.Kamel Youssef Hassan, Pediatrician Consultant , Palestine - Gaza E-mail: [email protected] 154

7- A 4-month-old previously healthy infant presents in January with a 2-day history of coughing and breathing difficulty and a 1-day history of refusing to take oral fluids. He has no history of choking. Physical examination reveals a pale infant who has dry mucous membranes, retractions, a respiratory rate of 70 breaths/min, and breath sounds decreased bilaterally with diffuse expiratory wheezing.Of the following, the BEST management isA. administration of intramuscular ceftriaxoneB. administration of nebulized albuterol at homeC. administration of oral albuterolD. hospital admission for cardiorespiratory monitoring

8- A 12-year-old girl presents with signs and symptoms suggestive of asthma. You order pulmonary function studies.Of the following, spirometry is MOST useful to measureA. airway resistanceB. expiratory flow rateC. functional residual capacityD. intrapulmonary shuntingE. static compliance

9- A 2-year-old child who has asthma that is well controlled with home nebulizer therapy suddenly develops coughing after playing with a small plastic toy. Physical examination reveals unlabored breathing, with an intermittent, nonproductive cough and diffuse expiratory wheezing that is greater on the right side.Of the following, the appropriate NEXT step in determining the cause of this patient's symptoms is toA. administer a nebulized bronchodilatorB. arrange for a pH probe studyC. initiate a trial of oral antibioticsD. obtain inspiratory and expiratory chest radiographsE. perform pulmonary function spirometry

Page 155: Mcqs Pediatrics

MCQs PEDIATRICS

Dr.Kamel Youssef Hassan, Pediatrician Consultant , Palestine - Gaza E-mail: [email protected] 155

10- You are asked to assess a hospitalized 3-year-old girl who has asthma and respiratory distress without cyanosis while receiving 3 L/min of oxygen by nasal cannula. Blood pressure and hemoglobin concentration are normal.Of the following, the MOST important factor affecting your decision whether to intubate the child for assisted ventilation isA. arterial PO2B. heart rateC. magnitude of pulsus paradoxusD. respiratory effortE. respiratory rate

11- A 7-year-old child has had persistent nighttime cough for 1 year. Administration of over-the-counter cough syrups has not been helpful.Among the following, the MOST likely cause of the cough isA. foreign bodyB. hyperresponsive airway diseaseC. immotile cilia syndromeD. psychogenic coughE. tuberculosis